X



トップページ数学
1002コメント380KB
高校数学の質問スレ Part419
レス数が1000を超えています。これ以上書き込みはできません。
0001132人目の素数さん垢版2022/05/20(金) 22:46:42.02ID:OqV7vzMH
【質問者必読!!】
まず>>1-4をよく読んでね

数学@5ch掲示板用 掲示板での数学記号の書き方例と一般的な記号の使用例
http://mathmathmath.dotera.net/

・まずは教科書、参考書、web検索などで調べるようにしましょう。(特に基本的な公式など)
・問題の写し間違いには気をつけましょう。
・長い分母分子を含む分数はきちんと括弧でくくりましょう。
  (× x+1/x+2 ;  ○((x+1)/(x+2)) )
・丸文字、顔文字、その他は環境やブラウザによりうまく表示できない場合があります。
 どうしても画像を貼る場合はPCから直接見られるところに見やすい画像を貼ってください。
 ピクトはPCから見られないことがあるので避けてください。
・質問者は名前を騙られたくない場合、トリップを付けましょう。
 (トリップの付け方は 名前(N)に 俺!#oretrip ←適当なトリ)
・質問者は回答者がわかるように問題を書くようにしましょう。
 でないと放置されることがあります。
 (変に省略するより全文書いた方がいい、また説明なく習慣的でない記号を使わないように)
・質問者は何が分からないのか、どこまで考えたのかを明記しましょう。
 それがない場合、放置されることがあります。
 (特に、自分でやってみたのに合わないので教えてほしい、みたいなときは必ず書くように)
・回答者も節度ある回答を心がけてください。
・970くらいになったら次スレを立ててください。

※前スレ
高校数学の質問スレ Part416
http://rio2016.5ch.net/test/read.cgi/math/1644770756/
高校数学の質問スレ Part417
https://rio2016.5ch.net/test/read.cgi/math/1648557700/
高校数学の質問スレ Part418
https://rio2016.5ch.net/test/read.cgi/math/1650534943/
0002132人目の素数さん垢版2022/05/20(金) 23:00:10.20ID:OqV7vzMH
剰余定理で質問なんですけど、割り算で、余りの次数は商の次数よりも低くなりますか?
0003132人目の素数さん垢版2022/05/20(金) 23:02:04.80ID:oi654H9m
ここには面倒なルールは一切ありません。
自由に投稿しましょう。
0004132人目の素数さん垢版2022/05/21(土) 04:21:09.49ID:cSSYrRod
>>2

剰余の定理は、高次式を1次式で割った
商×1次式+余(定数)
を示すこと

次数は当然商より低いし、次数0とかなしとかとらえられる
0005132人目の素数さん垢版2022/05/21(土) 05:20:49.24ID:4TsQhaBr
https://cdn-ak.f.st-hatena.com/images/fotolife/k/k-kawanishi/20200413/20200413115751.png

θの計算方法がうろ覚えです

この式の場合、実際にθを代入して計算するとどういう途中式になるのでしょうか
たとえばr= 1 ,θ= 90° の場合

θは角度なのに()の中をどう代入するのか
sin90°が1なのは理解できるのですが
S = 1/2 × (90°- 1 )
となってしまうのですか?
0006132人目の素数さん垢版2022/05/21(土) 06:14:50.21ID:GwdsIp6q
>>4
xp(x)を(x−3)(x−2)^2で割った時の商をQ(x)、余りをR(x)とすると、
xp(x)=(x−3)(x−2)^2Q(x)+R(x)という式を作るのを目標にするという説明の後に、R(x)の次数<Q(x)の次数と書いてあったんですけど、余りの次数は商の次数よりも小さいとは限らませんよね?
0007132人目の素数さん垢版2022/05/21(土) 06:24:05.09ID:GwdsIp6q
>>4
(1)xの整式p(x)をx−3で割った余りは2、(x−2)^2で 割った余りはx+1である。p(x)を(x−2)^2で割っ た商をq(x)とするとき、q(x)をx−3で割った余り を求めよ。
(2)p(x)は(1)と同じ条件を満たすものとする。この とき、xp(x)を(x−3)(x−2)^2で割った余りを求め よ。

問題文です
0008132人目の素数さん垢版2022/05/21(土) 08:01:50.41ID:cSSYrRod
>>6
紛らわしいな

正式のの計算なら、商の次数が余りの次数より小さいことはあり得るよ

それと剰余の定理は別の話

正式を一次式で割り込んだ時、余は定数になるよね
それで一次式が0になる変数の値の時、元の正式の値は余りの定数になることを、利用しろってこと
0010132人目の素数さん垢版2022/05/21(土) 08:32:55.61ID:cSSYrRod
>>7
答えを知りたいのか、ヒントを知りたいのか?

(1)の結果により
q(x)=r(x)(x-3)-2
と置けることがヒント
0011132人目の素数さん垢版2022/05/21(土) 13:24:23.53ID:PvQneS+V
複素数平面上の相異なる2点A(α)、B(β)を通る直線ABに原点から下ろした垂線の足をH(γ)とする。
γ=αβとなるために、α、βが満たすべき必要十分条件を求めよ。
0012132人目の素数さん垢版2022/05/21(土) 13:28:54.62ID:PvQneS+V
xについての方程式
x^2+(cost)x+sint=0…(*)
について、以下の問に答えよ。

(1)tが0≦t<2πを動くとき、方程式(*)が実数解を持つようなcostの範囲を求めよ。

(2)tが0≦t<2πを動くとき、方程式(*)の解が存在する複素数平面上の領域を図示せよ。
0013132人目の素数さん垢版2022/05/21(土) 13:33:07.82ID:jtYP0lkU
(2)って何の絵を描けばいいの?
0014132人目の素数さん垢版2022/05/21(土) 13:33:09.33ID:PvQneS+V
円に内接する六角形ABCDEFにおいて、3本の対角線AD,BE,CFは1点で交わり、かつ四角形BCEFは正方形であるという。
このとき、この六角形は正六角形と言えるか。
0016132人目の素数さん垢版2022/05/21(土) 13:47:59.61ID:NL+dCxCj
>>3
できました

単調関数の可積分性。fを単調増加とする。Iの任意の分割に対してm=f(x(k-1))、M=f(xk)
0≦S(⊿)-s(⊿)=
Σ[k=1, m](f(xk)-f(x-1))(xk-x(k-1))
≦Σ[k=1, m](f(xk)-f(x-1))d(⊿)
=d(⊿)(f(b)-f(a))→0となるから
リーマンの可積分条件が満たされる。
0017132人目の素数さん垢版2022/05/21(土) 13:58:34.93ID:elf1lior
正解です。
0018132人目の素数さん垢版2022/05/21(土) 14:20:20.94ID:NL+dCxCj
>>14
できました

連続関数の可積分性。コンバクト集合I上で連続な関数fはI上で一様連続であることは証明済み。
従って任意の正数εに対して正数δが存在して|x-y|<δを満たす全てのx, y∈Iに対して|f(x)-f(y)|<εが成り立つ。
d(⊿)<δとなる任意の分割⊿をとると任意の区間|Ik|<δとなるので、x, y∈Ikの時, |f(x)-f(y)|<εとなる。
0≦Mk-mk=sup|f(x)-f(y)| (x, y∈Ik)<εとなる。
0≦S(⊿)-s(⊿)=Σ[k=1, m](Mk-mk)|Ik|<ε(b-a)→0。
ゆえにS(⊿)=s(⊿) (d(⊿)→0)
リーマンの可積分条件によりfはI上可積分である。
0019132人目の素数さん垢版2022/05/21(土) 15:04:50.74ID:GwdsIp6q
>>10
解説にR(x)の次数<Q(x)の次数となることに注意すると書いてあったので、xp(x)の次数が分からないのに、なんでそうなるのかを知りたかったです。
0020132人目の素数さん垢版2022/05/21(土) 15:38:18.20ID:IbYvDcG8
たぶん
・読み間違い
・解説を書いた奴が底抜けのバカ
のどっちか
0021132人目の素数さん垢版2022/05/21(土) 15:46:07.66ID:bk+VeqUG
>>19
で理解したのかな
>>4 と >>8
でりかい
0022132人目の素数さん垢版2022/05/21(土) 15:46:21.76ID:bk+VeqUG
できるはず
0023132人目の素数さん垢版2022/05/21(土) 15:48:17.32ID:bk+VeqUG
ちなみに
xP(x)だと、P(x)より次数は一つ上がるね
0024132人目の素数さん垢版2022/05/21(土) 15:57:23.77ID:8BMkx4bc
質問です
f(x)=x^-1 (0<x)を積分するとF(x)=log(x)が得られるのは納得したのですがグラフから0<x<1の範囲でF(x)は負の値をとっています
F(x)はf(x)とx軸の面積の値ではないのですか?
0026132人目の素数さん垢版2022/05/21(土) 16:17:16.40ID:GwdsIp6q
>>25
解説の画像を載せたので、分かる方いらっしゃいましたら教えてほしいです
0029132人目の素数さん垢版2022/05/21(土) 17:24:43.11ID:NL+dCxCj
できました

リーマン・ルベーグの定理
I=[a, b]、fはI上可積分の時,
lim[t→∞]∫[a, b]f(x)sintadx、
sintxをcostxに変えても同様。
a=bの時, 自明。
a<bの時, 任意の正数εに対して正数δが存在して、d(⊿)<δとなるIの任意の分割⊿に対して
(1) 0≦S(⊿)-s(⊿)<ε/2となる。
fはI上可積分であるからfは有界である。すなわち正数Mが存在して
0≦|f|<Mとなる。a∈I。
今、(1)を満たす分割⊿を1つ固定する。t>0に対して
|costx|≦1、|sintx|≦1
|∫fsin|≦|Σ∫(f-fk)sin|+|Σ∫fksin|
≦Σ(Mk-mk)(xk-x(k-1))+
(M/t)|costxk-cost(k-1)|
≦(S(⊿)-s(⊿))+2mM/t
t0=4mM/εとおくとt≧t0で
|∫fsin|<ε/2+ε/2=ε

Σn/(n^2+k^2)=
Σ[k=1, n](1/n)(1/(1+(k/n)^2))
f(x)=1/(1+x^2)の区間I=[0, 1]をn等分して得られる分割⊿nに関するリーマン和の1つである。代表点ξk=k/nとした。
fはI上単調減少または連続であるから可積分である。積分を実行して、π/4。
0030132人目の素数さん垢版2022/05/21(土) 20:03:01.28ID:AgFpa+Ir
正解です。
0031132人目の素数さん垢版2022/05/22(日) 00:11:30.07ID:0fmBr18t
できました

指数関数の直交性。
複素数に対して内積を入れる。

F'=f、(F○φ)'(t)=F'(φ(t))φ'(t)
=f(φ(t))φ'(t)
φ(t)はJ上連続であるからf(φ(t))はI上連続で可積分であり、φ'(t)はI上可積分であるから
∫f(x)dx=∫f(φ(t))φ'(t)dt
(積分区間は対応して変わる)
変数変換公式、置換積分法の公式

左辺の微分=fg'、
右辺の微分=g'f+gf'-gf'=g'f
部分積分法の公式

f(x)=(1/(x^2+2px+q))
D=0の時, -1/(x+p)
D>0の時, (1/2√(p^2-q))log|(x+p-√)/(x+p+√)|
D<0の時, {1/√(q-p^2)}Arctan((x+p)/√(q-p^2))
0032132人目の素数さん垢版2022/05/22(日) 01:12:44.43ID:0fmBr18t
>>27
できました

x^2/(x+1)^2(x-2)
x^2=A(x-2)+B(x+1)(x-2)+C(x+1)^2
A=-1/3, C=4/9, B=5/9
1/3(x+1)+(5/9)log|x+1|+(4/9)log|x-2|

1/(x^3+1)
1=A(x^2-x+1)+(Bx+C)(x+1)
A=1/3、B=-1/3、C=2/3
(1/3)log|x+1|-(1/6)log|x^2-x+1|
(1/√3)Arctan((2x-1)√3)
0033132人目の素数さん垢版2022/05/22(日) 01:14:27.53ID:+qXDKhxt
>>26
ちょっと解説忘れよう

>>10 は理解できるか?
0034132人目の素数さん垢版2022/05/22(日) 01:20:34.87ID:+qXDKhxt
>>26
で、理解できたら
p(x)をr(x)含んだ式で書いてくれる
0035イナ ◆/7jUdUKiSM 垢版2022/05/22(日) 01:34:25.05ID:IUQ/9Pio
>>12(1)
D=cos^2t-4sint≧0
-2√sint≦cost≦2√sint
∴-2≦cost≦2
こうかな?
0036132人目の素数さん垢版2022/05/22(日) 02:07:52.36ID:4G8gOEyF
そいつは解説の「余りの次数<商の次数」の文言に疑義を呈しているのであって問題自体を教えてくれとは言ってないだろ
言い方は悪いが質問の意図すら理解できないヤツに「理解できるか?」と上から目線で講釈垂れられてるのはさすがに可哀想で見てられない
0037132人目の素数さん垢版2022/05/22(日) 04:47:12.91ID:5dQM5zgU
>>36
お前が教えてやれ
0038132人目の素数さん垢版2022/05/22(日) 05:15:15.55ID:5dQM5zgU
>>36
俺も質問者と同じ疑義持つよ

だけど解説に対して間違いだと言い切る根拠もない
だとしたら、解説考えるのやめて、別の解き方教えるのが良くない?
0039132人目の素数さん垢版2022/05/22(日) 05:21:41.12ID:2UPenvR9
>>38
割る数が3次式なので結局、式を変形して余りを2次式以下にしないといけないことは理解しています。しかし,どうしても余りの次数<商の次数という文言が気になりました。おそらく解決しないようなので、問題の解説自体は大丈夫です。
0040132人目の素数さん垢版2022/05/22(日) 05:23:27.88ID:5dQM5zgU
>>36
R(x)はたかだか2次式確定だけど
Q(x)が1次式であること、解く前に否定できる?
0041132人目の素数さん垢版2022/05/22(日) 05:24:04.82ID:2UPenvR9
>>39
理系数学入試の核心という定番の問題集なんで、流石に誤植というわけではいと思うんですけどね...
0042132人目の素数さん垢版2022/05/22(日) 05:25:39.41ID:5dQM5zgU
>>39
私も正直、疑義持ちます。
でも間違いと言い切る根拠がない。

だとしたら解説無視したらとしか言いようがない。
0044132人目の素数さん垢版2022/05/22(日) 05:38:08.43ID:2UPenvR9
xp(x)かQ(x)の次数のどちらかを確定することができる方法があれば、解決できると思います。
0045132人目の素数さん垢版2022/05/22(日) 05:41:37.98ID:5dQM5zgU
>>44
どうなんだろね
解決できないんじゃない?

初めからわかるって、問題の意義なさないし
余りがたかだか2次式しか、言えないでしょ

問題が解けてるなら、正直言うことなし
0046132人目の素数さん垢版2022/05/22(日) 06:57:27.43ID:5dQM5zgU
>>36
でさ
お前、解説に即した解答してみろよ

可哀想で見てられないんだろ
0047132人目の素数さん垢版2022/05/22(日) 07:07:30.57ID:5dQM5zgU
参考書や問題集って、結構誤植や誤りってあるんだよ
でも、正面切って否定できる根拠がなければ、別の解き方教えるのが建設的なんだって

今回の場合、誤植ってレベルでなく誤りっぽいんだけど、それ証明する手間暇考えたら、別の解き方したらにしたほうがいいんじゃないかって、それだけの話さね
0048132人目の素数さん垢版2022/05/22(日) 08:21:44.44ID:5dQM5zgU
>>18
リーマンって言葉普通の高校生知らんぞ
それ高校数学の話か?
0049132人目の素数さん垢版2022/05/22(日) 09:24:02.23ID:SaGpQLT8
>>24
まず主題とは関係ない間違いとして、1/x(0<x)の原始関数F(x)は
F(x)=log(x)+C(Cは積分定数)だね

んで端的に説明すると、不定積分と定積分を混同してる
不定積分で出てくるのは、原始関数であり、これ自体は面積では無い

面積S=∫[a→b]f(x)dx=F(b)-F(a)だから

つまり面積の符号は、ある点におけるF(x)の符号じゃなくて、原始関数の増減で判断できる

log(x)は狭義単調増加するからF(b)-F(a)は正となって、1/xのグラフを書いた時に期待できる面積の符号が正であることとも合致する
0050132人目の素数さん垢版2022/05/22(日) 09:38:14.45ID:5dQM5zgU
小学生での四角形上の点の速度なんかの問題は
あれ、数学じゃなく、微積や物理の問題だよね

まだ、物理、数学に分化してないレベルだからしょうがないけど
0051132人目の素数さん垢版2022/05/22(日) 10:29:29.11ID:6Sjag5sj
>>26
できました

その解説は誤りである。
以下の解答により、商の次数が不要であることを示す。

mod (x-2)^2(x-3)で考える。
2つの条件より
p(x)≡-2(x-2)^2+x+1とおける。
∴xp(x)≡-2x(x-2)^2+x^2+x
≡-6(x-2)^2+x^2+x
=-5x^2+25x--24 (答え)

この解答には商が出現しない。
この問題は「p(x)を適当な多項式によって分類する問題(剰余類)の問題」なので商は関係ない。従ってその解説は誤りである。
0052132人目の素数さん垢版2022/05/22(日) 11:00:54.46ID:6Sjag5sj
>>24
できました

以下最後まで、x>0、0<a≦bとする。
関数f(x)=1/xは単調減少関数または連続関数なのでx>0の適当なコンパクト集合上で可積分である。

y=1/xとx軸、縦線x=a、x=bで囲まれた部分の面積は∫[a, b]dx/xで表される。→定積分。
この場合、コンパクト集合I=[a, b]というのが前提で、∫[a, b]f=-∫[b, a]fが成り立つ。

ご質問のケースではx>0において
logx=∫[1, x]dt/tから出発して、x>1では正、x<1では負、x=1では0になると考えると良い(不定積分の下端をx=1に固定する)。logxの符号は関数f(x)=1/xをコンパクト集合[1, x]で積分するか[x, 1]で積分するかの違いに相当する。コンパクト集合というのはここでは積分区間(有界閉区間)のこと。
0053132人目の素数さん垢版2022/05/22(日) 11:04:14.78ID:OR576ecB
ついやってもうたてへぺろな間違いじゃなくて、ものすごく頭の悪そうな間違いだよね
0054132人目の素数さん垢版2022/05/22(日) 11:33:18.09ID:5dQM5zgU
>>36 が何に憤りを覚えてんのか知らんが
可哀想と思うなら自分で教えろって

参考書に従った解答示せよ
0055132人目の素数さん垢版2022/05/22(日) 18:36:19.44ID:6Sjag5sj
できました

tan(x/2)=tとおく
dx=2dt/(1+t^2)、cosx=(1-t^
2)/(1+t^2)で変数変換する。
2(1-a)/(1+a)dt/
(1-a)^2/(1+a)^2 +t^2
2Arctan{(1+a)/(1-a) tan(x/2)}

tanx=tとおく
dx=dt/(1+t^2)で変数変換する。
(dt/b^2)/(a^2/b^2+t^2)
(1/ab)Arctan(btanx/a)

t=√(x-α)/(x-β)とおく
x-β=(α-β)/(-t^2+1)
dx=(α-β)2tdt/(1-t^2)^2で変数変換する
dx/t(x-β)=2dt/(1-t^2)
log|(√x-α+√x-β)/(√x-α-√x-β)|
0056132人目の素数さん垢版2022/05/22(日) 20:56:12.23ID:rbw8Nn8J
if関数の意味を教えて下さい
floorとはなんですか?また、どうして数式内に以上、以下があるのでしょうか?
0057132人目の素数さん垢版2022/05/22(日) 22:19:30.30ID:MPZjBhYc
m,nともに任意の自然数であるとき、
10^mn+10^n=1≡mod 10^n-1は値を問わず成り立ちますか?
0058132人目の素数さん垢版2022/05/23(月) 00:13:58.75ID:dNv8OJVf
>>56
エクセルの話かな
①if関数の意味を教えて下さい
→ある条件を設定して合致した時としなかった時で処理を変えてくれる関数

②floorとはなんですか?
→入力された値を任意の基準値の倍数に最も近い値へと端数処理してくれる関数

③どうして数式内に以上、以下があるのでしょうか?
→たとえば会計が1万円「以上」の場合10%割引
9999円「以下」の場合5%割引という条件設定して①で処理する
さらに割引後の金額をうん十円を切って端数をうん百円で揃えて提示したい時②の基準値を100に設定し処理すればよい
0059132人目の素数さん垢版2022/05/23(月) 00:58:35.29ID:dFeHLbX/
>>57
10^mn+10^n=1≡mod 10^n-1は値を問わず成り立ちますか?

→ 10^mn+10^n≡1 (mod 10^n-1)が成り立つかという質問でいいのかな

m=n=1のとき
10^1+10^1=20
mod 10^1-1=9
∴20≡2 (mod 9)

じゃないかな
0061132人目の素数さん垢版2022/05/23(月) 08:27:19.68ID:gX92QYxJ
ともに有理数だと仮定するとtan1°が有理数となり
その倍角も有理数となるのでtan64°とtan4°も有理数となり
加法定理よりtan(64°-4°)=√3が有理数となるので矛盾
0062132人目の素数さん垢版2022/05/23(月) 09:19:04.54ID:MCzd2jTR
結局倍角公式の他に加法定理も使うなら、最初から使えばいいのに
0063132人目の素数さん垢版2022/05/23(月) 09:55:51.59ID:gX92QYxJ
確かに
0065132人目の素数さん垢版2022/05/23(月) 17:08:39.03ID:tLPgIHh6
できました

x+1=1/tとおくとdx=-dt/t^2、
-dt/√(t-1/2)^2+3/4
-log|t-1/2+√(t^2-t+1)|
-log|(-x+1+2√(x^2+x+1))/2(x+1)|

楕円積分((1-x^2)/(1+x^2))/√(1+x^4)
x+1/x=1/tとおくと
((1-x^2)/x^2)dx=dt/t^2
(dt/1-2t^2)=
(1/√2)Arcsin(√2x/(x^2+1)

部分積分法、漸化式
s^(n-1)c'=(n-1)s^(n-2)-ns^(n)
I(n)=(n-1)/nI(n-2)-s^(n-1)c/n
これでn=0, 1, -1, -2に帰着させる

定積分I(n)=((n-1)/n)・I(n-2)
I(2n)=(2n-1)!!/(2n)!!・π/2
I(2n+1)=(2n)!!/(2n+1)!!・1
t=π/2-xと変数変換するとI(n)=J(n)が示せる。

A=[0, 1)でfは連続であるから任意のu∈Aに対してfは[0, u]上可積分である。π/2、広義積分は収束する。

A=[0, ∞)でfは連続であるから任意のu∈Aに対してfは[0, u]上可積分である。広義積分はπ/2に収束する。

A=(0, 1]でfは連続であるから任意のu∈Aに対してfは[u, 1]上可積分である。広義積分は-1に収束する。
0068132人目の素数さん垢版2022/05/24(火) 14:06:00.07ID:oVdCHLxD
半径1、中心Oの円Cがある。
Cの1つの半径上に点AをOA=a(0<a<1)となるようにとる。
C上を点Pが動くとき、△OAPの面積の最大値をaで表せ。
0070132人目の素数さん垢版2022/05/24(火) 14:37:36.10ID:k9EXpS8K
逆数は
1/a[n+1] = n!/a[n] + 1
1,2,5,31,745,89401,64368721...
なんも思いつかんな
無理ちゃう?
0071132人目の素数さん垢版2022/05/24(火) 21:23:36.76ID:CuQTUJu7
>>66
1/a[n]=b[n]と置くと b[1]=1 b[n+1]=b[n]n!+1
b[n+1]/b[n]=n!+1/b[n] このときもし1/b[n]を無視できれば
b[n+1]/b[n]=n! だからb[n]=b[1]Π[k=1,n-1]k!=Π[k=1,n-1]k!
となるがこのΠ[k=1,n-1]k!を簡単にn?とする
(n+1)?/n?=n! 2?=1 さらにb[1]=1となるよう1?=1とする
b[n]をc[n]n?と置くと b[n+1]-b[n]n!=1 における左辺は
c[n+1](n+1)-c[n]n?n!=(n+1)?(c[n+1]-c[n]) だから c[n+1]-c[n]=1/(n+1)?
ゆえにc[n]-c[1]=Σ[k=1,n-1]1/(k+1)? 
そして b[1]=c[1]1? より c[1]=1 だから c[n]=1+Σ[k=2,n]1/k? 
よってb[n]=c[n]n?=(1+Σ[k=2,n]1/k?)n? だから
a[n]=1/Π[k=1,n-1]k!/(1+Σ[k=2,n]1/Π[m=1,k-1]m!)  
0072132人目の素数さん垢版2022/05/24(火) 21:37:37.46ID:CuQTUJu7
>>68
底辺OAがa、高さOPはOAと直交するとき最大で1だから△OAPの最大はa/2
0073132人目の素数さん垢版2022/05/24(火) 21:53:45.52ID:CuQTUJu7
>>71
間違えた
✕c[n+1](n+1)-c[n]n?n!=(n+1)?(c[n+1]-c[n]) だから c[n+1]-c[n]=1/(n+1)?
○c[n+1](n+1)?-c[n]n?n!=(n+1)?(c[n+1]-c[n]) だから c[n+1]-c[n]=1/(n+1)?
0075132人目の素数さん垢版2022/05/25(水) 08:44:40.58ID:S6YQpB3H
実数から実数への、恒等関数でない関数f(x)で、3回合成関数f(f(f(x)))が恒等関数になるものはありますか?
0076132人目の素数さん垢版2022/05/25(水) 11:17:46.58ID:QMliOGhF
>>75
ないんじゃないの
三次関数の解の公式求める際に同じこと議論しないかい
0078132人目の素数さん垢版2022/05/25(水) 12:48:28.51ID:QMliOGhF
>>77
ゴメン、それ恒等関数?
0079132人目の素数さん垢版2022/05/25(水) 13:07:17.64ID:bIFS35BC
f○f(x)で、x-1が分母に表れてしまうんですね。手直しします。、

x=0 のとき 0 で、x=1のとき 1で、それ以外では 1-1/x
0080132人目の素数さん垢版2022/05/25(水) 13:18:15.95ID:pOQ8KVVo
要するにRを3つA,B,Cに分けて全単射p : A→B、q : B→Cをとってきて r = (qp)⁻¹と定めて
f(x) = p(x) ( if x ∈ A )
q(x) ( if x ∈ B )
r(x) ( if x ∈ C )
にすればいい
A = ∪[3n,3n+1)
B = A+1,
C = B+1
でp,qはx+1をA,Bに制限してすればいい
0081イナ ◆/7jUdUKiSM 垢版2022/05/25(水) 17:56:03.52ID:4GcGXpmg
△OAPにおいて∠POAをθ(0°≦θ≦360°)とおくと、
△OAP=(1/2)OA・OPsinθ=asinθ/2
θ=90°,270°のとき
△OAP=a/2(最大値)
sin270°=-1だけど図は最大だからOK
0082132人目の素数さん垢版2022/05/26(木) 15:10:35.21ID:q9MoU3q6
a[1]=1
a[n+1]={a[n]}^3+1
で与えられる数列がある。
このときa[n]をa[k](k=1,...,n-1)で割った余りを求めよ。
0084132人目の素数さん垢版2022/05/26(木) 18:51:14.66ID:680pz756
今日明日の天気、週間天気、ピンポイント天気で共通する主な天気マークです。
1晴れ
2くもり
雨は降らない
3雨
4雪
5くもり
雨の可能性あり
6大雨
7大雪
8暴風雨
9暴風雪
この9つのうち全てのパターンを教えてください。

なお、主な予報の種類は以下のパターンになります。

『のち』: 前半と後半で天気が変化するときに用います。

『時々』: 天気が断続的に変わります。断続的な天気の合計時間は予報期間の1/2未満です。

『一時』: 一時的に天気が変わります。一時的な天気は予報期間の1/4未満です。
0085132人目の素数さん垢版2022/05/26(木) 18:56:28.11ID:OQtwIc5s
ルールが全く分からん
使える単語は
雨のち雨一時雨時々雨
とかもいいのか?
晴れ一時曇り時々雨
とかもいいの?
つまり“のち”、“時々”、“一時”は好きな回数使ってよくて“晴れ”、“雨”、“曇り”は自由に入れていいの?
こんな気象用語のルールなんか知らないよ
0086132人目の素数さん垢版2022/05/26(木) 19:01:05.91ID:OQtwIc5s
例えば基本晴れだけど断続的な雨が1/10くらいはあって一時的に1時間ほどずっと雨の状態が1/10両方ある場合に「晴れ時々雨一時雨」もありなん?
0087132人目の素数さん垢版2022/05/26(木) 19:44:31.89ID:680pz756
>>85
失礼。
言いたいことは雨のち晴れとか
晴れのち雨とかに使います。
ただ雨のち雨は無効でお願いします。
それなら雨のみなので。
また雨ときどき曇りとか
曇りときどき雨とかも含まれます。
また一時雨とかそういうものも含まれます。
他に質問はないでしょうか?
宜しくお願いします。
0088132人目の素数さん垢版2022/05/26(木) 19:46:02.33ID:680pz756
>>86
基本晴れなら晴れ一時雨です。
宜しくお願いします。
0089132人目の素数さん垢版2022/05/26(木) 20:38:43.24ID:eAFLqPiP
>>87
イヤ、分からんところ山ほどあるけど
例えば
・午前中は基本晴れ、一時的に雨、でも曇りになる時もちらほら
・午前中は基本雨、一時的に晴れ、でも曇りになる時もちらほら
なら

晴れ一時雨時々曇り、のち雨一時晴れ時々曇り

はありなん?
とりあえずホントの気象庁のルールはともかくとして、数学の組み合わせの問題として答え出したいならその辺のルールは自分でキチンと決めとかないと答えなんか出せるはず無いよ
基本ルールは
・A、A一時B、A時々C、A一時B時々Cの4パターン、そこにA,B,Cに晴れ、雨、曇りが入る(同じのは入らない)
・上でできる3+6+6+6=21パターンが午前と午後で切り替わって21×20通りもあり得る
だけ?他にない?
0090132人目の素数さん垢版2022/05/26(木) 21:09:43.40ID:15hU6TaT
nについての帰納法は

induction on n ですか
induction over n ですか
0091132人目の素数さん垢版2022/05/26(木) 21:19:50.04ID:680pz756
>>89
・午前中は基本晴れ、一時的に雨、でも曇りになる時もちらほら
こちらは晴れのち雨もしくは曇りで大丈夫そうです。
基本条件としては定義をするなら

前回にあるように

『のち』: 前半と後半で天気が変化するときに用います。
『時々』: 天気が断続的に変わります。断続的な天気の合計時間は予報期間の1/2未満です。
『一時』: 一時的に天気が変わります。一時的な天気は予報期間の1/4未満です。
これだと9かける8=72通りが3つで216通りで
あったますでしょうか?自信がないです。
0093132人目の素数さん垢版2022/05/27(金) 05:22:35.94ID:iVuENKxY
>>91
考えてみたけどもっとめんどくさい

9種類の果物で100mlのミックスジュースを作る
この時配合割合によって名前が変わる
ベース=b
時々=c
一時=d

1種類=9
2種類=9*8*3
3種類bcc=9*(8C2)
3種類bcd=9*8*7
3種類bdd=9*(8C2)

4種類、この時25パーずつをどう捉えるか

天気に戻すと
晴れ、雨、雪、くもりがそれぞれ6時間の場合
これをccccとしてカウントするのか
bcccとして「晴れ時々雨時々雪時々くもり」
「雨時々晴れ時々雪時々くもり」と朝一の天気で名前を変えるのか

とここまで考えたがめんどくさくなって諦めた
もっと簡単な考え方があるかも?
0094132人目の素数さん垢版2022/05/27(金) 07:31:52.31ID:HjXXWazB
時刻0に、xy平面上の原点(0,0)が赤く塗られている。
時刻n+1において、時刻nの時点で赤く塗られている格子点に隣接する格子点を赤く塗る。
たとえば時刻1においては、4点(1,0),(0,1),(-1,0),(0,-1)が赤く塗られる。
時刻nにおいて、何個の格子点が赤く塗られているか。nで表せ。
0096132人目の素数さん垢版2022/05/27(金) 09:44:44.16ID:Pih5AOEH
>>93
んあー
3種類の時も4:4:2の時をbcdとしてカウントしちゃってるしやっぱりこれじゃダメだ
0097132人目の素数さん垢版2022/05/27(金) 09:49:38.71ID:l3ieoOtU
時刻tに第一象限にある赤い点の個数をa(t)とする
原点でないx軸上の赤い点の個数をb(t)、原点でないy軸上のそれをc(t)とする
a(1)=0、b(n)=n、c(n)=n は明らか
a(t+1)は時点tにおける第一象限と原点でないx軸上にある赤点全体を一つ上に
ずらしたものなのでa(t+1)=a(t)+b(t)=a(t)+t
ゆえにn>1のとき a(n)=a(1)+Σ[t=1,n-1]n=n(n-1)/2
題意の数は n>1のとき4a(n)+2b(n)+2(c)+1=2n(n-1)+2n+2n+1=2n(n+1)+1
0098132人目の素数さん垢版2022/05/27(金) 09:53:10.11ID:l3ieoOtU
間違えた
✕原点でないx軸上の赤い点の個数をb(t)
○x軸上の正の部分にある赤い点の個数をb(t)
0099132人目の素数さん垢版2022/05/27(金) 14:39:23.85ID:13SLrMxC
1種類=9
2種類=9*8*3
3種類bcc=9*(8C2)
3種類bcd=9*8*7
3種類bdd=9*(8C2)

これの合計ってわかりますか?
0100132人目の素数さん垢版2022/05/27(金) 15:23:36.90ID:L3HckGkK
>>99
> 1種類=9
> 2種類=9*8*3=216
> 3種類bcc=9*(8C2)=252
> 3種類bcd=9*8*7=504
> 3種類bdd=9*(8C2)=252

合計=1233だけども

上は晴れ4割くもり4割雨2割とかの時が場合分けされてる総パターンで以降も4種類〜9種類までの場合分けも考えなきゃいけないしあんまり役立たないと思う
なんかのテキストの問題なら多分他の計算方法があるはず
0101132人目の素数さん垢版2022/05/27(金) 16:12:37.02ID:13SLrMxC
100さん。
ありがとうございます。
合計=1233ですね。
0102132人目の素数さん垢版2022/05/29(日) 12:34:10.68ID:1uOwMRWh
家に松坂和夫の解析入門という
全6巻の本があります。
父が昔読んだそうです。
30年ぐらい前の本です。

これは今売ってる松坂の数学入門全6巻と
内容は同じでしょうか?

どうして解析入門から数学入門へと
タイトル変えたのでしょうか?
0103132人目の素数さん垢版2022/05/29(日) 16:14:43.44ID:PU522fyq
a,b,c,dは自然数でA(a,b),B(a+c,b+d).C(c,d).O(0,0)とする。これらを頂点とする平行四辺形OABCの周を除いた内部をSとするとき、
ad-bc=2のとき、Sの中に格子点があれば、それは平行四辺形の対角線の交点であることを示せ
0105132人目の素数さん垢版2022/05/29(日) 17:00:07.64ID:r53RALY0
>>104

x-y が因数になるよ
0106132人目の素数さん垢版2022/05/29(日) 17:01:41.96ID:r53RALY0
1項2項5項と3項4項を分けて考える
0107132人目の素数さん垢版2022/05/29(日) 17:04:58.52ID:EL738sdl
↑(a,b)と↑(c,d)の張る平行四辺形の面積の平方は両ベクトルのなす角をtとするとき
両ベクトルの長さの二乗の積*(sint)^2=両ベクトルの長さの二乗の積*内積の二乗
=(a^2+b^2)(c^2+d^2)-(ac+bd)^2=(ad-bc)^2
ゆえに格子点が作る平行四辺形の面積の最小は1で格子点が作る三角形の最小は1/2
そしてad-bcは題意の平行四辺形の面積となるので面積は2

仮にSに格子点pがあるとすると△POA、△POB、△POC、△PODがあるが
これら面積の和が題意の平行四辺形の面積である2を超えないためには
これら4つの三角形の面積は全て等しく1/2であることが必要
もしpが対角線の交点になければ面積の等しくない三角形があることになるので矛盾
0108132人目の素数さん垢版2022/05/29(日) 17:07:54.42ID:QVOPcRt4
>>107
自然数で三角形の最小値は1/2にはならんやろ

考え方はいいけど
0109132人目の素数さん垢版2022/05/29(日) 17:15:20.51ID:QVOPcRt4
あ、なるか、すまん、ゴメン
0110132人目の素数さん垢版2022/05/29(日) 17:34:57.73ID:XoniIjXD
>>107
ありがとうございます。

一つ質問なんですが、解答だとOAを底辺としてPの高さがCの高さの半分…@、OCを底辺と見ればPの高さはAの高さの半分…A
この2つの点を満たすPは明らかに平行四辺形なら対角線の交点である。

と解答に書いてあるんですがなんかこの@かつAっていうのがいまいちしっくりこないです。@またはAじゃないですか?@を考えたときに高さ半分のところに格子点があれば良いだけであって一つしか格子点がないとも限らなくないですか?
申し訳ないんですけど回答お願いします。
0111132人目の素数さん垢版2022/05/29(日) 17:39:57.24ID:XoniIjXD
文字化けすみません

一つ質問なんですが、解答だとOAを底辺としてPの高さがCの高さの半分…1、OCを底辺と見ればPの高さはAの高さの半分…2
この2つの点を満たすPは明らかに平行四辺形なら対角線の交点である。

と解答に書いてあるんですがなんかこの1かつ2っていうのがいまいちしっくりこないです。1または2じゃないですか?1を考えたときに高さ半分のところに格子点があれば良いだけであってその直線上に一つしか格子点がないとも限らなくないですか?
0112132人目の素数さん垢版2022/05/29(日) 17:52:18.62ID:EL738sdl
間違えた
✕△POA、△POB、△POC、△POD
○△POA、△PAB、△PBC、△PCO

>>110
>@またはAじゃないですか?
@だけだと△POA=△BPCが言えるだけでなのでもしAを満たさない場合は
△PAB=△PCOが言えなくなるのでこのうちのどちらかは1/2を超える
つまり4つの三角形の面積の和は2を超えて矛盾なのでAも満たす必要がある
0114132人目の素数さん垢版2022/05/29(日) 17:59:55.03ID:B+rRnRn4
「明らかに」がいい感じに胡散臭さにのスパイスになってるね
0115132人目の素数さん垢版2022/05/29(日) 18:07:33.62ID:XoniIjXD
あー格子点の三角形の面積の最小値が1/2なのに=じゃないって事はもう片方は1/2よりは大きくなるって事か
結果的にかつじゃないとダメって事ですね
わかりました。ありがとうございます。
他の回答してくれた方もありがとうございます。
0116132人目の素数さん垢版2022/05/29(日) 18:40:21.05ID:w7lNKTnm
>>105

その因数を使ってどのように因数分解するんでしょうか?
0117132人目の素数さん垢版2022/05/29(日) 18:41:28.87ID:ujVLAYR/
あ、ごめんなさい>>116ですがやっぱ出来そうです
ありがとうございました
0118132人目の素数さん垢版2022/05/29(日) 18:46:44.93ID:Z9Rnfc+H
>>104が(x-y)二乗-z(x-y)になったのですがこれで完成ですか?
あとどうすれば良いでしょうか
0119132人目の素数さん垢版2022/05/29(日) 18:48:26.15ID:EL738sdl
よく見ろ
0120132人目の素数さん垢版2022/05/29(日) 18:49:41.51ID:KiboFTWA
ここから括り方が分かりません
0121132人目の素数さん垢版2022/05/29(日) 19:18:55.52ID:sKENTqzl
放物線y=x^2で、点(0,0)は頂点と呼ばれますが、双曲線やだ円でも頂点と呼べる点はありますか。

たとえば双曲線xy-1=0では(±1, ±1)は頂点でしょうか。
だ円x^2+(y/2)^2=1で(±1,0)や(0,±2)は頂点でしょうか。
0124132人目の素数さん垢版2022/05/29(日) 23:48:35.92ID:amc6gREE
仮説検定の基準となる確率って
いったいどこから湧いてくるんですか?
(´・ω・`)
0125132人目の素数さん垢版2022/05/30(月) 00:08:04.79ID:5taxJkGY
>>124
1%,5%のこと?
0127132人目の素数さん垢版2022/05/30(月) 03:52:42.02ID:8N1SDg/2
>>122
>>126
ありがとうございます!
0128132人目の素数さん垢版2022/05/30(月) 19:00:02.65ID:Ltmhtv2W
実数xおよび自然数nが与えられたとき、
Σ(k=1,n-1)[x+k/n]=[nx]
が成り立つ事を証明せよ。

解答で
x=m+a/n(0≦a<n,[a]=u)と置いて解いてるのですがこう置いても問題ないのはなぜですか?
0129132人目の素数さん垢版2022/05/30(月) 19:38:35.70ID:/K5FNeRC
放物線C:y=x^2上に2点A(a,a^2),B(a+1,(a+1)^2)をとり、C上に∠APB=90°となるように点Pをとる。
このような点Pは各aの値に対して何個あるか。
0130132人目の素数さん垢版2022/05/30(月) 22:38:51.65ID:sujSMj+B
正の実数x,y,zが
(x+1/x)^2+(y+1/y)^2+(z+1/z)^2=4+(x+1/x)(y+1/y)(z+1/z)
を満たすとき, xyz=1を示せ
0132132人目の素数さん垢版2022/05/31(火) 05:43:43.94ID:A+YmhEqq
一辺の長さが1の正五角形の周および内部に含まれる正方形で、面積最大のものの面積を求めよ。
0133132人目の素数さん垢版2022/05/31(火) 10:43:59.46ID:IdmCutAz
この証明をお願いします
sin(x) cos(x) (tan(y) + tan(x + y)) - sin(y) cos(y) (tan(x) + tan(x + y)) = sin(x + y) cos(x + y) (tan(x) - tan(y))
0135132人目の素数さん垢版2022/05/31(火) 12:26:28.00ID:IdmCutAz
これ因数分解できるのか、、手計算の加法定理や和作積公式の変形は嵌ってぜんぜんできんが、
sin(x) cos(x) (tan(y) + tan(u)) - sin(y) cos(y) (tan(x) + tan(u)) - sin(u) cos(u) (tan(x) - tan(y))
=sec(u) sec(x) sec(y) sin(u + x) sin(u + y) sin(x - y) sin(u - x - y)

u= x+y のとき sin(u - x - y)=0
0136132人目の素数さん垢版2022/05/31(火) 12:43:05.20ID:IdmCutAz
重心と垂心と外心の重心座標から3点が一直線上(オイラー線)にあることの証明のために
行列式を手計算で出来なかったのでした.
Determinant({{1,1,1},{sin(2A),sin(2B),sin(2C)},{tan(A),tan(B),tan(C)}})=
-2 sec(A) sec(B) sec(C) sin(A - B) sin(A - C) sin(B - C) sin(A + B + C)
0139132人目の素数さん垢版2022/06/01(水) 06:50:52.29ID:AbZqwpel
>>133>>135
>この証明をお願いします
>sin(x) cos(x) (tan(y) + tan(x + y)) - sin(y) cos(y) (tan(x) + tan(x + y)) = sin(x + y) cos(x + y) (tan(x) - tan(y))

まず、方針を立てた方が良い。チャート式として
<チャート式>
1)式をにらむ。式複雑。で等号証明問題だと分かる
2)複雑→簡単 の式変形は楽。例えば、二つの式のかけ算を展開するとかは単純計算だ。逆は難しい、例えば因数分解は展開より難しい。
3)等号証明問題の場合、a)左辺→右辺、b)右辺→左辺、c)左辺→簡単な左辺、右辺→簡単な右辺として、簡単な左辺=簡単な右辺を示す

<具体的当て嵌め>
1)再度式をにらむ、まず右辺 sin(x + y) cos(x + y) は加法定理で、二つの式のかけ算にして展開する
 左辺 tan(x + y)も、加法定理でバラス。但し、このとき tan(x + y)=sin(x + y)/cos(x + y)とする方が良いことに気づく(下記)
 tan(x)とかtan(y)も、sin(x)/cos(x)、 sin(y)/cos(y) にして、sinとcos の式だけにして単純化するべし
(単純化がキーワードです。tan(x)=sin(x)/cos(x)は気づかないと。テクニックとして常に意識するべし)
2)上記1)の方針で、左辺をばらして、sin cos の順で、べきも昇べき順に整理するべし。右辺も同様
 整理した左辺と右辺を比べる。「同じ式になった。よって等号成立。」と書く
3)”これ因数分解できるのか”>>135 は、受験テクニックとしては、方針違い。積の展開を優先すべき。受験外として、因数分解を考えるのは楽しいかも

>>134
WOLFRAM か。面白いね
”結果 0”とあるから、上記方針でばらしていけば、0じゃないかな

通りすがりですが
この方針でやれば、出来るはず(やってないけど)
やってみてください
0140132人目の素数さん垢版2022/06/01(水) 06:55:51.01ID:AbZqwpel
>>139
> 3)等号証明問題の場合、a)左辺→右辺、b)右辺→左辺、c)左辺→簡単な左辺、右辺→簡単な右辺として、簡単な左辺=簡単な右辺を示す

補足
等号証明問題の場合、方針が3つあるってことね
・左辺が複雑で、右辺が簡単なら、左辺→右辺
・逆なら、右辺→左辺 (同じ等式でも、こっちの方が 少しだけ難易度上かも)
・両方複雑ならば、複雑→簡単 の方針で、各辺をばらして比較する
ってことね
0141132人目の素数さん垢版2022/06/01(水) 12:31:13.79ID:0PtNjg1m
任意の正整数nに対して、方程式
x^n-nx+p=0
が整数解を持つような0でない整数pが存在することを示せ。
また、そのようなpは無数に存在するかどうか答えよ。
0143132人目の素数さん垢版2022/06/01(水) 22:23:19.85ID:rN79qJpo
「音声の時代が来る」ボイシーCEO、緒方憲太郎さん

音声の時代を予感したのは、技術への着眼もある。
「動画は映画、テレビ、ユーチューブ、ティックトックと
新しいフォーマットが次々と出てきたが、音声はラジオのままだった」
ネットの時代に合った音声フォーマットを作れば、需要はあると
踏んだのだ。歩きながらでも、家事をしながらでも、音声は聞く
ことができる。テキストや映像より身体拘束が少ない。
「歴史的に人は、情報を得る手間を省いてきた。情報との接点が
より自然になると、音声はもっと生活の中に入ってくると思う」
ボイシーを起業後、スマートスピーカーやワイヤレスイヤホンなど、
音声機器が急速に浸透してきた。
音声との接点が増え、追い風が強くなっている。時代があとから
ついてくるのは、起業家の醍醐味(だいごみ)だろう。
月収600万円を稼ぐパーソナリティーも出てきた。
0145132人目の素数さん垢版2022/06/02(木) 14:44:15.02ID:UO/0CKTZ
なんであっても因数分解はできるんじゃね?
0146132人目の素数さん垢版2022/06/02(木) 15:19:41.60ID:ki9A3cdf
pが素数のとき、C[p-1,k] (0≦k≦p-1) はmod p で1か-1になるといえますか
0148132人目の素数さん垢版2022/06/02(木) 20:30:43.59ID:ZoEZCLWC
甥っ子に質問受けて全然覚えておらず、、、どなたか助けてくださいm(__)m

・1/10の確率で当たるくじを30回引くとき、当たりくじの本数がa本となる
確率、期待値、分散、標準偏差を求めよ。

確率が30Ca×(1/10)*a×(9/10)*(30-a)だとは思うのだけど、期待値でΣに
落とし込むのがもう思い出せぬ。ましてや文系出身で分散、標準偏差もほぼ壊滅、
お願いしますm(__)m
0150132人目の素数さん垢版2022/06/02(木) 23:46:01.46ID:S1fuojX4
aの取りうる値全体を確率変数と考えれば
値は求まるでしょ

まあ、次からは
問題文を正確に書いた方がいい
0151132人目の素数さん垢版2022/06/03(金) 01:09:23.57ID:4jOZ8hqh
>>150
その用語の使い方もおかしい
まずそもそもこんな数学Aで習う範囲の用語すらまともに意味わかって無くて理解なんぞできるハズない
そんな付け焼き刃で身内とはいえど数学指導するなどもってのほか
0152132人目の素数さん垢版2022/06/03(金) 02:24:50.29ID:2KubHTLJ
>>148の質問自体は普通に意味の通る質問でしょう。

>>149 はこのスレで回答者となるためにはちょっとエスパー能力が低いかな。
>>150 は確率変数という言葉を不用意に使っている点で、うっ? だね。
0153132人目の素数さん垢版2022/06/03(金) 02:27:51.22ID:MAWxPZk8
>>152
問題の意味ももちろんわかる
しかしこのレベルの質問者に答えの出し方だけ教えてもかえって逆効果
あまりにもひどすぎる
そもそも指導してるなら手元に高校の教科書あるんやろそこに書いてあるレベルのしかもごく初歩の用語がわかってない
おそらくこのレベルだと教わってる方がわかってる
こんな奴に教えても無駄
0154132人目の素数さん垢版2022/06/03(金) 07:28:57.24ID:QzpiuYaY
>>148
>・1/10の確率で当たるくじを30回引くとき、当たりくじの本数がa本となる
>確率が30Ca×(1/10)*a×(9/10)*(30-a)だとは思うのだけど

探せば、どこかにあると思うが、
すぐ見つからず 面倒なので下記をば

1)まず、小さい数で試すことから始める
 この場合、くじを戻す(下記)と考えて
 xが当り、y外れとして (x=0.1, y=0.9, x+y=1)
 1回試行なら、当りか外れかで、当り確率x=0.1
 2回試行なら、1回当りは 先にxの後yか、先にyの後xかで、xyとyx で、2*0.1*0.9=0.18 注1
 因みに、xxは、0.1^2=0.01。yyは、0.9^2=0.81。xx+2xy(xyとyx)+yy=0.18+0.01+0.81=1成立
 これは、(x+y)^2=x^2+2xy+y^2 と2項展開になっています
 3回試行なら、2回当りは 上記同様 xxy,xyx,yxx の3通りで、3xxy=3*0.1*0.1*0.9=0.009*3=0.027
 略すけど、2回試行と同様に、この場合も、2項展開 (x+y)^3=x^3+3xxy+3xyy++y^3 と試行の場合分けが一致します
 その合計は、x+y=1から、(x+y)^3=1が従います
2)よって、一般n回試行でr回当りの確率は、n次の2項展開 (x+y)^n で、x^r の係数に等しいと分かるので
 その確率は、nCr*(x^r)*(y^n-r) 注2
3)これを、元の問題に当て嵌めると、「1/10の確率で当たるくじを30回引くとき、当たりくじの本数がa本となる確率」
 は、30C5*(x^5)*(y^25)=30C5*(0.1^5)*(0.9^25) となる
 余談ですが、エクセルでそのまま計算させると、精度がちょっと心配かも(桁あふれとかに気をつけて、うまく順番を調整して計算するのが良さそう)
 (上記の「30Ca×(1/10)*a×(9/10)*(30-a)」は、ちがうね)

注1:*は積の記号で、エクセル記法と同じ
注2:(y^n-r)は、(y^(n-r))の略記です。カッコが多すぎると、みにくいので、略記した。手書きだと指数がカッコなしで 肩に書けるが、ここでは書けないので。

つづく
0155132人目の素数さん垢版2022/06/03(金) 07:29:30.42ID:QzpiuYaY
>>154

つづき

(参考)
https://hs-math.komaro.net/kakuritsu-kuzibiki/
高校数学の無料オンライン学習サイトko-su-
数学A確率くじ引きの確率・確率の総合問題

https://study-line.com/kakuritsu-kuji/
数スタ 中学2年生確率【くじ引きの確率】くじを戻す、戻さないそれぞれの問題を解説!

https://sukinakazu.net/tokikata/kakuritu-tokikata-kuji.html
確率の求め方・くじ1
●くじの問題パターン
●くじを戻すときと戻さないときの違い
●くじを戻すときの樹形図の書き方
●くじを戻す問題
●くじを戻す問題の解き方1
●くじを戻すときの樹形図
●くじを戻す問題の解き方2

https://qiita.com/yz2cm/items/03261054b718af032205
Qiita @yz2cm 更新日 2015年06月07日
場合の数(nPr,nCrの意味について)

https://ja.wikipedia.org/wiki/%E4%BA%8C%E9%A0%85%E4%BF%82%E6%95%B0
二項係数
歴史と記法
nCk などがあり、何れも文字 C は組合せ (combination) や選択 (choice) を表している。
(引用終り)
以上
0156132人目の素数さん垢版2022/06/03(金) 07:47:33.34ID:QzpiuYaY
>>154 訂正です

 は、30C5*(x^5)*(y^25)=30C5*(0.1^5)*(0.9^25) となる
  ↓
 は、30Ca*(x^a)*(y^(30-a))=30a*(0.1^a)*(0.9^(30-a)) となる

です
30C5*(x^5)*(y^25)=30C5*(0.1^5)*(0.9^25) は、a=5の場合です
a=5を試しに考えたりしていたので、まちがった
お詫びして、訂正します
0157132人目の素数さん垢版2022/06/03(金) 09:30:04.02ID:H+1A+40N
>>148
二項分布の期待値と分散は検索すればいろんなところに
証明がのってるけど、npとnp(1-p)になる。簡単に言うと、
期待値は1回当たりの期待値がpなので、試行回数分足し合わせてnp。
分散も1回当たりの分散p(1-p)^2+p^2(1-p)=p(1-p) をn回分足し合わせるだけ。

よって、期待値は30*(1/10)=3、分散は 30*(1/10)*(9/10)=2.7
0158132人目の素数さん垢版2022/06/03(金) 10:23:20.17ID:+ML75ssy
>>154
> (上記の「30Ca×(1/10)*a×(9/10)*(30-a)」は、ちがうね)

ここ
ひょっとして、(1/10)*aが べき (1/10)^a の意図なら、正しいね
ただ、べき は普通 ^ か、または** ですね(下記)
なお、* 一つだけだと 積の意味です。普通は(下記)

(参考)
https://math-fun.net/20191030/3305/
趣味の大学数学
数学における^記号の意味、読み方は?
2019年10月30日2020年5月4日
数学における^記号の意味、読み方
^記号は、数学においてはべき乗(掛け算の繰り返し)の指数を表すために使われます。
^記号は、サーカムフレックス、キャレット、ハット記号と呼ばれます。
僕の日本語話者との経験では、ハット記号と呼ぶのが一番伝わりやすいかと。ただし、3^2と書かれていたら、「さんのにじょう」と読むでしょうが。
コンピュータにおける計算でも、ベキを表すために、^または**が使われます。
ワードソフトやTEXを使わない普通の文章では、上付き文字を使うことができません

つづく
0159132人目の素数さん垢版2022/06/03(金) 10:24:26.13ID:+ML75ssy
>>158
つづき

https://www.nli-research.co.jp/report/detail/id=62419?site=nli
シンクタンクならニッセイ基礎研究所 > 保険 > 保険計理 > 数学記号の由来について(1) 四則演算の記号(+、-、×、÷)
2019年09月02日 保険研究部 研究理事 中村 亮一
第1回目の今回は、四則演算の記号(+、-、×、÷)の由来について、報告する(なお、実際のより詳しい記号の歴史や経緯等については、脚注に掲げた米国の数学者、数学史家のフロリアン・カジョリ(Florian Cajori)の文献1等を参考にしていただくことにして、ここでは筆者の判断に基づいて、ポイントのみを報告している(次回以降の報告でも同様である))。
1 主として、以下の文献を参考にした。
Florian Cajori「A History of Mathematical Notations」(1928、1929)の冊子の再発行版(2012)(Dover Publications Inc.)

掛け算(かける)記号のその他の例
さて、掛け算を表す記号には、「×」以外にも、例えば「・」(ドット)という記号が用いられることもある。むしろ、「・」の方が「×」よりも早くから使用されていたようである。
「・」は、有名なドイツの数学者であるゴットフリート・ライプニッツ(Gottfried Wilhelm Leibniz)によって、掛け算の記号として提唱されたと言われているようである(これにも異論があるようだが、ここでは述べない)。
「*」(アスタリスク)も、掛け算の記号として使用されることがある。これは、1659年に、スイスの数学者ヨハン・ハインリッヒ・ラーン(Johann Heinrich Rahn)の代数学の著書「Teutsche Algebra」において使用された。
因みに、Microsoft社のExcelでは、掛け算は「*」の記号が使用されている。
(引用終り)
以上 
0160132人目の素数さん垢版2022/06/03(金) 11:14:02.05ID:FKNPUJPa
引くとプラス1される赤玉
引くとマイナス1される黒玉が袋の中に52:48の割合て入っていて
それを無造作に200回引いて最終的にマイナスの値になる確率って何%ですか?
0161132人目の素数さん垢版2022/06/03(金) 11:53:17.82ID:LHyOAYvq
(a+b+c)(ab+bc+ca)+kabc
が整数係数の1次以上の多項式で因数分解できるとき、整数kの値をすべて求めよ。
0163132人目の素数さん垢版2022/06/03(金) 13:00:00.81ID:tysSY7+l
(a+b+c)(ab+ac+bc)+kabc=(a+b+c)p(a,b,c).
k=0.

(a+b+c)(ab+ac+bc)+kabc=(pa+qb+qc)(qa+pb+qc)(qa+qb+pc).
pq^2=0.
p=0.
q^3=1.
k=-1.
0165132人目の素数さん垢版2022/06/03(金) 14:21:36.69ID:+l+lqrds
対称一次因子を持つ→k=0
対称一次因子なし→一次因子3つが移り合う→因子はp(a+b+c)-aの形しか許されない
(p(a+b+c)-a)(p(a+b+c)-b)(p(a+b+c)-amc)
=p³t₁³-p²s₁t₁²+ps₂t¹-s₃
=s₁s₂+ks₃
p=1,k=-1
0166132人目の素数さん垢版2022/06/03(金) 14:25:34.38ID:wB+C7Hb6
>>161
与式に b=-1, c=2 を代入すると a^2 - (2k+1)a -2 であるが、これも同様の因数分解ができなくてはならず、
したがって (a+1)(a-2) または (a-1)(a+2) でなければならない。これらは、それぞれ k=0, k=-1 の場合である。
与式は、k=0 のとき (a+b+c)(ab+bc+ca)、k=-1 のとき (a+b)(b+c)(c+a) とどちらの場合も因数分解できる。
したがって、答えは k=0, -1。
0167132人目の素数さん垢版2022/06/03(金) 19:56:08.85ID:3jMDD8Ot
整数a[1],...,a[m]を用いて
a[m]*m!+a[m-1]*(m-1)!+...+a[1]
と表される整数を{a[m],...,a[1]}と書く。

(1){a[m],...,a[1]}と表示される整数はただ1つに定まることを示せ。

(2)任意の整数は{a[m],...,a[1]}の形に書けることを示せ。
0168132人目の素数さん垢版2022/06/03(金) 21:21:18.24ID:QzpiuYaY
>>164-166
ちょっと考えてみたけど
この証明は、高校数学を超えている気がするが、どう?

>>165
>対称一次因子なし→一次因子3つが移り合う→因子はp(a+b+c)-aの形しか許されない

・「対称一次因子なし→一次因子3つが移り合う」の証明がないけど、これ対称式の理論?
・「因子はp(a+b+c)-aの形しか許されない」の証明もないけど。そもそもこの命題の「・・しか許されない」って言える?
・ なので、高校数学の範囲外では?

>>166
・「与式に b=-1, c=2 を代入すると a^2 - (2k+1)a -2 であるが・・したがって (a+1)(a-2) または (a-1)(a+2) でなければならない」
 は、厳密ではない気がする
 2次方程式の解の公式を使うと
 a^2 - (2k+1)a -2=(a-(2k+1+√{(2k+1)^2+8})/2)(a-(2k+1-√{(2k+1)^2+8})/2) と因数分解できる
 k=0, -1 のとき、どちらも {(2k+1)^2+8}=9で √9=3を得て、(2k+1+√{(2k+1)^2+8})/2などは整数になる
・しかし、k=0, -1に限ることが言えていない
 実際{(2k+1)^2+8}を見ると、奇数の二乗と8との和だ。これが、整数の二乗になれば嬉しい(√が取れるから)
 かつ、(2k+1+√{(2k+1)^2+8})/2だから、奇数の二乗になれば、2で割り切れるのでOKだ
 繰り返すが、これが k=0, -1に限ることが言えていないと思う

なので、高校数学を超えている気がする。整数解の非存在を示すディオファントス問題は、見かけ以上に高等数学を必要とする場合が多い
もし、高校数学の範囲外なら、このスレの話題としては、どうなのでしょう(特に、無理に成立していない証明を書くのは良くない気がする)
もし、高校数学の範囲での証明を得ているなら、教えてほしい
0170132人目の素数さん垢版2022/06/03(金) 23:21:16.11ID:FhpV48k4
どこが厳密じゃないのかわからない
0171132人目の素数さん垢版2022/06/04(土) 00:10:38.83ID:eRgJR2a7
>>170
指摘したところの証明がないよね
補題としてでも良いから、証明がいるよ

命題1) 「対称一次因子なし→一次因子3つが移り合う」
命題2)「因子はp(a+b+c)-aの形しか許されない」
命題3)「与式に b=-1, c=2 を代入すると a^2 - (2k+1)a -2 であるが、これも同様の因数分解ができなくてはならず、
 したがって (a+1)(a-2) または (a-1)(a+2) でなければならない」
 (補足:k=0又は-1に限ることの証明がない)

この3つの命題の証明がない
0173132人目の素数さん垢版2022/06/04(土) 03:50:42.02ID:KWZVVNy+
高校の範囲で、行列から、極形式、複素平面を教えるように変わってから勘違いする馬鹿多発してるよね

複素平面の話全部わかってるつもりで解答述べる数学科の馬鹿多すぎ

行列の時代だって2x2の行列までしかやってないのに、リーマン面とか話出す奴もいるし
0174132人目の素数さん垢版2022/06/04(土) 04:09:29.82ID:KWZVVNy+
あと、嬉々として問題出す側が、高校の範囲を逸脱してる問題をでしてるってのもある
0177132人目の素数さん垢版2022/06/04(土) 08:38:57.08ID:eRgJR2a7
>>175
ありがとう
やってみて
0178132人目の素数さん垢版2022/06/04(土) 08:54:03.63ID:+ZoDzV30
対称一次因子がないなら一次因子G=pa+qb+rc、p≠qがあるとしてよい、p≠0として良い
F = GH
と分解されてる
特に因数定理よりa → (-(qb+rc)/p)によりFは0になる
置換してから代入しても0になるので因数定理の逆でFは
pa+qc+rb、pb+qa+rc、pb+qc+ra、pc+qa+rb、pc+qb+ra
で割り切れる
しかし元々三次式なので2つずつ一致するしかない
よってp,q,rのうちどれか一組は等しく一次因子はu(a+b+c)-aの形とその置換としてよい
0179132人目の素数さん垢版2022/06/04(土) 08:55:19.45ID:+ZoDzV30
そもそもこんなもん受験で出題されないから受験参考書には書いてないが理系の高校生なら本来できなあかん奴
セタは高校レベルの数学力ありません
0180132人目の素数さん垢版2022/06/04(土) 09:25:41.02ID:mmbn9+Uk
>>171
このスレでは水を得た魚の如く意気軒昂ですね
高校数学がちょうどいいレベルなんでしょう

さて高校数学の問題です
1000m進むとn m登る勾配をn パーミルといいます
では20パーミルは角度で言うと
1°より大か小か?
2°より大か小か?
0184132人目の素数さん垢版2022/06/04(土) 11:00:19.03ID:Dq5EU9Eg
>>180
>20パーミルは角度で言うと

勾配2パーセントの方が通りがいいと思うが、なんでパーミル?

sinθ<θより、
sin1°=sin(π/180) <π/180 <3.15/175=9/50=0.018
また、
cos1°> cos15°=cos(45°-30°)=(√6+√2)/4≧(2.4+1.4)/4 >0.9
よって
tan1°<0.018/0.9=0.02

また、
tanθ>θより、
tan2°=tan(π/90) >π/90>0.02

以上より、tan1°<0.02<tan2°
0185132人目の素数さん垢版2022/06/04(土) 11:45:46.29ID:eRgJR2a7
>>178
なるほど、それはうまい証明だね
レベル高いね
因数定理は、下記ね

>対称一次因子がないなら一次因子G=pa+qb+rc、p≠qがあるとしてよい、p≠0として良い

それは、>>171 で指摘した 命題1) 「対称一次因子なし→一次因子3つが移り合う」の部分だね
やっぱ、証明いるじゃんw

>pa+qc+rb、pb+qa+rc、pb+qc+ra、pc+qa+rb、pc+qb+ra

G=pa+qb+rcの式で、a,b,cの置換をするんだね。なお、G=pa+qb+rcは、ガロア第一論文のガロア分解式に似ている

>しかし元々三次式なので2つずつ一致するしかない

三次式なのでというより、a,b,c各文字につき、二次式だからだね

>よってp,q,rのうちどれか一組は等しく一次因子はu(a+b+c)-aの形とその置換としてよい

これは、>>171 で指摘した 命題2)「因子はp(a+b+c)-aの形しか許されない」の部分だね
やっぱ、証明いるじゃんw

なお、”どれか一組は等しく一次因子はu(a+b+c)-aの形とその置換としてよい”は、
高校数学の範囲内か? 範囲外とも言えないだろうけど

また
高校生の答案としては、もう少し丁寧に誘導しないと、”ごまかしている”とされるかも
(というか、入試記述問題の採点は、採点基準が作ってあって、採点基準にある記述がない場合に、どんな採点されるか不明だよ
 へんに省略すると、分かってないのをごまかしだと、減点の可能性あるだろう)

(参考)
https://manabitimes.jp/math/1050
高校数学の美しい物語
因数定理とその重解バージョンの証明 更新日時 2021/03/07
因数定理
多項式 f(x) が (x-a) を因数に持つ ?f(a)=0
因数定理は多項式の因数分解に使える強力な定理です。因数定理とその拡張を証明します。

https://manabitimes.jp/math/831
高校数学の美しい物語
対称式について覚えておくべき7つの公式 更新日時 2021/10/28
目次
2変数の対称式に関する基本公式と例題
n乗の和を基本対称式で表す
引き算も対称式で表せる場合がある
三変数の対称式を基本対称式で表す
対称式の基本定理
(引用終り)
以上
0186132人目の素数さん垢版2022/06/04(土) 13:29:14.96ID:Veunfmdm
△ABCにおいて、AからBCに下ろした垂線の足をH、BからCAに下ろした垂線の足をI、CからABに下ろした垂線の足をJとする。
AB,BC,CAが変化するとき、比(AH+BI+CJ)/(AB+BC+CA)の取りうる値の範囲を求めよ。
0188イナ ◆/7jUdUKiSM 垢版2022/06/04(土) 17:09:18.58ID:iEcNyb1q
正対させた正五角形に頂点あわせて菱形状に置ければ、
(7+2√5)/8
左右が支えたらちっさなる。
0190132人目の素数さん垢版2022/06/04(土) 19:45:36.17ID:eLRqDsMX
内接円との接点で3辺を切ってa=v+w, b=w+u, c=u+vとする
u,v,wはℝ⁺×ℝ⁺×ℝ⁺全体を動く
外接円の半径をR、面積をSとする

2R( AH+BI+CJ )
= 4R²( sinBsinC + sinCsinA + sinAsinB )
= (u+v)(u+w) + (v+w)(w+u) + (w+u)(w+u)

BC + CA + AB
= 2u+2v+2w

R
= abc/(4S)
= (v+w)(w+u)(u+v)/(4√(uvw(u+v+w)))

∴ ( AH+BI+CJ )/( BC + CA + AB )
=
((u+v)(u+w)+(v+w)(w+u)+(w+u)(w+v))√(uvw)
/((v+w)(w+u)(u+v)√(u+v+w))
= (√u √(vw)/(v+w) + √v √(wu)(w+u) + √w √(uv)/(u+v) )/√(u+v+w)
≦(√u + √v + √w )/(2√( u+v+w ))
≦√3/2

((u+v)(u+w)+(v+w)(w+u)+(w+u)(w+v))√(uvw)
/((v+w)(w+u)(u+v)√(u+v+w))
の下限は明らかに0
0191イナ ◆/7jUdUKiSM 垢版2022/06/05(日) 01:17:02.23ID:PSDKcbIz
>>188
>>132
正五角形を正対させ、
内部に正方形を菱形状におき、
正方形の上側の頂点が、
正五角形の上側の頂点P(0,p)に、
わずかに届かない状態を作図し、
図のpをどんどん小さくしていくと、
正方形の面積は2p^2になり最大。
正五角形の対角線の長さは黄金比(1+√5)/2
PRを斜辺とする直角三角形について、
ピタゴラスの定理より(2p)^2+(1/2)^2={(1+√5)/2}^2
4p^2+1/4=(6+2√5)/4
16p^2+1=6+2√5
4p=√(5+2√5)
p=√(5+2√5)/4
正方形の一辺の長さは√(5+2√5)/2√2が最大。
2p^2=(5+2√5)/8
∴(5+2√5)/8
0193132人目の素数さん垢版2022/06/05(日) 07:44:16.26ID:n5vX6CbC
>>185
>なお、”どれか一組は等しく一次因子はu(a+b+c)-aの形とその置換としてよい”は、
>高校数学の範囲内か? 範囲外とも言えないだろうけど

まとめておくよ
1)”どれか一組は等しく一次因子はu(a+b+c)-aの形とその置換としてよい”は、下記の置換論を知っていれば常識だが、高校数学の知識外(大学数学)
2)下記wikipediaの引用程度なので、集合 {1, 2, 3} の置換自体は、高校レベル(中学レベルかも)
3)しかし、多分入試には出ないだろうし、出ても誘導がついて、部分点をゲットすれば十分と思う
4)元の問題>>161 "(a+b+c)(ab+bc+ca)+kabc が整数係数の1次以上の多項式で因数分解できるとき、整数kの値をすべて求めよ"を、高校数学の知識だけで完答するのは、相当難しい
5)試験の時間戦略としては、k=0,-1 まではきっちり示して、あとは各人の時間配分と力量しだい(完答には力を入れずに、見直しに時間を使うのもあり)
6)なお、昔”不変式論”とかあって(いま死語)、向井 茂先生の不変式の関連記事を下記に紹介する(ハイレベル高校生向けな)

では

(参考)
https://ja.wikipedia.org/wiki/%E7%BD%AE%E6%8F%9B_(%E6%95%B0%E5%AD%A6)
置換 (数学)
置換(ちかん、英: permutation)の概念は、いくつか僅かに異なった意味で用いられるが、いずれも対象や値を「並べ替える」ことに関するものである。有り体に言えば、対象からなる集合の置換というのは、それらの対象に適当な順番を与えて並べることを言う。例えば、集合 {1, 2, 3} の置換は、
(1,2,3), (1,3,2), (2,1,3), (2,3,1), (3,1,2), (3,2,1)
の全部で六種類ある順序組である。

https://www.kurims.kyoto-u.ac.jp/~mukai/paper/SuSemi06.pdf
不変式の話
数学セミナー連載,2005 年 12 月号,2006 年 1,2,4 月号
向井 茂
P6
§2 対称式
0194132人目の素数さん垢版2022/06/05(日) 09:06:39.58ID:cC/PFlmJ
a^2 - b^2 =9

ab = 3

のとき、a+bの値を求めよ。

a,b共に実数。


ツイッターで流れてきたのですが解けません・・・。
0195132人目の素数さん垢版2022/06/05(日) 09:21:52.25ID:7zPiOD96
>>193
高校数学では元気やな アンタ
ところで、n×n正方行列のランクがnのとき
そのときに限り行列式が0でないのは理解した?
階段化の操作で行列式が不変なのも分からん奴が
不変式語っても笑われるだけやで ホンマに
0196132人目の素数さん垢版2022/06/05(日) 10:07:41.67ID:9jGmxaiV
ガロアスレであれだけアホ晒してたからな
いい加減群論なんか自分の知能では理解できそうもないとわかりそうなもんだけどな
それすらわからんほどアホなんやろ
0197132人目の素数さん垢版2022/06/05(日) 13:48:35.26ID:BU8OeBNL
>>194
x=a+b, y=a-bとおけば
xy=9
x^2 -y^2=12
y=9/xとおいて、あとは流れで。
0198132人目の素数さん垢版2022/06/05(日) 14:17:52.10ID:kOiAfxWB
>>146 の pが素数のとき、C[p-1,k] (0≦k≦p-1) はmod p で1か-1
の証明は次でよいでしょうか:

C[p-1,k]=(p-1)(p-2)…(p-k)/k! = cとおく。
(p-1)(p-2)…(p-k) = c*k!
よってmod pで (-1)(-2)…(-k)≡c*k! ∴(-1)^k*(k!) ≡ c*k!
k! は mod pで0ではないので、(-1)^k ≡ c 。(終わり)
0199132人目の素数さん垢版2022/06/05(日) 15:53:07.15ID:47FVZaM+
直行座標系の(+,+)となる座表面において
ある直線に常に90度で交わる任意の直線を引くとします。
その任意の直線を構成する座標を求めたいです。
数学的にどのような道具を使う必要がありますか?
0200132人目の素数さん垢版2022/06/05(日) 16:00:47.25ID:0/u4ldUx
>>198
微妙
受験で
ac≡bc ( mid p)、p|̸c → a≡b ( mod p )
は認めてもらえないかもしれない、少なくとも教科書には載ってないし、コレを証明させる問題が出題されたこともある
0201132人目の素数さん垢版2022/06/05(日) 17:23:37.45ID:ScHgF4R0
回答する人より質問する人の方が利口なことって良くあるよね
0204132人目の素数さん垢版2022/06/05(日) 20:46:52.34ID:BU8OeBNL
出題バカが常駐しちゃってるからな。
相手するほうも馬鹿なんだから、二人だけで別スレ立ててくれりゃいいんだが、意固地になってここに居続けてる。
0206132人目の素数さん垢版2022/06/05(日) 23:16:58.79ID:/X3faoR9
>>205
できました

x>0, Γ(x)=∫[0, ∞)e^(-t){t^(x-1)}dt
f(x)は積分区間A=[0, ∞)の上で連続関数であるから任意のコンパクト集合(⊂A)の上で可積分である。

任意のn∈Nに対してt>0ならば
e^t≧Σ[k=0, n]t^k/k!>t^n/n!なので
e^(-t)t^(x-1)≦n!t^(x-n-1)
t→∞の時, e^(-t)t^(x-1)=Ο(t^(x-n-1))
x>0を任意に1つ固定すると、n>xとなるような正整数nが取れてx-n-1<-1だから∫[1, ∞)f(t)は収束する。

・x≧1で、f(t)はコンパクト集合[0, 1]上連続であり可積分である。
・0<x<1で、f(t)→∞ (t→+0)であり広義積分となるが、f(t)=Ο(t^(x-1)) (t→+0)なので、∫[0, 1]f(t)は収束する。
以上により絶対収束することが示された。
0208イナ ◆/7jUdUKiSM 垢版2022/06/06(月) 20:33:49.69ID:DGGbSDo4
>>191
>>194
a=b/3
(3/b)^2-b^2=9
9/b^2=b^2+9
b^4+9b^2-9=0
b^2={-9+√(81+36)}/2
b=±√(√117-9)/√2
=±√(2√117-18)/2
a=±(√18+2√117)/2
∴a+b=±{√(18+2√117)+√(2√117-18)}/2
0213132人目の素数さん垢版2022/06/07(火) 09:19:27.40ID:I0E0j060
答だけ与えても意味ないでしょうに。
>>194には>>197で示した置き換えで、x=(a+b)に対して、
x^2-81/x^2 =12
X=x^2で置き換えれば、
X^2-12X-81 =0
という2次方程式になるので、解の公式から
X =6±3√13
が得られるが、xは実数なので、6-3√13 <0 は不適。
よって、
a+b = x =±√X =±√(6+3√13)
0214132人目の素数さん垢版2022/06/07(火) 14:31:13.00ID:BCneTluU
x+y=a,xy=b,x-y=cとする。

(1)cをaとbで表せ。
(2)a,b,cが実数かつ、c<b<aとなるとき、x,yが満たすべき条件を求めよ。
0215132人目の素数さん垢版2022/06/07(火) 20:06:13.30ID:BCneTluU
aを実数の定数とする。
またx+y=s,xy=tとする。

(1)x+ayをsとtで表せ。
(2)-1≦s≦1,-1≦x+ay≦1のとき、tの取りうる値の範囲をaで表せ。
0216132人目の素数さん垢版2022/06/07(火) 20:40:06.10ID:BCneTluU
x+y=a,x-y=b,xy=cとする。

(1)a,b,cがすべて相異なるとき、x,yが満たすべき条件を求めよ。
(2)a,b,cが3次方程式t^3+pt^2+qt+r=0の相異なる3つの実数解となり、かつa,b,cがすべて実数であるとき、p,q,rをx,yで表せ。
(3)p,q,rは(2)の条件を満たすとする。3次関数f(t)=t^3+pt^2+qt+rの増減を調べよ。
0217132人目の素数さん垢版2022/06/07(火) 23:15:45.58ID:Lw9An7n/
x軸とy軸のように
実数部と虚数部のように
相互関係の無い関係をなんて言うのでしょうか?

互いに素
は違う気がする
0219132人目の素数さん垢版2022/06/08(水) 01:36:09.57ID:3HqQcgqI
元でしょ

3次元の空間座標だと
直交って言葉に意味はあると思うけど
0220イナ ◆/7jUdUKiSM 垢版2022/06/08(水) 07:41:54.97ID:kvNADvy9
>>208,209補足。
>>194
a+b=±{√(18+2√117)+√(2√117-18)}/2
=±{√(9+9√13)+√(9√13-9)}
=±4.10081136196……
0223132人目の素数さん垢版2022/06/08(水) 12:33:23.93ID:3HqQcgqI
>>222
これ、五角形の頂点や辺に、正方形の頂点全て内接してるの?
たまたま?
0224132人目の素数さん垢版2022/06/08(水) 14:40:49.65ID:97XlER1e
>>223
正方形の1個の頂点が五角形の頂点、
正方形の1個の頂点が五角形の辺の上
になっているようにみえる。
0226132人目の素数さん垢版2022/06/08(水) 15:00:36.74ID:K09UVSu3
2x+1=1の時、(x+y)^2+4^2=x^2+y^2-2x-2y+2

これはどうでしょう
0227132人目の素数さん垢版2022/06/08(水) 15:36:27.46ID:9tbTCDsz
2x+1=1の時、(x+y)^2+4x^2=x^2+y^2-2x-2y+2 です
すみません
0228132人目の素数さん垢版2022/06/08(水) 15:38:12.75ID:zMkmqLTe
あ、2x+y=1の時です
何度も申し訳ない
0231132人目の素数さん垢版2022/06/08(水) 20:57:33.05ID:+VS9u+Pa
tを実数の定数、iを虚数単位とする。
x+y=a,xy=b,x+ity=c
と定める。

(1)cをa,b,tを用いて表せ。
(2)x,yがともに実数であるために、a,bが満たすべき条件を求めよ。
(3)x,yがともに実数であるとする。a<|c|<bとなるようなx,yの範囲を求めよ。
0233132人目の素数さん垢版2022/06/08(水) 22:11:42.68ID:+VS9u+Pa
[z]でzを超えない最大の整数を表す。
実数x,yに対して、
x+y=a,xy=b,[x]+[y]=cとする。

(1)a-cの取りうる値の範囲を求めよ。
(2)-1≦a≦1かつ-1≦b≦1のとき、cの取りうる値の範囲を求めよ。
0234132人目の素数さん垢版2022/06/08(水) 22:32:38.80ID:+VS9u+Pa
aを実数とする。
実数x,yが
x+y=a^2+1,xy=a
を満たしているとき、x-yが実数でないようなaの範囲を求めよ。
0236132人目の素数さん垢版2022/06/09(木) 00:45:29.15ID:pZmlKEyz
質問おじさんはその辺を叫びながら歩いてる人みたいなもん
関わったら負け
0237132人目の素数さん垢版2022/06/09(木) 00:49:16.91ID:8AsCFq/b
>>236
だよなぁ。

質問おじさん(=ID:+VS9u+Pa )にいちいち回答してるやつのアホ面を一度見てみたいわ。
度の強いメガネかけてそうw
0238132人目の素数さん垢版2022/06/09(木) 05:11:53.62ID:T2CkkA8M
aを実数とする。
複素数x,yが
x+y=a^2+1,xy=a
を満たしているとき、x-yが実数でないようなaの範囲を求めよ。
0239132人目の素数さん垢版2022/06/09(木) 05:17:35.00ID:T2CkkA8M
x+y=a,xy=b,x-y^2=cとする。

(1)cをa,bで表せ。
(2)a,b,cがある1つの三角形の3辺の長さとなるとき、x,yが満たすべき条件を求めよ。
0240132人目の素数さん垢版2022/06/09(木) 05:19:31.82ID:T2CkkA8M
ある実数tが存在して、
x+y=sin(t),xy=cos(t)
となるとき、実数x,yが満たす条件を求めよ。
0244132人目の素数さん垢版2022/06/09(木) 10:29:55.48ID:8AsCFq/b
>>238-240
馬鹿にされて、ムキになってろ問題を連投してるw

こいつに反応してるのも、イナとかプログラムおやじみたいな
ろくでもないヤツばかりwww

たまにくる質問も、こいつらのアホな投稿のノイズに埋もれてしまう。
0246132人目の素数さん垢版2022/06/09(木) 13:35:31.86ID:hllhmSL+
a,b,cは複素数x,yを用いてx+y=a,xy=b,x-y=cと表せるという。

(1)cをaとbで表せ。
(2)a,b,cが実数で、x,yの少なくとも一方が実数でないとき、積abcの取りうる値の範囲を求めよ。
0252132人目の素数さん垢版2022/06/09(木) 21:01:58.68ID:hllhmSL+
a,b,cは複素数x,yおよび虚数単位iを用いてx+y=a,xy=b,x-iy=cと表せるという。

(1)cをaとbで表せ。
(2)a,b,cが実数で、x,yの少なくとも一方が実数でないとき、積abcの取りうる値の範囲を求めよ。
0253イナ ◆/7jUdUKiSM 垢版2022/06/09(木) 21:12:52.43ID:bXLmZfxx
>>132
>>232
正五角形を長さ1の辺を底辺にして正対させ、
正方形をいちばん上の頂点をあわせて菱形状に、
辺の長さxが最大になるようにとると、
1/sin63°=x/sin72°
最大の面積x^2=(sin72°/sin63°)^2
=1.13933354134
やっと解けた。
0255132人目の素数さん垢版2022/06/10(金) 01:15:13.24ID:w/cC8K/y
AB=1,BC=t,∠ABC=90°の直角三角形ABCの辺BC上を動く点Pがある。
PからABに平行な直線を引き、そのACとの交点をQとする。
BP=x(0<x<t),AP+PQ+QB=L(x)とするとき、L(x)の増減を調べよ。凹凸は調べなくて良い。
0259イナ ◆/7jUdUKiSM 垢版2022/06/10(金) 12:35:47.53ID:lFUX59ZB
>>253
>>132
正五角形の頂点を(1/2,√(5+2√5)/2),((1+√5)/4,√(10-2√5)/4),(0,0),(-(1+√5)/4,√(10-2√5)/4),(-1/2,√(5+2√5)/2)にとり、
正方形の頂点を(t/√2,t/√2),(0,0),(-t/√2,t/√2),(0,t√2)にとると、
直線y=xとy=-√(5+2√5)x+√(5+2√5)の交点の座標は、
(√(5+2√5)/{1+√(5+2√5)},√(5+2√5)/{1+√(5+2√5)})
正方形の一辺の長さの最大値は、
√2(5+2√5)/{1+√(5+2√5)}
正方形の面積の最大値は、
2(5+2√5)/{1+√(5+2√5)}^2
=(5+2√5)/{3+√5+√(5+2√5)}
=(5+2√5){3+√5-√(5+2√5)}/(9+4√5)
={25+11√5-(5+2√5)√(5+2√5)}(9-4√5)
=5-√5-(5-2√5)√(5+2√5)
=1.13933354134……
0263イナ ◆/7jUdUKiSM 垢版2022/06/10(金) 15:24:30.15ID:lFUX59ZB
>>259
>>254
正五角形の中の最大の正方形の一辺をtとすると、
1/sin63°=t/sin72°
sin63°=sin72°/t
=2sin36°cos36°/t
=2(√(10-2√5)/4)(1+√5)/4√{5-√5-(5-2√5)√(5+2√5)}
={(1+√5)√(10-2√5)}/8√{5-√5-(5-2√5)√(5+2√5)}
=0.891……
0264132人目の素数さん垢版2022/06/10(金) 17:44:22.09ID:ynmnzgJq
-1<x^2+ax+b<1
となる実数xが存在するために、a,bが満たすべき必要十分条件を求めよ。
0267132人目の素数さん垢版2022/06/10(金) 19:00:52.30ID:zN5Rq5hS
>>264
できました

I=∫[0, ∞]sinx/x dx、0<v<u<∞とする。
部分積分法により
|∫[v, u]f|=|-cosx/x|-∫[v, u]sinx/x²
<1/u+1/v+∫[v, u]/x²=2/v→0
広義積分はコーシーの収束条件をみたすので収束する。
∫[nπ, (n+1)π]|sinx|/x
=∫[0, π]sinx/(x+nπ)>2/(n+1)πより
∞に発散するから絶対収束しない。
リーマン・ルベーグの定理により、[0, π]上で連続のfに対して
∫fsin(2n+1)x/2→0、I=π/2
t<0の時, -tx=yとおく。
ロピタルの定理により
1/t-1/sint=(sint-t)/tsint
=(cost-1)/(sint+tcost)
=-sint/(cost+cost-tsint)
→-0/(1+1-0)=0
0268132人目の素数さん垢版2022/06/10(金) 23:21:07.63ID:xYOrhzfA
x+y=a,xy=b,x-y=cとする。

(1)cをaとbで表せ。
(2)a[n]=x^n-y^nとする。a[n+1]をa[n],a,bのうち必要なもので表せ。
0271132人目の素数さん垢版2022/06/11(土) 04:57:11.82ID:wQZPGT08
△ABCの内部に定点Pをとり、Pを通る直線を考える。
Pは△ABCの2辺により切り取られるが、その切り取られる長さが最小となるような直線を決定せよ。
0275132人目の素数さん垢版2022/06/11(土) 08:57:06.06ID:MjSG8tq+
>>234
じゃあn=1の時に
x-y
をx+yとxyで表してみてよ
もちろん
x-y = ±√((x+y)²-4xy)
はできてないよ
左辺は確定した値ひとつだけ右辺は違う
どちらなのか判断する材料はa,bだけ
0277132人目の素数さん垢版2022/06/11(土) 11:32:49.36ID:eToWYhEE
a[1]=b[1]=c[1]=1
a[n+1]=b[n]+c[n]
b[n+1]=c[n]+a[n]
c[n+1]=a[n+1]+b[n]
で表される数列を考える。このうち数列{c[n]}の一般項を求めよ。
0278132人目の素数さん垢版2022/06/11(土) 11:39:13.01ID:eToWYhEE
平面上に△ABCがある。
この△ABCの面積を2等分するように、同一平面上に直線を引く。
このような直線の中で、△ABCの周および内部の領域に含まれる部分の長さが最長になるものを考えるとき、それは頂点A,B,Cのうちいずれかを通ることを示せ。
0279132人目の素数さん垢版2022/06/11(土) 12:23:02.83ID:omZhMsFT
できました

I=∫[0, π/2]logsinθdθ
√θsinθ→0
ある正数ε (0<θ<ε)に対して
|logsinθ|≦c/√θ
よってIは絶対収束する。
π-θ, π/2-θと変数変換し、
θ=2ΦとおくとI=(-π/2)log2

リーマンのゼータ関数
s≦0の時, 、1/n^s≧1より発散する
f(x)=1/x^sはI=[1, ∞)で単調減少であるからマクローリンの判定法が適用出来る。s>1の時, 収束し、s≦1の時, 発散する。
0282132人目の素数さん垢版2022/06/11(土) 12:31:24.04ID:eToWYhEE
△ABCは以下の条件を満たす。
(i)∠B=90°
(ii)AB,BC,CAはいずれも整数
(iii)AB,BC,CAのうち2つは素数
このような△ABCは無数に存在することを示せ。
0284132人目の素数さん垢版2022/06/11(土) 12:39:10.95ID:bBWjjxAb
>>283
だからそもそもこんなの高校数学の縛りプレイで証明させると言う発想そのものがくだらないんだよ
0287132人目の素数さん垢版2022/06/11(土) 16:20:52.05ID:viqqY7Dh
ってか、そもそも「出題」と「質問」は違うからね。

解き方がわかってる問題の解き方を「質問」するのはおかしいだろ。

スレチガイ
0288132人目の素数さん垢版2022/06/11(土) 18:11:30.19ID:9M/J0sDa
s=(e^t+e^(-t))/2とする。

(1)∫[0,1] √(1+x^2) dxをx=sと置換して求めよ。

(2)双曲線xy=1の1≦x≦2の部分の長さを求めよ。
0289132人目の素数さん垢版2022/06/11(土) 23:06:15.77ID:dNCkxHtu
>>272
グラフ描いて
逆関数作って
積分すればいいんでないの

てきとうに考えたけど
0294132人目の素数さん垢版2022/06/12(日) 00:40:56.71ID:92AmhRXt
賢い質問者はここまでは分かっていて、ここからここへ行く計算内容や発想が分からない、というような質問の仕方をする
問題がまるごと分からないのなら、その問題を解くレベルに達していないので同じ単元の簡単な問題から解くといい
0295132人目の素数さん垢版2022/06/12(日) 00:52:10.04ID:3JkkVP6w
>>292
何処が虚しいか言ってみ
0297132人目の素数さん垢版2022/06/12(日) 05:37:25.96ID:CxfgtIfs
aを正の実数とする。
xy平面上の直線l:y=xのa≦x≦a+1の部分をCとする。
x軸上に点Aをとり、Cの両端とAを結ぶ2本の線分がなす角∠Aを最大化する。
このときのAの座標をaで表せ。
0298イナ ◆/7jUdUKiSM 垢版2022/06/12(日) 07:57:08.51ID:eeWq68YY
>>293
>>297
点(a+1/2,a+1/2)を通り傾き-1の直線が、
x軸と交わる点(2a+1,0)が、
求めるそれと考えられる。
∴(2a+1,0)
0304132人目の素数さん垢版2022/06/12(日) 09:51:38.71ID:mnZ/SpO/
>>303
ホントに出てるならそんなレスつけるわけないやん?
”××年の後期に出てます”で終わりやろ
なんでそんな秒でウソとわかるウソが吐けるん?
嘘つく事に抵抗なくしてるん?
もう人間として終わってるよ
0306132人目の素数さん垢版2022/06/12(日) 13:58:59.97ID:gbjnW0LA
>>302
イナ氏のレスはトンチンカンな解答ばかりだから読むだけ無駄
でも、コテハンにしてNG可能にしてるのは良心的とも言える
0307132人目の素数さん垢版2022/06/12(日) 14:11:47.94ID:3JkkVP6w
>>305
そう言う意味ね
了解
0312132人目の素数さん垢版2022/06/12(日) 16:31:34.69ID:uWkRcJ1E
AB=√5、BC=√7、CA=4である三角形ABCがある。
ベクトルABとベクトルBCの内積はいくらか。

これ
教えてください
0314132人目の素数さん垢版2022/06/12(日) 16:49:57.69ID:WDOlgNV3
>>298
Cを弦としx軸に接する円を考えろ
0316イナ ◆/7jUdUKiSM 垢版2022/06/12(日) 17:00:32.01ID:eeWq68YY
>>298
>>312 →AB・→BC=AB×BC×cos∠BAC=(√5)(√7)cos∠BAC
cos∠BAC=(5+7-4)/(2×√5×√7)=4/√35
∴ →AB・→BC=4
0318132人目の素数さん垢版2022/06/12(日) 17:30:42.69ID:WDOlgNV3
>>316
cos∠BACじゃなくてcos∠ABC
余弦の分子は5+7-4じゃなくて5+7-4^2
0320132人目の素数さん垢版2022/06/12(日) 18:54:07.89ID:gbjnW0LA
>>317
ああすまん、ベクトルABとベクトルBCのなす角は∠Bの外角だったね。
ボケとるわ。
内積=AB・BC・cos(πー∠B)= - AB・BC・cos∠B = 2
0322イナ ◆/7jUdUKiSM 垢版2022/06/12(日) 19:59:42.12ID:zqNp9xpr
>>316
>>314
Cを弦としてx軸と接する円は2つあり、
{x-2a-1+√(2a^2+2a)}^2+{y-√(2a^2+2a)}^2={2a+1+√(2a^2+2a)}^2
{x-√(2a^2+2a)}^2+{y-2a-1+√(2a^2+2a)}^2={2a+1-√(2a^2+2a)}^2
0324132人目の素数さん垢版2022/06/12(日) 20:31:20.23ID:gbjnW0LA
要するに、ベクトルの内積に分配法則が使えるって前提なら
(AB⃗ + BC⃗ )・(AB⃗ + BC⃗) = AC⃗・AC⃗
AB^2 +2AB⃗・BC⃗ + BC^2 =AC^2
で簡単。

分配法則に抵抗があるなら余弦定理からいけばいい。
0325132人目の素数さん垢版2022/06/12(日) 21:01:16.33ID:C3PDwMKw
教科書ではないのですがIT関係の読み物を読んで、
公開鍵暗号方式の、2つの素数による暗号化・復号化の原理を、計算式のレベルでは理解しました。
それでもひとつ疑問な部分があります。

現在広く使われている2048bitの合成数をを作るための素数はそれぞれ、十進数にして300桁台の巨大数になるはず。
しかし、その300桁の素数というものをどこから持ってくるのですか?
毎回ランダムで数字を発生させるとして、その数字が素数であるとどうやって分かるのですか?
この桁数だと現実的な計算回数では素数判定できないような・・・と思うのですが。
0327132人目の素数さん垢版2022/06/12(日) 21:14:18.03ID:ZvKHhyKN
ここには面倒なルールは一切ありません。
自由に投稿しましょう。
0328 【大凶】 垢版2022/06/13(月) 00:45:25.11ID:o98Rql5Y
>>322訂正。
>>312
→AB・→BC=AB×BC×(-cos∠ABC)
=(√5)(√7)(-1){(√5)^2+(√7)^2-4^2}/{2(√5)(√7)}
=-(5+7-16)/2
=4/2
=2
0329132人目の素数さん垢版2022/06/13(月) 00:56:56.65ID:lui0pYLD
内積ってなんなんですか?
a・b = |a| |b| cosθ
aの大きさと、bのa方向成分の大きさ、の積だなぁって見てるけど
長さx長さ=面積 だけど、同じ向きだから面積ではないし
いったいなんなんだ?と悩んでます
0330132人目の素数さん垢版2022/06/13(月) 01:08:00.93ID:wyAIsLcU
>>325
ミラーラビン法により確率的な判定法で実用上は素数(かもしれない数)は得られるてるんじゃなかったっけ。
ある数が素数かどうかはガウス和を巧妙に使ったRSAの3人の中の誰かかが発明したアルゴリズムで確定的に判定出来た筈。
0332132人目の素数さん垢版2022/06/13(月) 01:32:38.27ID:8iOy26u0
e^π、π^e、3^π、π^3の大小を比較せよ。
ここでe=2.718...は自然対数の底、π=3.141...は円周率である
0335132人目の素数さん垢版2022/06/13(月) 14:39:32.45ID:WoTOsuoo
♾に持ってく問題出すのやめたら
0336132人目の素数さん垢版2022/06/13(月) 15:58:12.77ID:65ce49oY
出題爺さんは気ままに出題してるだけだから、なにを忠告しても無駄だよ。

無視するのが一番。
0337132人目の素数さん垢版2022/06/13(月) 18:36:27.69ID:Z7rrah1p
>>329
ベクトルどうしの「掛け算」を定義したい
できれば分配法則 a・(b+c)=a・c+a・c が成り立ってほしい
これをかなえるためにはa・b=|a||b|ではダメ・・・
じゃあ定義するべきか・・・?って話だよ
0338132人目の素数さん垢版2022/06/13(月) 18:48:24.78ID:8iOy26u0
2つのベクトルaとbの間の演算□を以下のように定義する。
a□b=(a・b)(a+b)

(1)a□aをaおよび|a|で表せ。

(2)c□dが単位ベクトル(1,0)となるとき、2つのベクトルcとdが満たすべき条件を求めよ。
0339132人目の素数さん垢版2022/06/13(月) 21:50:51.72ID:8iOy26u0
a[1]=a,b[1]=b、aとbは正整数
a[n+1]=2a[n]+3b[n]+p[n]
b[n+1]=3a[n]+2b[n]
p[n]はa[n]とb[n]の最大公約数

このとき任意のnに対してa[n]とb[n]が互いに素となるような正整数の組(a,b)は存在しないことを示せ。
0340132人目の素数さん垢版2022/06/13(月) 22:15:36.19ID:V9uLqthF
2つの円、x^2+y^2=8 と x^2+y^2-x-2y-5=0 は異なる2つの交点を持つ。
これら2つの交点を通り、x軸に接する円の方程式を求めよ。

この問題ですが、
x^2+y^2-x-2y-5+k(x^2+y^2-8)=0とおいて条件からkを求めるタイプだと思うのですが、
kの求め方がわかりま円。
どのようにkを定めればいいですか。
0342132人目の素数さん垢版2022/06/13(月) 23:52:37.95ID:8iOy26u0
>>340
x軸(y=0)に接するんだから、円の式とy=0とを連立した方程式が重解を持てば良い
するとkの値が定まる
0343132人目の素数さん垢版2022/06/14(火) 16:10:10.49ID:TXFumJ+y
なんでベクトルって和語漢語に訳さなかったんですか
0344イナ ◆/7jUdUKiSM 垢版2022/06/14(火) 17:08:31.43ID:QkPAmB3v
>>328
>>340
題意の2円はx^2+y^2=(2√2)^2と(x-1/2)^2+(y-1)^2=(5/2)^2
原点(0,0)を中心とした半径2√2の円と
点(1/2,1)を中心とした半径5/2の円
求める円は2円の共通弦x+2y-3=0に垂直な
直線y=2x上に中心があり、
y座標が半径rだから(x-r/2)^2+(y-r)^2=r^2
点(r/2,r)と直線x+2y-3=0の距離は、
|r/2+2r-3|/√(1^2+2^2)=|5r/2-3|/√5
原点(0,0)と直線x+2y-3=0の距離は、
|0+0-3|/√5=3/√5
弦の長さの半分はピタゴラスの定理より、
√{8-(3/√5)^2}=√(31/5)
ピタゴラスの定理より半径rは、
r=√{(5r/2-3)^2/5+31/5}
5r^2=25r^2/4-15r+9+31
5r^2-60r+160=0
r^2-12r+32=0
(r-4)(r-8)=0
中心は(2,4)ではないから、
r=8すなわち中心は(4,8)
∴円の方程式は(x-4)^2+(y-8)^2=64
0345132人目の素数さん垢版2022/06/14(火) 22:11:49.90ID:CA3pMv7W
相異なる複素数a,b,cがa/(1-b)=b/(1-c)=c/(1-a)=kを満たすとき, kの値を求めよ.

複素平面で幾何っぽく解けますか?
0352イナ ◆/7jUdUKiSM 垢版2022/06/15(水) 14:10:39.21ID:dCA3eDXv
>>347kを使って解いて円が2つあることに気づいた。
>>340
x^2+y^2-8+k(x^2+y^2-x-2y-5)=0
x^2-kx/(1+k)+(1+k)y^2-2ky/(1+k)=(5k+8)/(1+k)
{x-k/2(1+k)}^2+{y-k/(1+k)}^2=(5k+8)/(1+k)+5k^2/4(1+k)^2
中心のy座標の二乗が半径の二乗と等しいから、
k^2/(1+k)^2=(25k^2+52k+32)/4(1+k)^2
4k^2=25k^2+52k+32
21k^2+52k+32=0
(3k+4)(7k+8)=0
k=-4/3,-8/7
k=-4/3のとき(x-2)^2+(y-4)^2=16
k=-8/7のとき(x-4)^2+(y-8)^2=64
0353132人目の素数さん垢版2022/06/15(水) 14:12:31.92ID:N5e1pvnY
>>340
x^2+y^2-x-2y-5+k(x^2+y^2-8)=0で、y=0として得られるxの2次方程式
(1+k)x^2-x-(5+8k)=0
が重解をもてばよい。判別式を考えて
1-4(1+k)(5+8k)=0 これを解いてk=-3/4 or -7/8

k=-3/4 のとき x^2+y^2-4x-8y+4=0
k=-7/8 のとき  x^2+y^2-8x-16y+16=0
0354イナ ◆/7jUdUKiSM 垢版2022/06/15(水) 14:19:00.26ID:V5BMh5A5
>>352
>>340
kを使う方法も知識として記憶にあったけど、
「点と直線の距離」と「ピタゴラスの定理」から、
二つある「中心の座標」を求めるのが自然だと思う。
0355132人目の素数さん垢版2022/06/15(水) 14:33:19.09ID:MzaEK5A4
>>354
kを使う方法は高校の教科書にも普通に載ってるし連立方程式の理解があれば自然な方法
ピタゴラスの定理のほうが自然?できなかった言い訳だろ
お前の学歴教えろ
0356イナ ◆/7jUdUKiSM 垢版2022/06/15(水) 14:40:12.27ID:WCgojkqP
>>354訂正。
>>340
題意の2円はx^2+y^2=(2√2)^2と(x-1/2)^2+(y-1)^2=(5/2)^2
原点(0,0)を中心とした半径2√2の円と
点(1/2,1)を中心とした半径5/2の円
求める円は2円の共通弦x+2y-3=0に垂直な
直線y=2x上に中心があり、
y座標が半径rだから(x-r/2)^2+(y-r)^2=r^2
点(r/2,r)と直線x+2y-3=0の距離は、
|r/2+2r-3|/√(1^2+2^2)=|5r/2-3|/√5
原点(0,0)と直線x+2y-3=0の距離は、
|0+0-3|/√5=3/√5
弦の長さの半分はピタゴラスの定理より、
√{8-(3/√5)^2}=√(31/5)
ピタゴラスの定理より半径rは、
r=√{(5r/2-3)^2/5+31/5}
5r^2=25r^2/4-15r+9+31
5r^2-60r+160=0
r^2-12r+32=0
(r-4)(r-8)=0
r=4のとき中心は(2,4)
r=8のとき中心は(4,8)
∴円の方程式は、
(x-2)^2+(y-4)^2=16
(x-4)^2+(y-8)^2=64
0358イナ ◆/7jUdUKiSM 垢版2022/06/15(水) 18:28:46.05ID:0PEYiS3x
>>356
>>357
図はイメージには必要だけど、
計測するには歪みすぎてる。
その点、式と計算はうそつかない。
点と直線の距離、ピタゴラスの定理を使えば、
重解を持つ条件とか判別式とかより視覚的に理解できると思う
0360132人目の素数さん垢版2022/06/16(木) 02:07:03.15ID:SWAjn0BI
(ED)⇒(PID) の証明って、 A⊂R を R の任意のイデアルとするとイデアルの性質より、0∈A である。 (i) A={0} のときは明らか。 (ii) A≠{0} のとき、 0 ではない A の元をとることができ、特に元 m∈A を ϕ(m) が最小となるようにとる。ここでPIDの性質より、任意の a∈A に対してa=mq+r;r=0 または ϕ(r)<ϕ(m) なる q,r∈R が一意的に存在し、a∈A,mq∈A より、r=a−mq∈A である。ϕ(m)の最小性よりr=0であるから任意のa∈Aに対して q∈R が存在して a=mq が成り立つからA⊂mR である。また、m∈A より、任意の q∈R に対して mq∈A であるから、mR⊂A である。ゆえにA=mRが言え、(ED)⇒(PID) が示せた。であってる?
0362132人目の素数さん垢版2022/06/16(木) 05:39:41.70ID:aj/izZcd
△ABCの頂点AからBCに下ろした垂線の足をH、CAの中点をMとする。
AHとBMの交点をPとするとき、ベクトル↑APを↑ABと↑ACで表せ。
0363132人目の素数さん垢版2022/06/16(木) 05:44:46.04ID:sTx3yUoj
>>358
>その点、式と計算はうそつかない。

>>298が嘘なんですがそれは
0365イナ ◆/7jUdUKiSM 垢版2022/06/16(木) 07:16:03.98ID:tOLqAOlK
>>364
>>298
A(x,0)(a+1<x<2a)
向かいの辺の長さは√2
式を立てて微分したい。
三角形の頂点の座標は、
(a,a),(a+1,a+1),(x,0)
x=3a/2+1/2のとき∠Aかなり大きい。
0366132人目の素数さん垢版2022/06/16(木) 07:41:24.98ID:sTx3yUoj
Cを弦としてx軸に接する円を考えれば円周角により接点で角は最大
0367132人目の素数さん垢版2022/06/16(木) 09:08:27.94ID:Zqgmjxqk
f(x)=x^2+ax+bと表される2次関数f(x)で、任意の整数nに対してf(n)≧0であり、またf(t)<0であるような実数tが存在するものを考える。
それらのうち、b^2-4acが最大であるものを求めよ。
0368イナ ◆/7jUdUKiSM 垢版2022/06/16(木) 11:55:07.62ID:Z4dQ4hk/
>>365訂正。
>>297
(a,a),(a+1,a+1)を通りx軸と接する円の中心は、
y=-x+2a+1上にあり、
y座標はr=-x+2a+1
x座標はx=2a+1-r
点(2a+1-r,r)と直線x-y=0の距離は、
|2a+1-r-r|/√(1+1)
弦Cの半分は√2/2
ピタゴラスの定理より、
(2a+1-2r)^2/2+(√2/2)^2=r^2
(2a+1-2r)^2+1=2r^2
2r^2-4(2a+1)r+(2a+1)^2+1=0
r^2-2(2a+1)r+2a^2+2a+1=0
r=2a+1±√(4a^2+4a+1-2a^2-2a-1)
=2a+1±√(2a^2+2a)
2a+1-r=干√(2a^2+2a)
-√(2a^2+2a)のとき∠Aは小さい。
∴A(√(2a^2+2a),0)
0369132人目の素数さん垢版2022/06/16(木) 11:58:20.89ID:GwBPrwa2
>>367
cってなんだよ?
0370132人目の素数さん垢版2022/06/16(木) 12:59:30.24ID:aj/izZcd
f(x)=x^2+ax+bと表される2次関数f(x)で、任意の整数nに対してf(n)≧0であり、またf(t)<0であるような実数tが存在するものを考える。
このとき、a^2-4bの取りうる値の範囲を求めよ。
0371イナ ◆/7jUdUKiSM 垢版2022/06/16(木) 14:19:22.64ID:4t2wggdi
>>368
>>370
f(x)=(x+a/2)^2+b-a^2/4
たとえばy=f(x)が(1,0),(2,0)でx軸と交わるとすると、
頂点の座標は(3/2,-1/4)だから、
頂点のy座標の範囲は浮かず沈まずで、
-1/4≦b-a^2/4≦0
辺々-4倍すると求める範囲となる。
∴0≦a^2-4b≦1
0374132人目の素数さん垢版2022/06/16(木) 16:49:05.36ID:sTx3yUoj
>>372
n=1のとき解はx=1 以下n>1とする
nlogx=xlogn n/logn=x/logx y=x/logx 0<x<1のとき負 1<xのとき正
dy/dx=(logx-1)/(logx)^2 0<x<eのとき減少 e<xのとき増加
e<xのときyは単調増加でeよりいくらでも大きくなり
1<x<eのとき単調減少でいくらでも大きい数からeに近づく
y=mの解は m<0とき一つ 0≦m<eのときなし m=eのとき一つ m>eのとき2つ
m=n/lognと置くとmはeより大きいのでy=mの解は2つあり一つはx=nで自明
非自明な方は解き方が分からないので教えてください
0376132人目の素数さん垢版2022/06/16(木) 22:49:49.43ID:idXYmLu5
数列a[0],a[1],a[2],……は Σ_[k,0,n-1]a[k]=1/(1+n^2) (n=1,2,…)を満たす。
任意の自然数mに対し、Σ_[k,1,m] k*a[m-k] <5/4 を示せ。

此れはどう解けばいいですか。
0378132人目の素数さん垢版2022/06/16(木) 23:05:37.86ID:idXYmLu5
a[0]は正ですが
0380132人目の素数さん垢版2022/06/17(金) 00:12:48.52ID:Un+Ku3Rt
a[0]=Σ[k=0,1-1]a[k]=1/(1+1^2)=1/2
n>0のとき a[n]=Σ[k=0,n]a[k]-Σ[k=0,n-1]a[k]=1/(1+(n+1)^2)-1/(1+n^2)
=(1+n^2-(1+(n+1)^2))/(1+(n+1)^2)/(1+n^2)
>-(2n+1)/(n^2(n+1)(n+1/2))=-2/(n^2(n+1))
n*a[n]>-2/(n(n+1))=2(1/(n+1)-1/n)

Σ[k=1,n-1]a[k]=Σ[k=0,n-1]a[k]-a[0]=1/(1+n^2)-1/2≦0 等号成立はn=1

Σ[k=1,m](m-k)*a[k]=mΣ[k=1,m]a[k]-Σ[k=1,m]k*a[k]
<0-Σ[k=1,m]2(1/(k+1)-1/k)=-2(1/(m+1)-1)<2<5/4
0381132人目の素数さん垢版2022/06/17(金) 01:05:44.17ID:Un+Ku3Rt
間違えた
Σ[k=1,m](m-k)*a[k] でなくて Σ[k=1,m]k*a[m-k]だった
[k=1,7]k*a[k]>-1だから

Σ[k=1,m]k*a[m-k]=Σ[k=0,m-1](m-k)*a[k]
=mΣ[k=0,m-1]a[k]-Σ[k=0,m-1]k*a[k]
=m/(1+m^2)-Σ[k=1,7]k*a[k]-Σ[k=8,m-1]k*a[k]
<m/(1+m^2)-(-1)-Σ[k=8,m-1]2(1/(k+1)-1/k) 
=m/(1+m^2)+1-2/m+2/8
={m/(1+m^2)-2/m}+4/5<4/5
0382132人目の素数さん垢版2022/06/17(金) 01:08:31.54ID:Un+Ku3Rt
間違えた
✕={m/(1+m^2)-2/m}+4/5<4/5
○={m/(1+m^2)-2/m}+5/4<5/4
0385132人目の素数さん垢版2022/06/17(金) 09:47:50.95ID:9S/OlvJL
S[m]=Σ_[k,1,m] k*a[m-k]
とおけば、S[1]=a[0]=Σ_[k,0,0]a[k]=1/(1+1^2)=1/2
m≧2に対して
S[m]-S[m-1]
=a[m-1]-a[m-2] +2a[m-2]-2a[m-3]+3a[m-3]-3a[m-4]+…-(m-1)*a[0]+m*a[0]
=a[m-1]+a[m-2]+a[m-3]+…+a[0]
=Σ_[k,0,m-1] a[k]
=1/(1+m^2)
両辺の狽ニれば S[m]-S[1]=Σ_[k,2,m]1/(1+m^2)となるので、
S[m]=Σ_[k,1,m]1/(1+m^2) <∫[0,∞]1/(1+x^2)dx
x=tanθとおいて置換積分すれば
∫[0,∞]1/(1+x^2)dx=∫[0,π/2]dθ=π/2
あれ?π/2 >5/4か、、、、ちっ!
仕事の時間だから、あとは誰か詰めといて。
0386132人目の素数さん垢版2022/06/17(金) 17:34:50.58ID:uqfW4H9d
2n個の重りw_1,w_2,...,w_2nがあり、その重さg_1,g_2,...,g_2nは2n^2以下の正整数であるという。
このときg_i+g_j=g_a+g_bなるn以下の整数の組(i,j,a,b)で、i≠aかつi≠b であるものは存在するか。
0388132人目の素数さん垢版2022/06/17(金) 20:05:47.12ID:9S/OlvJL
>>385
×両辺の狽ニれば S[m]-S[1]=Σ_[k,2,m]1/(1+m^2)となるので、
○両辺の狽ニれば S[m]-S[1]=Σ_[k,2,m]1/(1+k^2)となるので、

f(x)=1/(1+x^2)はx>0で単調減少より、
Σ_[k,2,m]1/(1+k^2) < ∫[1,∞]f(x)dx =∫[π/4,π/2]1/(1+tan^2θ) dθ/d(tanθ)=π/4
よって、S[m]=S[1]+Σ_[k,2,m]1/(1+k^2) <1/2+π/4 =(2+π)/4
うーん、(2+π)/4>5/4かよ!

じゃ、m≧3で、
S[m]=S[1]+1/5+Σ_[k,3,m]1/(1+k^2) < 1/2+1/5 +(π/2 - arctan 2 )
arctan√3 =π/3 < arctan2 より、
S[m] < 7/10 +π/2 - arctan2 < 7/10 +π/2 - π/3 =7/10 + π/6 =(42+10π)/60 <75/60 =5/4
S[m]は単調増加だから、すべての自然数mに対してS[m]<5/4
0389132人目の素数さん垢版2022/06/17(金) 20:06:18.64ID:Atvy2FXz
a,bは0≦a<b<2πの実数とする。
xy平面上の円C:x^2+y^2=1上の相異なる2点A(cosa,sina),B(cosb,sinb)を両端とする劣弧ABを考える。
いま、xy平面のy>0の方向から、y軸に平行な光が降り注いでいる。このとき、劣弧ABが光を遮ることにより直線y=-2にできる影の長さをa,bで表せ。
0390132人目の素数さん垢版2022/06/17(金) 20:15:24.25ID:9S/OlvJL
すまん、[k=2,m]とか書くべきところを、[k,2,m]とか書いてしまってたわ。

ところで、lim S[m] = 農[k=1,∞]1/(1+k^2) の収束値はなんなん?
0391132人目の素数さん垢版2022/06/17(金) 20:31:57.65ID:9S/OlvJL
自分で検索してみたら、1/(a^2+k^2) = πcosh(πa)/2asinh(πa) -1/2a^2
って公式がめっかったわ。

ってことは、 lim S[m] = (π/2)(e^π+1/e^π)/(e^π-1/e^π) -1/2 ≒1.076684…

つまらん。
0392132人目の素数さん垢版2022/06/18(土) 06:55:27.52ID:ODah8i+s
ありがとうございます!!
0393132人目の素数さん垢版2022/06/18(土) 08:32:45.12ID:WOikZCoB
>>389
1-max(cosa,cosb)をp
max(cosa,cosb)-min(cosa,cosb)をq
1+min(cosa,cosb)をrとすると
劣弧が(1,0)を含む場合はp+q、(-1,0)を含む場合はq+r、それ以外はq

劣弧が(1,0)を含むのはsinaとsinbが異符号でb-a>πのとき
劣弧が(-1,0)を含むのはsinaとsinbが異符号でb-a<πのとき
劣弧が(-1,0)も(1,0)も含まないのはsinaとsinbが同符号のときだから長さは
q+(1-sgn(sina*sinb))/2*{(1+sgn(b-a-π))/2*p+(1-sgn(b-a-π))/2*r}
0394イナ ◆/7jUdUKiSM 垢版2022/06/18(土) 10:17:04.75ID:6hDQrTFC
>>375
>>389
b≦πのときcosa-cosb
a<π<bのとき、
b-π<π-aすなわちa+b<2πのとき、
1+cosa
b-π>π-aすなわちa+b>2πのとき、
1+cosb
π≦aのときcosb-cosa
0395イナ ◆/7jUdUKiSM 垢版2022/06/18(土) 10:51:52.64ID:6hDQrTFC
>>394訂正。
>>389
(i)b-a<πのとき、
b≦πのときcosa-cosb
a<π<bのとき、
b-π<π-aすなわちa+b<2πのとき、
1+cosa
b-π>π-aすなわちa+b>2πのとき、
1+cosb
π≦aのときcosb-cosa
(ii)b-a≧πのとき、
a<π<bのとき、
b-π<π-aすなわちa+b<2πのとき、
1-cosb
b-π>π-aすなわちa+b>2πのとき、
1+cosb
0396イナ ◆/7jUdUKiSM 垢版2022/06/18(土) 10:57:19.92ID:6hDQrTFC
>>395訂正。
>>389
(i)b-a<πのとき、
b≦πのときcosa-cosb
a<π<bのとき、
b-π<π-aすなわちa+b<2πのとき、
1+cosa
b-π>π-aすなわちa+b>2πのとき、
1+cosb
π≦aのときcosb-cosa
(ii)b-a≧πのとき、
a<π<bのとき、
b-π<π-aすなわちa+b<2πのとき、
1-cosb
b-π>π-aすなわちa+b>2πのとき、
1-cosa
0397132人目の素数さん垢版2022/06/18(土) 10:58:21.17ID:NVIq2b00
a,b,c,dは実数とする。xy平面の点(x,y)※を、以下のように点(x',y')に移す。
(x',y')=(ax+by,cx+dy)
このとき以下の問に答えよ。

(1)(x',y')=(x,y)となる(x,y)が少なくとも1つ存在するとき、a,b,c,dが満たすべき条件を求めよ。

(2)a,b,c,dがすべての実数を動くとき、(x',y')=(x,y)となる(x,y)がいくつ存在するか求めよ(無数に存在する場合も含む)。

(3)a,b,c,d,x,yがすべて0でない相異なる整数であるとき、(x',y')=(x,y)となる(x,y)は存在するか。存在するならば、そのa,b,c,d,x,yを一組求めよ。
0398132人目の素数さん垢版2022/06/18(土) 10:59:59.53ID:FC+H+wBw
>>325
Adleman-Pomerance-Rumely primality テスト - うぃきぺぢあ

APR-CL素数判定法 - wacchoz’s note

コンピュ-タと素因子分解 / 和田 秀男【著】 - 紀伊國屋書店ウェブストア|オンライン書店|本、雑誌の通販、電子書籍ストア
第7章 アドレマン‐ルメリー法
0399イナ ◆/7jUdUKiSM 垢版2022/06/18(土) 12:43:15.09ID:6hDQrTFC
>>396
>>397(1)
x'=ax+by=x
y'=cx+dy=y
(a-1)x+by=0
cx+(d-1)y=0
a=d=1,b=c=0
または(a-1)(d-1)x+b(d-1)y=0
bcx+b(d-1)y=0
辺々引いてad-bc=1
ad-bc=0はa=d=1,b=c=0を包含している。
∴ad-bc=1
(2)a,b,c,dが無数に存在するので、
(x',y')=(x,y)となる(x,y)は無数に存在する。
(3)a=5,b=-2,x=3,y=6のとき、
c=0,d=1
0400イナ ◆/7jUdUKiSM 垢版2022/06/18(土) 12:46:55.40ID:6hDQrTFC
>>399訂正。
>>397(1)
x'=ax+by=x
y'=cx+dy=y
(a-1)x+by=0
cx+(d-1)y=0
a=d=1,b=c=0
または(a-1)(d-1)x+b(d-1)y=0
bcx+b(d-1)y=0
辺々引いてad-bc=1
ad-bc=0はa=d=1,b=c=0を包含している。
∴ad-bc=1
(2)a,b,c,dが無数に存在するので、
(x',y')=(x,y)となる(x,y)は無数に存在する。
(3)a=5,b=-2,x=3,y=6のとき、
c=8,d=-3
0402132人目の素数さん垢版2022/06/19(日) 00:43:19.80ID:3d6Ebr6v
方程式 1/(x^2+3x+2) + 1/(x^2+5x+6) = k が
異なる2つの実数解をもつとき、実数の定数kの値の範囲はどうなりますか?
0403132人目の素数さん垢版2022/06/19(日) 01:06:01.92ID:X4QNjPPu
1/(x+1)/(x+2)+1/(x+2)/(x+3)=k
(x+3)+(x+1)=k(x+1)(x+2)(x+3)
1/k=(x+1)(x+2)(x+3)/(2*(x+2))
2/k=(x+1)(x+3)
x^2+4x+3-2/k=0
判別式=4-(3-2/k)=1+2/k>0
k^2+2k=k(k+2)>0
k<-2または0<k
0404132人目の素数さん垢版2022/06/19(日) 01:51:43.96ID:qo4X3/mu
△ABCの周および内部の領域に2つの長方形を置き、それらの面積の合計が最大となるようにする。
このとき以下の(1)(2)が成り立つか調べ、結論を述べよ。

(1)少なくとも1つの長方形の1つの辺は、△ABCのいずれかの辺に含まれる。
(2)少なくとも1つの長方形は正方形である。
0405132人目の素数さん垢版2022/06/19(日) 03:30:15.35ID:X4QNjPPu
△ABCの長い辺、例えばBC上に長方形を置きその上に小さい長方形を置くとする
下に置いた長方形の左上の角をp、上に置いた長方形の左上の角をqとする
p,qはAB上にあり、Ap:pq:qBはx:y:z、0<x,y,z<1、x+y+z=1であるとする
全体の△ABCから2つの長方形を除いたら5個の三角形が残るが
それらの面積の和は全体の△ABCのx^2倍、y^2倍、z^2倍の和となる
↑(x,y,z)と↑(1,1,1)の内積を考えると内積はx+y+z=1だが
内積はなす角が0のとき最大となりベクトルの長さの積
√(x^2+y^2+z^2)√(1^2+1^2+1^2)となる
内積の平方=1^2≦(x^2+y^2+z^2)*3 つまり 1/3≦x^2+y^2+z^2
等号成立は両ベクトルのなす角が0つまりx=y=z=1/3のとき
このとき5個の小さな三角形の面積の和は△ABCのx^2+y^2+z^2=1/3倍に最小化
されるので2つの長方形の面積の和は△ABCの2/3倍に最大化できる
0406132人目の素数さん垢版2022/06/19(日) 09:17:47.00ID:6Ox1M8+M
今日の作問爺ID:qo4X3/muとその相棒ID:X4QNjPPu
ふたりとも延々とスレ違い投稿を続けるソシパス。

せめてコテハンにしろよ。さもなきゃ死んでくれ。
0408132人目の素数さん垢版2022/06/19(日) 09:26:09.43ID:3d6Ebr6v
>>403
3行目で 1/k が登場しますが
k=0のときは別に考えなければだめですか?
0409132人目の素数さん垢版2022/06/19(日) 09:36:13.07ID:fBILhsD6
abc/defは(abc)/(def)と見なす
0410132人目の素数さん垢版2022/06/19(日) 09:55:53.53ID:fBILhsD6
>>408
そりゃそうだけど
2/(x+1)(x+3)=k
だからあり得ない
また
x^2+4x+3=2/k
がx=-1,-2,-3を解に持つ場合は除外だから
2/k=0,-1,0
すなわちk=-2は除外

ちょうど除外されてるのか
1/x(x+1)-3/x(x+3)=k
とかだと
-2/(x+1)(x+3)=k
から
(x+1)(x+3)+2/k=0
D/4=4-(3+2/k)=1-2/k>0
k^2-2k=k(k-2)>0
k<0または2<k
でx=0,-1,-3は除外だから
-2/k=3,0,0
すなわちk=-2/3は除外で
k<-2/3,-2/3<k<0,2<k
みたくなる
0411132人目の素数さん垢版2022/06/19(日) 10:04:51.97ID:z0L/UyVW
この人の解説すごくわかりにくい
0412イナ ◆/7jUdUKiSM 垢版2022/06/19(日) 10:15:17.98ID:/q78BUfc
>>400
>>402
1/(x+1)(x+2)+ 1/(x+2)(x+3)=k
x+3+x+1=k(x+1)(x+2)(x+3)
2(x+2)=k(x+1)(x+2)(x+3)
2=k(x+1)(x+3)
kx^2+4kx+3k-2=0
D/4=4k^2-k(3k-2)>0
k^2+2k>0
(k+2)k>0
∴ k<-2,0<k
0413テメーが死ねヴォケ6Ox1M8+M垢版2022/06/19(日) 10:16:27.69ID:X4QNjPPu
>>406
了解しました
0414132人目の素数さん垢版2022/06/19(日) 10:45:10.92ID:fBILhsD6
>>410
>x^2+4x+3=2/k
>がx=-1,-2,-3を解に持つ場合は除外だから
>2/k=0,-1,0
>すなわちk=-2は除外
除外はされていて答えは変わらないけど
この吟味はするべき
0415132人目の素数さん垢版2022/06/19(日) 11:14:55.34ID:rHBDsfq1
実数tの小数点以下第2位までをa[t]とする。
例えばt=√3であればa[t]=0.73、t=3.3であればa[t]=0.3、t=5であればa[t]=0である。a[t]の整数部分は0とし、小数点以下の3桁目以降は切り捨てる。

(1)a[t]=7.7となるtをすべて求めよ。
(2)a[t]=7.77となるtをすべて求めよ。
0417132人目の素数さん垢版2022/06/19(日) 11:29:39.11ID:6Ox1M8+M
>>414
判別式でなくグラフを使って解くのならその形のほうがいいと思うが、
kx^2+4kx3k-2 =0 としたほうが素直じゃないか?
まあ、どっちでもいいけど。

グラフを使った解法だと、x=-2が除外されることは一目瞭然。
0421132人目の素数さん垢版2022/06/19(日) 11:36:09.50ID:BHOfDNIM
なんか、生産性無いスレだね
0422132人目の素数さん垢版2022/06/19(日) 11:40:30.14ID:YBPQ8RrW
>>421
このスレにおける生産性を以下のように定義する。
・ある書き込みの生産性は、以下の2つの整数値の合計である
(ア)n文字の書き込みの生産性はn
(イ)k個のレスがついている書き込みの生産性は10^k

この投稿の生産性を求めよ。
0423132人目の素数さん垢版2022/06/19(日) 11:46:15.43ID:pTmq1+Ma
>>422
わかった
炎上すればいいんだね
0424132人目の素数さん垢版2022/06/19(日) 11:46:40.19ID:fBILhsD6
>>421
(ア)13+(イ)10^2=113
0425132人目の素数さん垢版2022/06/19(日) 13:54:34.27ID:TTpnU9OR
すみません、この問題なんですけど、『すべてはずれるようなテスト結果がf(k)通りある』の意味がわかりません
どう考えればいいですか

https://i.imgur.com/mMssScI.jpg
0426132人目の素数さん垢版2022/06/19(日) 13:57:12.92ID:fBILhsD6
>>425
酷い文章だな
山梨医大のお里が知れる(山梨)
0428132人目の素数さん垢版2022/06/19(日) 14:02:38.35ID:fBILhsD6
>>425
k個のものの正体を当てるときに
それぞれがどれであるかを答えるから
k個それぞれについて正体として答えるものがk通り考えられるが
k個の異なるものに対して同じものを答えないと考えて(ここが曖昧)
k個についての答え方がk!通り
その中ですべてが外れている答え方をf(k)通りと定義して
f(k)を求めさせようということ
0429132人目の素数さん垢版2022/06/19(日) 14:08:38.28ID:TTpnU9OR
>>428
迅速な解説ありがとうございます
とてもわかりやすいです!
0430132人目の素数さん垢版2022/06/19(日) 14:12:40.74ID:fBILhsD6
n個の場合にn個についての答え方が重複して良いとするとn^n通り
重複もある答え方はそのうちのk個が正しくないとすれば
n-k個は正しいそれがnこのうちどれかがnCn-k通りk個が全部正しくないからf(k)通りで
k小型だ市区内答えからがnCn-kf(k)通りある
kとしては0からnまで排反に考えることが出来るので
n^n=Σ[k=0,n]nCn-kf(k)
0431132人目の素数さん垢版2022/06/19(日) 16:04:15.32ID:6Ox1M8+M
>>430
まあ、常識的に考えて>>428でいいんだろうけど、たしかに曖昧だね。
「異なるグラスには異なる銘柄をあてはめるようにして当てる」と
いうような条件つけとくべきだね。

n個のグラスに対して、n個の銘柄名をどれにいれるかはn!通り。
そのうち正解は1通りなので、n!-1がハズレとなる。
n個のうちk個だけがはずれであとは正解となるのはnCn-kf(k)通りだが、
1個だけ外れることはありえないので、k≧2
よって、ハズレとなる答え方は 納k=2,n]f(k) =n! -1
0434132人目の素数さん垢版2022/06/19(日) 20:39:33.87ID:YBPQ8RrW
直交座標で表しても極座標で表しても座標が同じになる点をすべて求めよ。
0435132人目の素数さん垢版2022/06/19(日) 21:08:21.81ID:RrP90q6t
x軸の正の部分と原点?
0436132人目の素数さん垢版2022/06/19(日) 22:28:54.08ID:fBILhsD6
>>425,431
f(n+1)通りの間違いのそれぞれについて
n+1個目がk番目の答えになっていたとして
n+1番目の答えがk個目の場合とそうで無い場合に分けて
k個目の場合はこの2つの答えを入れ替えたら両方正解になって
あとのn-1個は全部不正解のままつまりf(n-1)通り
k個目で無い場合はこの2つの答えを入れ替えたらn+1番目の答えは正解だけどk番目の答えは不正解のままつまりf(n)通り
kとしては1〜nまで考えられるから
f(n+1)=n(f(n-1)+f(n))
なおf(0)=1,f(1)=0
順に
1 0 1 2 9 44 265 …
0437132人目の素数さん垢版2022/06/20(月) 00:13:22.79ID:+8hq6mcn
無理数の証明について質問です

「a,bを有理数とする。a+b√2=0⇒a=b=0であることを証明せよ。ただし、√2が無理数であることは用いて良い」

という問題について、塾の生徒が次のような解答を持ってきましたが?されており隣に背理法で証明しましょうと書いてありました。
何がダメか聞かれたのですが、私もどこがダメか分からないため教えて頂けると嬉しいです

(解答)
対偶をとるとa≠0またはb≠0⇒a+b√2≠0

(1)a≠0かつb=0のとき a+b√2=a≠0
(2)a=0かつb≠0のとき a+b√2=b√2≠0
(3)a≠0かつb≠0のとき a,bは有理数、√2は無理数なのでa+b√2=有理数+無理数≠0

(1)~(3)より対偶が証明されたので、元の命題も証明された。

宜しくお願いします!
0438132人目の素数さん垢版2022/06/20(月) 00:50:16.66ID:uVksNd7u
そりゃ有理数+無理数≠0を当たり前みたいに使ったらあかんって意味じゃないの
0439132人目の素数さん垢版2022/06/20(月) 00:53:45.19ID:XM8QImeR
>>436
brilliant!

グラスに番号をつけて、入ってるワインの銘柄をその番号順に並べておくと考えればよいのね。
でもって、k番目のグラスをσ(k)番目の銘柄に指定するという解答において、σはS={1,2,...,n}
からSへの全単射になってるわけね。σのうち不動点を持たないものをσ'とするとその総数Ñ(σ')=f(n)。

グラスと銘柄がn+1個の場合を考え、σ'(n+1)=k (k≠n+1) となるσ'_kが何通りあるか考える。
i)そのうちσ'_k(k)=n+1であるものは、{1,2,..,k-1,k+1,...n}におけるσ'になっているので、f(n-1)通りある。
ii)そのうちσ'_k(k)≠n+1となるものは、{1,2,...n}におけるσ'になっているのでf(n)通り。
よって、f(n+1) =納k=1,n]N(σ'_k)=納k=1,n]{f(n-1)+f(n)}=n{f(n-1)+f(n)}
ただし、
n=1の場合はσ'は存在しないので、 f(1)=0
n=2の場合、(σ'(1),σ'(2))=(2,1)だけなのでf(2)=1
0440132人目の素数さん垢版2022/06/20(月) 01:38:46.16ID:ytydw+h7
>>425
全正解が1,2,,,,,kとすると
すべてはずれる場合を列挙させてみる
k=4の場合
> calc(4)
[,1] [,2] [,3] [,4]
[1,] 2 1 4 3
[2,] 2 3 4 1
[3,] 2 4 1 3
[4,] 3 1 4 2
[5,] 3 4 1 2
[6,] 3 4 2 1
[7,] 4 1 2 3
[8,] 4 3 1 2
[9,] 4 3 2 1

k=1-10までをカウント

[,1] [,2] [,3] [,4] [,5] [,6] [,7] [,8] [,9] [,10]
[1,] 0 1 2 9 44 265 1854 14833 133496 1334961
0442132人目の素数さん垢版2022/06/20(月) 08:08:41.41ID:JSmDI/YE
bが0でない有理数のときb√2が無理数なのが自明って、結局元の問題が自明ってことじゃん
0443132人目の素数さん垢版2022/06/20(月) 08:16:30.58ID:ytydw+h7
>>425
>>440
応用問題

10種類のワインの銘柄を当てる。
同じ銘柄を複数回答えてはならない。
(1)無作為に答えるとき正解の数が最も多いのはいくつか?
(2)ワインの数が9種類のときではいくつになるか?
0444132人目の素数さん垢版2022/06/20(月) 08:19:16.71ID:XM8QImeR
>>437
(3)で有理数と無理数の積が無理数になることを背理法で示す必要があるんじゃない?
それは、√2 = -b/a が成り立つとすると√2が有理数になってしまうので矛盾する
という背理法による証明と同じになるよね。
0446132人目の素数さん垢版2022/06/20(月) 12:44:57.62ID:+8hq6mcn
>>438,442
なるほど!確かに足し算と掛け算は問題文に書かれてないので使ったらまずかったですね

>>444
対偶を使っても結局背理法に行きつくんですね。無駄に場合分けを増やしただけになってますね…


ありがとうございました!
0447132人目の素数さん垢版2022/06/20(月) 13:29:42.37ID:4BbGW9u+
cosx=1/3を満たす実数x(0<x<π/2)を考える。

(1)a[n]=cos(nx),b[n]=sin(nx)とするとき、a[n]とb[n]をcosxとsinxで表せ。

(2)n≧2のとき、a[n]は無理数であることを示せ。
0449132人目の素数さん垢版2022/06/20(月) 18:39:03.67ID:I6pDpaSs
n種類のワインの銘柄を当てる。
同じ銘柄を複数回答えてはならない。
無作為に答えるとき正解の数が最も多いのはいくつか?

予想 : nが偶数のとき0,奇数のとき1
0451132人目の素数さん垢版2022/06/20(月) 21:22:48.32ID:No1bKPNG
f^[1](b)=f^[3](b)=0 であっさり解ける
もしかしたら、合成関数の微分や三次導関数はこの学校の入試範囲外かもしれないけど
0453132人目の素数さん垢版2022/06/20(月) 21:56:49.58ID:2Rls5y8J
x^4+8x^3+ax^2-20x=(x-b)^4+A(x-b)^2+B
0456132人目の素数さん垢版2022/06/21(火) 13:43:06.35ID:O0TqJyhh
cosx=1/3を満たす実数x(0<x<π/2)を考える。

(1)a[n]=cos(nx),b[n]=sin(nx)とするとき、a[n]とb[n]をcosxとsinxで表せ。

(2)n≧2かつn≠2^k(k=1,2,...)のとき、a[n]は無理数であることを示せ。
0460132人目の素数さん垢版2022/06/21(火) 20:50:55.00ID:ZE5+gcQp
xy平面上に円を、円周上に格子点がちょうど3個になるように配置できますか?
0462132人目の素数さん垢版2022/06/21(火) 22:06:53.18ID:Zn4vpoyA
友達を傷つけて 
それでも手に入れる
愛があると 信じたわ
0466イナ ◆/7jUdUKiSM 垢版2022/06/22(水) 10:49:05.01ID:ts8l5ij4
>>412
>>460
上図のように点(c,0)を中心に、
3点(c+r,0),(c-rcosθ,rsinθ), (c-rcosθ,-rsinθ)をとると、
c+r=2
rsinθ=1
rcosθ=1+c
これらを解いてr=3/(1+cosθ)=1/sinθ
3sinθ=1+cosθ
9sin^2θ=1+2cosθ+cos^2θ
9-9cos^2θ=1+2cosθ+cos^2θ
10cos^2θ+2cosθ-8=0
5cos^2θ+cosθ-4=0
(5cosθ-4)(cosθ+1)=0
cosθ=4/5
r=3(5/9)=5/3
c=2-5/3=1/3
(x-1/3)^2+y^2=(5/3)^2=25/9
∴3x^2-2x+3y^2=8
0467132人目の素数さん垢版2022/06/22(水) 10:49:47.80ID:2TsYz3WQ
>>460
できるな。
ようわからんけど、3個しかないとすると、その3点のx,y座標はすべて異なるはず。
(同じ座標値のものがあるとすると、その3点で直角三角形になるので、直径となる
斜辺を共有するもう一つの直角三角形の頂点も演習上の格子点になる)
その円を平行移動してどれか1点を原点にもってくれば、他の2点は(m,n),(k,l)
m≠k≠0,n≠l≠0 とおける。でもって、m,n,k,lの組み合わせをできるだけ小さい
ものから選んで(m-a)^2+(n-b)^2=r^2,(k-a)^2+(l-b)^2=r^2, a^2+b^2=r^2を
wolframalpha様に解いてもらう。出てきたa,b,rを使って(N-a)2+(y-b)^2=r^2を
Nとりうる整数値を代入してwolframalpha様に解いてもらって(m,n),(k,l),(0,0)
以外の整数解がないことを確認する。以上の手順で見つかる。
0468132人目の素数さん垢版2022/06/22(水) 11:08:45.38ID:2TsYz3WQ
とりあえず試行錯誤で1つ見つかった。
(0,0),(3,2),(5,1)を通過する円 (x-39/14)^2+(x+13/14)^2 =845/98

もっと小さいのがあるかもしれんが、めんどくさいので誰か探して。
0469132人目の素数さん垢版2022/06/22(水) 11:17:05.51ID:2TsYz3WQ
>>467
すまん、
>同じ座標値のものがあるとすると、その3点で直角三角形になるので、
じゃなくて、対称性から、同じ座標値を共有する2点を結ぶ線分の垂直二等分線
に対して残りの1点と対称な点も円周上にくるが、その座標値も整数になるのは
明らか。
0472イナ ◆/7jUdUKiSM 垢版2022/06/22(水) 12:32:30.18ID:ts8l5ij4
>>471
中心(c,c)の円は(x-c)^2+(y-c)^2={(1-c)√2}^2
ピタゴラスの定理より中心(c,c)と(1,1)の距離と、
点(0,-1)との距離が等しいから、
2(1-c)^2=c^2+(1+c)^2
これを解いてc=1/6
(x-1/6)^2+(y-1/6)^2=(5√2/6)^2
(6x-1)^2+(6y-1)^2=50
36x^2-12x+36y^2-12y=48
3x^2-x+3y^2-y=4
最小半径5√2/6
0474132人目の素数さん垢版2022/06/22(水) 12:48:46.83ID:2TsYz3WQ
>>469
再び訂正
>対称性から、同じ座標値を共有する2点を結ぶ線分の垂直二等分線
に対して残りの1点と対称な点も円周上にくる

この場合、残りの1点は垂直二等分線上になければならない。
このケースが簡単か。A(0,0),B(0,2)、C(1,n)で、ACの垂直二等分線
に対称なBの鏡像点B'(x,y)の座標はx=3-4/(n^2+1),y=n-4n/(n^2+1)
なので、n=1はだめで、n=2ならOK。 (0,0),(0,2),(1,2)を通る円は
(x-1)^2+(y-3/4)^2 = 25/16
この整数解はその3点しかない。
半径5/4の円が最小か?
0475132人目の素数さん垢版2022/06/22(水) 12:59:45.41ID:2TsYz3WQ
>>473
おお、それが最小だね。
(0,0),(2,1),(-1,1)は探してなかったな。これで
(x-7/6)^2+(y-1/6)^2=25/16
がwolframalpha 様によって見いだされた。
0476132人目の素数さん垢版2022/06/22(水) 13:23:10.52ID:7CZv3w1/
(1)tを実数の定数とする。
xの2次方程式
x^2+(cost)x+sint=0
が実数解を持つようなcostの範囲を求めよ。

(2)xの方程式
x^2+(cosx)x+sinx=0
は何個の実数解を持つか。
0477132人目の素数さん垢版2022/06/22(水) 13:26:10.36ID:CVk+H1cV
先に解をでっち上げるのがコツだな

(0,0),(-1,1),(1,2)を通る円は(x-1/6)^2+(y-7/6)^2=50/36
円は1≦x≦1、0≦y≦2にあるからこれらが解の候補
y=7/6±√(50/36-(x-1/6)^2)だから
x=1のとき y=7/6±√(50/36-(x-1/6)^2)=7/6±√(25/36)=7/6±5/6=2,1/3
x=0のとき y=7/6±√(50/36-(x-1/6)^2)=7/6±√(49/36)=0,7/3
x=-1のとき y=7/6±√(50/36-(x-1/6)^2)=7/6±√(1/36)=1,4/3
他に整数解はなし
0478132人目の素数さん垢版2022/06/22(水) 13:32:54.00ID:qvuD6qGg
既に答え書いてあるリンク貼ってあって2行くらいで終わってるのに何やってんの?
0481132人目の素数さん垢版2022/06/22(水) 14:08:45.74ID:7CZv3w1/
俺が東京大学理系数学の出題者だったら、この6問の構成にする

1.立体図形と積分(B***)
2.複素数平面と方程式(C***)
3.確率と漸化式(C***)
4.平面図形と微分(B***)
5.整数(C***)
6.数列と論証(D****)
0483132人目の素数さん垢版2022/06/22(水) 14:13:50.12ID:X6WRwLQT
だらだらクソやきうの延長で後の番組が延びたときの絶望感
0484132人目の素数さん垢版2022/06/22(水) 14:21:37.77ID:zk7J1GTG
>>480
イヤ、もう答え出てるんやから楽しみたいなら自分一人でチラ裏でやれよ?
なんでここに書く?
0485イナ ◆/7jUdUKiSM 垢版2022/06/22(水) 14:28:11.83ID:ts8l5ij4
>>473
5/4-(5√2/6)=0.07148869802……>0
少なくとも5/4は最小ではない。
0487イナ ◆/7jUdUKiSM 垢版2022/06/22(水) 16:44:06.10ID:ts8l5ij4
>>486
>>450
x=bのときy'=4(b^3+6b^2+ab/2-5)=0
2b^3+12b^2+ab-10=0
y=b^4+8b^3+ab^2-20b
x=2bのときy=16b^4+64b^3+4ab^2-20b=0
4b^3+16b^2+ab-5=0
bの3次式どうし辺々引くと、
2b^3+4b^2+5=0
左辺を微分し6b^2+8b=0となるのは、
b=0,-4/3のとき……
bは-2よりやや小さい。
-5/2よりやや大きい。
0488132人目の素数さん垢版2022/06/22(水) 16:46:34.67ID:1n55kXJu
さすがイナさん!
0490132人目の素数さん垢版2022/06/22(水) 18:44:16.73ID:CVk+H1cV
x^4+8x^3+ax^2-20x=(x-b)^4+A(x-b)^2+B
=x^4-4bx^3+6b^2x^2-4b^3x+Ax^2-2Abx
3次の係数 8=-4b b=-2
x=b=-2のとき y'=4b^3+24b^2+2ab-20=4{-8+24-a-5}=0 a=11
0492イナ ◆/7jUdUKiSM 垢版2022/06/22(水) 19:12:26.96ID:R4LP7AoW
>>487訂正。
>>450勘で解きなおす。
y=x^4+8x^3+ax^2-20x
=x(x^3+8x^2+ax-20)
=(x+5)(x+4)x(x-1)
=x^4+8x^3+11x^2-20x
∴a=11,b=-2
0494132人目の素数さん垢版2022/06/22(水) 19:54:01.70ID:2TsYz3WQ
ああ、誰かの導き出したことを受け売りするしか能がないのか。
哀れだねw
0495132人目の素数さん垢版2022/06/22(水) 19:54:16.06ID:ElpfG1N8
n:evenとする
(x-1/2)²+y² = 1/4 5^(n/2-1) ( n : even )‥①
①⇔(2x-1)²+(2y)² = 5^(n/2-1)
u²+v²=5^(n/2-1)の整数解の個数は
u + vi = iᵐ(1+2i)ᵏ(1-2i)ˡ (m=0,1,2,3,k+l = n/2-1)
の個数だから2n個ある
全ての(u,v)のうちいずれがちょうど片方が奇数でありuが奇数である解全体とvが奇数である解の全体は(u,v)→(v,u)で移り合うからuが奇数である解はちょうどn個である

n:oddとする
(x-1/3)²+y² = 1/9 5^(n-1) ( n : odd )‥②
②⇔(3x-1)²+(3y)² = 5^(n-1)
u²+v²=5^(n-1)の整数解の個数は
u + vi = iᵐ(1+2i)ᵏ(1-2i)ˡ (m=0,1,2,3,k+l = n-1)
の個数だから4n個ある
全ての(u,v)は
u ≡ 1, v ≡ 0 ( mod 3 )‥A⃝
u ≡ 2, v ≡ 0 ( mod 3 )‥B⃝
u ≡ 0, v ≡ 1 ( mod 3 )‥C⃝
u ≡ 0, v ≡ 2 ( mod 3 )‥D⃝
のいずれかを満たすがA⃝とB⃝は対応(u,v)→(-u,-v)でC⃝とD⃝も同じ対応で、A⃝とC⃝は対応(u,v)→(v,u)で一対一に対応するから全て同じ個数である
よってA⃝に属する解はちょうどn個である
0497132人目の素数さん垢版2022/06/22(水) 20:12:07.81ID:2TsYz3WQ
>>496
なぜ?
高校数学の質問に答えただけで、誰かに挑戦してるわけではないよ。

最初から受け売りの答えでもいいから書けばよかっただけじゃん。
馬鹿にもほどがあるわw
0501132人目の素数さん垢版2022/06/22(水) 21:45:30.24ID:/08V95Tp
xy平面の単位円C:x^2+y^2=1上の点P_t(cost,sint)に対して、点Q_t(cost+sint,costsint)を考える。
ただしtは0≦t<2πの実数である。
点Q_tが動きうる領域の長さを求めよ。
0503132人目の素数さん垢版2022/06/22(水) 21:52:35.04ID:iLvFkZ4z
大先生でも無理

https://www.wolframalpha.com/input?i=parametric+plot&assumption=%7B%22F%22%2C+%22ParametricPlotCalculator%22%2C+%22upperrange1%22%7D+-%3E%222*%CF%80%22&assumption=%7B%22F%22%2C+%22ParametricPlotCalculator%22%2C+%22lowerrange1%22%7D+-%3E%220%22&assumption=%7B%22F%22%2C+%22ParametricPlotCalculator%22%2C+%22variable1%22%7D+-%3E%22u%22&assumption=%7B%22C%22%2C+%22parametric+plot%22%7D+-%3E+%7B%22Calculator%22%7D&assumption=%7B%22F%22%2C+%22ParametricPlotCalculator%22%2C+%22xfunction%22%7D+-%3E%22cos%28u%29%2Bsin%28u%29%22&assumption=%7B%22F%22%2C+%22ParametricPlotCalculator%22%2C+%22yfunction%22%7D+-%3E%22sin%28u%29*cos%28u%29%22&lang=ja
0504132人目の素数さん垢版2022/06/22(水) 22:21:16.74ID:Ao9yryRw
(1/2)^n * tan(pi/2^n) の, n=2から∞までの無限級数は
どう求めればいいですか?

tan がついてなければ簡単なのですが、tanがつくときの公式が分かりません。
0505132人目の素数さん垢版2022/06/22(水) 22:36:14.72ID:F6nf2nnT
tan(2x) = 2tan(x)/(1-tan²(x))
2/tan(2x) = 1/tan(x) - tan(x)
tan(x) = 1/tan(x) - 2/tan(2x)
∴(1/2)ⁿtan(π/2ⁿ) = 1/(2ⁿtan(π/2ⁿ)) - 1/(2ⁿ⁻¹tan(π/2ⁿ⁻¹))
足す
0506132人目の素数さん垢版2022/06/22(水) 22:48:07.73ID:HJ9uZz2a
>>503
x=cost+sint
y=costsint
x^2=1+2y
y=(1/2)(x^2-1)
放物線の-√2≦x≦√2の部分だから
2∫[0,√2]√(1+x^2)dx
=2∫[0,arctan√2](1/cos^3θ)dθ
=2∫[0,arctan√2](cosθ/(1-sin^2θ)^2dθ
=2∫[0,√(2/3)](1/I1-t^2)^2)dt
=(1/2)∫[0,√(2/3)](1/(1-t)+1/(1+t)+1/(1-t)^2+1/(1+t)^2)dt
=(1/2)(log(1+√(2/3))/(1-√(2/3))+1/(1-√(2/3))-1/(1+√(2/3)))
=(1/2)(log(1+√(2/3))^2/(1/3)+2√(2/3)/(1/3))
=log(1+√(2/3))/√(1/3)+√6
=√6+log(√3+√2)
0507132人目の素数さん垢版2022/06/22(水) 22:50:53.35ID:HJ9uZz2a
>>501
>点Q_tが動きうる領域の長さ
放物線を2重に辿るのだけど
動きうる領域(という言い方はどうかと思うが)の長さだから2重にしてはダメ
0508132人目の素数さん垢版2022/06/22(水) 23:01:11.17ID:2TsYz3WQ
>>499
答の求め方は一通りでなくてもいいだろ。
あんた、自分の書き込みが無視されたからって、ムキになってるだけじゃん。
あほらし。
0509132人目の素数さん垢版2022/06/22(水) 23:04:54.52ID:Ao9yryRw
>>505
こんなの常人には思いつけるわけないと思いますが
0510132人目の素数さん垢版2022/06/22(水) 23:12:24.44ID:rh+oz9PB
>>508
頭悪いなぁ
だから自分で勉強するのは勝手にすればいいと言ってるやろ?
いちいちそのセンスゼロの解答をなんで公共の掲示板にあげる?
それが既に上がってる解答より優れてるならともかくとしてヘッタクソな能無し解答
しかもお前既に上がってる解答の意味わかってなかったやろ?
なんや>>498は?
アホなんですか?
0511 【中吉】 垢版2022/06/23(木) 00:58:10.24ID:FdX7O7bQ
>>492
>>501
部分積分が自信ないんで勘で解き、
y=x^2の-1≦x≦1の部分のながさが1.489ぐらいだから、
これを2√2倍し、
4.21152798168……
0512イナ ◆/7jUdUKiSM 垢版2022/06/23(木) 01:04:46.97ID:r7xHLlMi
>>511訂正。
>>501
部分積分が自信ないんで勘で解き、
y=x^2の0≦x≦1の部分の長さは√2より弛みのぶんだけやや長く、
1.489ぐらいだから、
これを2√2倍し、
4.21152798168……
∴約4.212
0513イナ ◆/7jUdUKiSM 垢版2022/06/23(木) 01:34:52.14ID:1vlkmx5N
>>512
501
横だけ√2倍になって長さが√2倍になるわけない。
y=x^2の0≦x≦1の部分の長さは√2より弛みのぶんだけやや長く、
1.489ぐらいだから、
これを2倍して2.978
x軸方向にのびるが√2倍はのびすぎ。
√(√2)倍で約3.5415でも長すぎる。
作図するとほぼほぼπ
0514132人目の素数さん垢版2022/06/23(木) 03:11:31.72ID:wYowfZgV
つべで見つけたんだけど

https://youtu.be/kv4I1tHDSG4

ω³=1、ω≠1とする
整式xⁿ+yⁿ+nxy-1をx+ωy-ω²で割ったときの余りが0であるとき、正整数nを求め、その商を因数分解せよ

というのが1993福井工大で出たとか言ってるんだけどそもそもℂ[x,y]はユークリッド整域の構造を複数持ち得てその各々で出た“余り”は一致しないので“余り”なんて定義できないハズ
UFDではあるので“割り切れる”とか“割り切れるときの商”は定義できるにしても“余り”なんて定義できないと思うんだけど
動画の主が勝手に改変したのか、本当にこんな欠陥ある問題が出題されたのか確かめたいんだけどググっても古すぎて出てこない
誰か持ってない?
0515132人目の素数さん垢版2022/06/23(木) 07:40:48.37ID:9c6nZhUF
相当かなりめちゃくちゃ非常にアレな問題だけど、もしかして院試の問題?
0516132人目の素数さん垢版2022/06/23(木) 07:46:40.88ID:9c6nZhUF
「余り」は定義できなくても「余り0」はうぇるでふぁいんど!定義できる(キリッ
という言い訳をするつもりだったのかも
0517132人目の素数さん垢版2022/06/23(木) 07:50:18.94ID:JIXc/OMw
>>454
f(b+x)=f(b-x)がxの恒等式になるようにa,b を求める

(b+x)^4+8*(b+x)^3+a*(b+x)^2-20*(b+x)=(b-x)^4+8*(b-x)^3+a*(b-x)^2-20*(b-x)

x*(a*b+2*(b^3+6*b^2+(b+2)*x^2-5))=0

b=-2
a=11
0519132人目の素数さん垢版2022/06/23(木) 08:47:13.59ID:t87N7Ouv
>>510
>いちいちそのセンスゼロの解答をなんで公共の掲示板にあげる?

君がセンスゼロだと思うかどうかなんて知ったこっちゃないわw
君の考えたセンスある解答を公共の利益のため自分で最初から
書き込めばいいだけの話にすぎん。

単にマウントをとりたいだけの人格異常者なんだよ、君は。
自覚して反省しろ。
0520132人目の素数さん垢版2022/06/23(木) 09:16:00.07ID:WHSHn6IS
さぁ?
とりあえず動画では“1993年に福井工大で出題された”としか言ってない
もちろんそもそもℂ係数の多項式である時点で例え1変数でも受験数学範囲外の可能性あるけど、2変数だと範囲外以前にそもそもユークリッド整域の構造が一意には決まらないので問題として成立してない
つまり院試だとしてもそもそも問題として成立してない
ユークリッド整域がどうとか言う話は数学科卒の人間いないとうっかりしてしまう可能性なくもないから本当に出題された可能性もゼロではないけどどうなんだろうと思って
流石に数学科卒の人間がこのレベルのミスはしないだろうし
0521132人目の素数さん垢版2022/06/23(木) 09:23:56.35ID:hUy6wB8i
.>>516
まぁおそらくそうなんだろうけど、それは言い訳にしかならず問題文として通用しないし数学者なら絶対やらないレベルのミス
福井工大とかだと中の人が全員工学部卒とか言う可能性もなくはないのかな?
ちょっと気になったんだけど流石に1993年だとネットには問題原文転がってなくた
0522イナ ◆/7jUdUKiSM 垢版2022/06/23(木) 09:52:40.88ID:wrdhvIL/
>>513
>>501
y=x^2の0≦x≦1の部分の長さは√2より弛みのぶんだけやや長く、
1.489ぐらいだから、
これを2倍して2.978
√(√2)倍で約3.5415はわるい数字じゃない。
縦が1、横が2√2の長方形内にV字を描く最短ルートで、
2√3=3.4641……
これより短いわけがない。
∴約3.5415
0523132人目の素数さん垢版2022/06/23(木) 10:02:45.56ID:YWNtBQte
積分定数は定義できるけど不定
0524132人目の素数さん垢版2022/06/23(木) 10:24:07.36ID:UYgInIKH
>>516
位相的場の理論がまさにそんな感じで
上手く定義し切れてないファインマン経路積分が自明になりたつ場所でだけの理論みたいな側面がある。
0525132人目の素数さん垢版2022/06/23(木) 12:57:10.19ID:M+nc8/Mt
>>501
X=√2sin(t+π/4) Y=1/2*sin(2t) -√2≦X≦√2 Y=(X^2-1)/2 

1+(dY/dX)^2=1+X^2=1+(sinhθ)^2=(coshθ)^2
coshθ=dsinhθ/dθ=dX/dθ sinha=√2

sinhacosha=a+√2√{1+√2^2}=a+√6
e^a=sinha+cosha=√2+√{1+√2^2} a=log(√2+√3)

2∫[0,√2]√{1+(dY/dX)^2}dX=2∫[0,a](coshθ)^2dθ
=2∫[0,a]{1+cosh(2θ)}/2dθ=∫[0,a]{θ+sinh(2θ)/2}'dθ
=a+sinh(2a)/2=a+sinhacosha=log(√2+√3)+√6
0526132人目の素数さん垢版2022/06/23(木) 13:22:28.11ID:M+nc8/Mt
間違えた
✕sinhacosha=a+√2√{1+√2^2}=a+√6
○sinhacosha=√2√{1+√2^2}=√6
0527イナ ◆/7jUdUKiSM 垢版2022/06/23(木) 17:05:29.25ID:nUDYPKK5
>>522訂正。
>>501
2∫[x=0→√2]√(1+x^2)dx
=2[x√(1+x^2)](x=√2)-2∫[x=0→√2]{x(1/2)2x/√(1+x^2)}dx
=2√6-2∫[x=0→√2]{x^2/√(1+x^2)}dx
=2√6-2∫[x=0→√2]√(1+x^2)+2∫[x=0→√2]{1/√(1+x^2)}dx
4∫[x=0→√2]√(1+x^2)dx=2√6+2∫[x=0→√2]{1/√(1+x^2)}dx
2∫[x=0→√2]√(1+x^2)dx=√6+∫[x=0→√2]{1/√(1+x^2)}dx
ここまで部分積分でやってきて今さら置換積分したくない状況。
じつはlogもいや。Arcsinとかも避けたい。
別解で2√(2+2√2)-√(√2-1)=3.75114220097……
値的に2√3=3.4641……よりやや大きいはずだから、わるくない。
0529132人目の素数さん垢版2022/06/23(木) 19:25:02.16ID:LhXCocHd
n,n+2,n^2+2がすべて素数となるような正整数nは有限個しか存在しないことを示せ。
0531イナ ◆/7jUdUKiSM 垢版2022/06/23(木) 19:52:52.98ID:B3VDlF+0
>>528
>>501
2∫[x=0→√2]√(1+x^2)dx
=2[x√(1+x^2)](x=√2)-2∫[x=0→√2]{x(1/2)2x/√(1+x^2)}dx
=2√6-2∫[x=0→√2]{x^2/√(1+x^2)}dx
=2√6-2∫[x=0→√2]√(1+x^2)+2∫[x=0→√2]{1/√(1+x^2)}dx
4∫[x=0→√2]√(1+x^2)dx=2√6+2∫[x=0→√2]{1/√(1+x^2)}dx
2∫[x=0→√2]√(1+x^2)dx=√6+∫[x=0→√2]{1/√(1+x^2)}dx
=√6+[log{x+√(1+x^2)}](x=√2)
=√6+log(√2+√3)
=2.9472849549……
0532132人目の素数さん垢版2022/06/23(木) 20:13:42.18ID:YWNtBQte
>>514
w=ω
x^n+y^n+nxy-1=(x+wy-w^2)f(x,y)
(w^2-wy)^n+y^n+n(w^2-wy)y-1=0
n=1 (w^2-1)+(w^2-w+1)y-wy^2≠0
n=2 w^4-2w^3y+w^2y^2+y^2+2w^2y-2wy^2-1=(w^2-2w+1)y^2+(2w^2-2w^3)y+(w^4-1)≠0
n=3 w^6-3w^5y+3w^4y^2-w^3y^3+y^3+3(w^2y-wy^2)-1=1-3w^2y+3wy^2-y^3+y^3+3w^2y-3wy^2-1=0 OK
n>3 (w^2-wy)^n+y^n+n(w^2-wy)y-1=(1+(-w)^n)y^n-nw^2w^(n-1)y^(n-1)+…=0
1+(-w)^n=0, nw^(n+1)=0 NG
n=3
x^3+y^3+3xy-1=(x+wy-w^2)(x+w^2y-w)(x+y-1)
0534132人目の素数さん垢版2022/06/23(木) 21:03:35.53ID:UOLj0FgV
ちなみに普通に「x+ωy-ω²がxⁿ+yⁿ+nxy-1の因子」と解釈するなら
xⁿ+yⁿ+nxy-1がx=y=-1で0になる事が必要
∴2(-1)ⁿ+n-1 = 0が必要
∴n=1-2(-1)ⁿが必要
∴n=3が必要
でいいんだけどな
0535132人目の素数さん垢版2022/06/23(木) 21:25:21.42ID:M+nc8/Mt
5以上の素数を6で割った余りは±1
n,n+2,n^2+2のどれもが5以上の素数と仮定するとこれらはどれも6で割った余りが±1
n,n+2の差が2である関係を見てn≡-1(mod6)
ところがn^2+2≡(-1)^2+2=3(mod6)だから矛盾

これらが素数であるならばnは5未満のときで該当するのはn=3のみ
0536132人目の素数さん垢版2022/06/23(木) 21:40:58.56ID:sseQLau6
cosA:cosB:cosC=1:2:3
を満たす三角形ABCは存在しまうか?

また、cosA:cosB:cosC=-1:2:3
を満たす三角形ABCは存在しますか
0537イナ ◆/7jUdUKiSM 垢版2022/06/23(木) 22:59:38.83ID:a4829VqK
>>531
それらしい値が出て安堵したんだけど、
y=(1/2)x^2-1/2のグラフの-√2≦x≦√2の長さは、
3点(-√2,1/2),(0,-1/2),(√2,1/2)を結ぶ折れ線の長さ、
2√3>3.464より長いはずです。
なぜですか?
0538132人目の素数さん垢版2022/06/24(金) 04:49:30.72ID:Qalu2pSA
>>531
1/√(1+x^2)の不定積分どうやって求めたの?
0539イナ ◆/7jUdUKiSM 垢版2022/06/24(金) 11:05:11.88ID:cb+zByZs
>>531
>>538
「放物線の長さ」でググって一個目の動画、
https://youtu.be/gnsLO9BSsow
黒ジャージの先生が板書してくれる。
一個目の式は∫[x=a→b]√{1+f'(x)^2}dx
a=0,b=√2が今回の問題。
二個目の式が前回の動画で説明されてるらしいんだけど、
それは見てない。結果に代入しただけ。
log{x+紙に書かないとむり。
0540イナ ◆/7jUdUKiSM 垢版2022/06/24(金) 11:53:27.51ID:cb+zByZs
>>539
なぜ放物線の長さが2√3=3.464……より長くならないのですか?
logの式は正しいの? 使い方間違ってんのかな?
放物線y=(1/2)x^2-1/2は、
1:√2:√3の直角三角形の中を通ってる。
斜辺√3+√3より長くならないとおかしい。
0541132人目の素数さん垢版2022/06/24(金) 12:01:13.86ID:Qalu2pSA
√2+√3>1.4+1.7=3.1>2.8*21/19=2.8(1+1/20)/(1-1/20)
log(√2+√3)>log(e(1+1/20)/(1-1/20))=1+2*1/20=1.1
√6+log(√2+√3)>2.4+1.1=3.5
0542132人目の素数さん垢版2022/06/24(金) 13:33:26.83ID:Qalu2pSA
>>540
x=sinht 1/√(1+x^2)=1/cosht=dt/dx ∫dx/√(1+x^2)=∫dt=t+C
2x=e^t-e^-t e^(2t)-2xe^t-1=0 e^t=x+√(x^2+1) t=log(x+√(x^2+1))
この対数はtを解いたものなので自然対数
しかしお前は常用対数を使っている
0543イナ ◆/7jUdUKiSM 垢版2022/06/24(金) 13:44:57.83ID:HToVU3bZ
>>540訂正。常用対数を使うGoogleがどうかしてる。
>>501
>>501
2∫[x=0→√2]√(1+x^2)dx
=2[x√(1+x^2)](x=√2)-2∫[x=0→√2]{x(1/2)2x/√(1+x^2)}dx
=2√6-2∫[x=0→√2]{x^2/√(1+x^2)}dx
=2√6-2∫[x=0→√2]√(1+x^2)+2∫[x=0→√2]{1/√(1+x^2)}dx
4∫[x=0→√2]√(1+x^2)dx=2√6+2∫[x=0→√2]{1/√(1+x^2)}dx
2∫[x=0→√2]√(1+x^2)dx=√6+∫[x=0→√2]{1/√(1+x^2)}dx
=√6+[log{x+√(1+x^2)}](x=√2)
=√6+log(√2+√3)
=√6+log(√2+√3)/log(e)
=3.59570557756……
ほらやっぱり。
3.464より弛みのぶん大きくなる。
0544132人目の素数さん垢版2022/06/25(土) 07:29:47.16ID:CLwM26ct
グーグル「お前の利用用途なんぞ知らんがな」
0545132人目の素数さん垢版2022/06/25(土) 09:16:00.98ID:r1CFlm+s
恒等式の係数決定において、数値代入法で解く際に最大次数より多くの数を代入して求めると、逆の確認をせずとも必要十分条件となるらしいのですが何故ですか??
0546132人目の素数さん垢版2022/06/25(土) 09:45:59.42ID:wJhpMrLc
>>545
連立方程式解く時
変数の数だけ式があればいいからだろ
0548132人目の素数さん垢版2022/06/25(土) 15:09:44.25ID:NO97kF0D
>>546
それが理由ではないだろ
実数解限定なら式1つでもいい場合があるしそもそも見た目の式の数は関係ない
0549132人目の素数さん垢版2022/06/25(土) 17:42:23.77ID:0rMO+MeK
>>548
それ言ったら行列式も一緒だろ
0551132人目の素数さん垢版2022/06/25(土) 18:24:31.33ID:HKHbvApV
>>550
それ事実上2式やん
0552132人目の素数さん垢版2022/06/25(土) 18:25:32.25ID:HKHbvApV
行列式でいったら分母が0な
0553132人目の素数さん垢版2022/06/25(土) 18:27:35.85ID:HKHbvApV
式を線形で掛け合わせて複合しても式の数は変わらんよ

そんなの当然の、話で、縮退しない式使わなきゃ
0554132人目の素数さん垢版2022/06/25(土) 18:32:34.46ID:HKHbvApV
2元1次の連立方程式では
直線が並行になって、解なしか
重なって直線の全てが解になる状態な
0555132人目の素数さん垢版2022/06/25(土) 18:35:49.10ID:HKHbvApV
>>550
で行列式も一緒だろ
0557132人目の素数さん垢版2022/06/25(土) 18:45:51.48ID:T0WmdezA
行列式の分母ってなに?
0559132人目の素数さん垢版2022/06/25(土) 18:53:00.40ID:HKHbvApV
揚げ足取りいいから
本質の話してくれよ
0560132人目の素数さん垢版2022/06/25(土) 18:55:57.88ID:CYNsVvOT
行列式の分母が重要らしいから、まさに本質なんでしょ
0561132人目の素数さん垢版2022/06/25(土) 19:03:37.57ID:HKHbvApV
内積ゼロとでも言い直せばいいかい
0562132人目の素数さん垢版2022/06/25(土) 19:04:53.55ID:HKHbvApV
なんでこう数学やってる奴は性格悪いの多いのかね?
目つき見てすぐわかるけど ww
0563132人目の素数さん垢版2022/06/25(土) 19:06:05.38ID:JoxY/lLK
わざわざここで自己紹介しなくてもいいよ
0564132人目の素数さん垢版2022/06/25(土) 19:10:20.11ID:HKHbvApV
確かに
大学行った時、俺より馬鹿な奴が数学科行ってたけど
自尊心が肥大してて大笑いした思い出
0565132人目の素数さん垢版2022/06/25(土) 19:13:31.97ID:FLjs/jKz
行列式の分母みたいな本質の話をするんじゃなかったの?
0566132人目の素数さん垢版2022/06/25(土) 19:15:14.33ID:HKHbvApV
>>565
内積ゼロでない話をすんだろ

本質は
0567132人目の素数さん垢版2022/06/25(土) 19:16:26.25ID:HKHbvApV
だから陰湿なんだよ
数学科の人間は
0568132人目の素数さん垢版2022/06/25(土) 19:17:59.75ID:HKHbvApV
まず、ほとんどがロンパリだからな

精神行ってる奴らの集まりだよ
0569132人目の素数さん垢版2022/06/25(土) 19:18:59.46ID:HKHbvApV
卒業アルバム見ると、あー、てなるの
数学科
0570132人目の素数さん垢版2022/06/25(土) 21:21:46.31ID:sNaC2Fwt
分母の行列式でも意味不明だな
クラメルの公式限定の話だろ
そもそも実質的な式の数は係数行列のランクを調べないと分からないだろ
0572132人目の素数さん垢版2022/06/25(土) 22:40:57.80ID:fh4QAtf2
(1,0),(0,1),(-1,0),(0,-1)を4頂点とする正方形の周及び内部の領域をDとする。
Dの点P(a,b)に対し、点QをQ(a+b,ab)で定める。

(1)PがD全体を動くとき、Qが動いてできる領域Eの面積を求めよ。

(2)E上の点(x,y)で、xy-yを最小にするものを求めよ。
0573イナ ◆/7jUdUKiSM 垢版2022/06/26(日) 01:34:57.51ID:ECW7PHAC
>>543
>>568
目に正面からパンチ受けると眼筋麻痺が起こるんだよ。
滑車神経麻痺ともいう。
少なからず数学好きの影響はあると思うけど。
なるときは急になるからね。
車を運転中に路面がせりあがってきて自力で戻せなくなる。
あんときはいよいよやばいと思ったぜ。
0574132人目の素数さん垢版2022/06/26(日) 03:42:13.21ID:Uu8SH82f
>>570
結局、内積がゼロにならない次数までだろ

結局そこまでは縮退させなきゃならんから
0575イナ ◆/7jUdUKiSM 垢版2022/06/26(日) 05:45:43.72ID:ECW7PHAC
>>573
>>572
(1)4(2)元の点が(-√2/2,-√2/2)のとき最小だから、
(-√2/2-√2/2-1)(-√2/2)(-√2/2)=-√2/2-1/2
0576イナ ◆/7jUdUKiSM 垢版2022/06/26(日) 05:58:50.51ID:ECW7PHAC
>>575訂正。
>>572
(1)4
(2)元の点(a,b)が(-√2/2,-√2/2)のとき最小だから、
最小値xy-y=(-√2/2-√2/2-1)(-√2/2)(-√2/2)=-√2/2-1/2
∴(a+b,ab)=(-√2,1/2)
0577132人目の素数さん垢版2022/06/26(日) 06:45:22.92ID:VURqE6zG
>>570
SET Aがいい例だけど
連立方程式の解とか逆行列を求めるのに
クラメルの公式とか余因子行列を行列式で割った式しか
方法知らない奴って理屈抜きの暗記だけで
試験誤魔化してきた記憶🐴🦌だよな

ガウス消去法で階段行列を作って
ランク求めるなんてむしろ簡単だけど
計算苦手な奴なのかな?
0578132人目の素数さん垢版2022/06/26(日) 06:58:09.29ID:r5bJRrKo
>>577
お前馬鹿じゃないの

それ理屈に合わせて機械的にやる方法なだけだろ
簡単簡単じゃないで判断する話じゃないぜ
0579132人目の素数さん垢版2022/06/26(日) 07:29:46.08ID:r5bJRrKo
基本はデターミナントがゼロにならない最高次数な

階段行列なんかそれを求める道具の一部でしかない
0580132人目の素数さん垢版2022/06/26(日) 07:42:49.98ID:IrpVi3+J
本質であるらしい行列式の分母って結局なんのこと?
0581132人目の素数さん垢版2022/06/26(日) 07:45:53.86ID:r5bJRrKo
>>580
こだわるなー
連立方程式の解の公式で
行列式が分母に表れるだけの話に、なにそんなに拘ってんの

数学科の馬鹿は話になんねーな
0582132人目の素数さん垢版2022/06/26(日) 07:48:47.52ID:r5bJRrKo
>>580
拘るんだったら
>>577みたいな道具を本質と間違える奴に突っ込めよ

それとも>>577がまともに見える馬鹿ですか、あなたは
0583132人目の素数さん垢版2022/06/26(日) 07:55:38.41ID:H0xgWMmW
数学科がバカとかキチガイとかいう話じゃなくて、

    行 列 式 の 分 母

という本質の話をするんじゃなかったの?
0584132人目の素数さん垢版2022/06/26(日) 07:57:05.52ID:VURqE6zG
>>578-579
階段行列のランクも、0でない小行列式のサイズも
理屈、つまり「本質」、とは異なる
本質な線形独立な元の最大個数
その表し方の違いでしかない

これ、大学1年の常識
0585132人目の素数さん垢版2022/06/26(日) 07:59:42.32ID:4GBckdBK
やっぱり行列式の分母が本質なんだ
0586132人目の素数さん垢版2022/06/26(日) 08:01:52.86ID:r5bJRrKo
>>584
そうだね
0587132人目の素数さん垢版2022/06/26(日) 08:02:39.42ID:r5bJRrKo
でも階段行列のランクって表現もおかしいよね
0588132人目の素数さん垢版2022/06/26(日) 08:03:41.58ID:r5bJRrKo
基本は素な元の個数だよね
0591132人目の素数さん垢版2022/06/26(日) 08:12:33.95ID:r5bJRrKo
>>590
基本的には独立したな
数学的には、難しいな
3次元的には直交しているって表現かな
0592132人目の素数さん垢版2022/06/26(日) 08:14:18.84ID:r5bJRrKo
まあ、並行してなきゃ構わんのだけどね
0593132人目の素数さん垢版2022/06/26(日) 08:24:13.50ID:VURqE6zG
線形代数は現代数学の理論の典型

線形空間という場の設定
線形写像という(場を保つ)写像の設定
線形独立という重要な性質の設定
そして線形独立性を判定し表現する方法である
消去法と行列式

ただ方法だけを覚え、2つの方法の同値性を
わけもわからず鵜呑みするのは
論理が分からん🐵の所業な
0594132人目の素数さん垢版2022/06/26(日) 08:25:16.95ID:r5bJRrKo
で結局
>>546 って話になんのよ
0595132人目の素数さん垢版2022/06/26(日) 08:27:09.38ID:r5bJRrKo
>>593
それ、意味も分からずランクの言葉を持ち出した奴らに言ってくれ
0596132人目の素数さん垢版2022/06/26(日) 08:31:27.86ID:r5bJRrKo
ちなみにランクって高校数学で習うんかい
0597132人目の素数さん垢版2022/06/26(日) 08:31:49.91ID:VURqE6zG
>>591
別に難しくはない
線形空間のn個の元をどう線形結合しても
0にならない場合線形独立という
目で見ようとしないこと
出来もせんことやろうとすると発狂すんで
0598132人目の素数さん垢版2022/06/26(日) 08:34:38.02ID:r5bJRrKo
>>597
それn次元の直交の再定義な

簡単ではないぜ、理解するには
0599132人目の素数さん垢版2022/06/26(日) 08:36:08.91ID:r5bJRrKo
高校数学の範囲でないし
0600132人目の素数さん垢版2022/06/26(日) 08:41:19.53ID:VURqE6zG
線形代数の現代的な特性が見過ごされるのは
線形空間の基底がn個のとき
n次元数ベクトル空間と同値になるから

つまり数ベクトル空間という
具体的な場の扱いに習熟すれば
理屈が全く分からんでも誤魔化せる
「はじき問題」は小学校だけではない
0601132人目の素数さん垢版2022/06/26(日) 08:43:57.68ID:r5bJRrKo
>>597
その話でokなら
デターミナントがゼロではない行列はって表現もokだろ
0602132人目の素数さん垢版2022/06/26(日) 08:45:15.23ID:r5bJRrKo
結局は直交したn次元で成り立つ、成り立たないって話に帰着するに過ぎない
0603132人目の素数さん垢版2022/06/26(日) 08:45:24.06ID:VURqE6zG
>>598
「直交」ではない
まあ、n個の基底があれば
それをいじくって
n個正規直交基底が作れるけどな
0604132人目の素数さん垢版2022/06/26(日) 08:46:32.27ID:r5bJRrKo
>>603
だから並行でなければいいって言ってるじゃん
0605132人目の素数さん垢版2022/06/26(日) 08:49:01.69ID:r5bJRrKo
>>603
だから理解してないでランクなんて言葉出した奴の方がおかしいだろ
0606132人目の素数さん垢版2022/06/26(日) 08:49:05.18ID:VURqE6zG
>>602
なんで「直交」が出てくるのか分からん
2つの元が線形独立だからといって
直交してるとは言えない
0607132人目の素数さん垢版2022/06/26(日) 08:51:07.19ID:r5bJRrKo
>>606
言い方悪いけど、直交した成分持たないと
線形独立にならないんじゃないの

元自体がお互いに直交する必要ないけど
0608132人目の素数さん垢版2022/06/26(日) 08:54:50.00ID:VURqE6zG
>>604
その言い方もダメな
3つの元の、どの2つも線形独立だからといって
3つの元が線形独立だとは言えない
3つの元が同一平面上あったら
線形独立にならない
これ初心者が落ちる穴の典型な
0609132人目の素数さん垢版2022/06/26(日) 08:56:18.91ID:r5bJRrKo
>>608
だからデターミナントで判断しろと
0610132人目の素数さん垢版2022/06/26(日) 08:57:02.98ID:r5bJRrKo
>>608
本質から外れてきてるぞ
0611132人目の素数さん垢版2022/06/26(日) 08:57:38.09ID:VURqE6zG
>>607
それは順序が逆
線形独立なら直交成分が抽出できる
0612132人目の素数さん垢版2022/06/26(日) 08:59:07.67ID:r5bJRrKo
>>611
???

言い負かそうと思ってムキになってない

同値だろそれ
0614132人目の素数さん垢版2022/06/26(日) 09:00:44.39ID:r5bJRrKo
最終的に人が判断するためには、直交だけの元にしないと理解できないとは思ってる
0615132人目の素数さん垢版2022/06/26(日) 09:01:15.45ID:r5bJRrKo
それこそ表現の世界だし
0616132人目の素数さん垢版2022/06/26(日) 09:02:28.80ID:r5bJRrKo
で結局
>>588
0618132人目の素数さん垢版2022/06/26(日) 09:07:59.63ID:r5bJRrKo
でさ
この話、高校数学なの?

キチガイが割り込んでくるからこうなるんじゃないの、特に数学科関連の奴ら
0619132人目の素数さん垢版2022/06/26(日) 09:09:27.20ID:VJMe5+Zj
>>596
行列自体無くなった
あったときも2次だけ
0621132人目の素数さん垢版2022/06/26(日) 09:23:22.41ID:r5bJRrKo
>>620
とりあえず
ここ、高校数学スレなんで、どっちが外れた話してるかは自明だと思うよ
0622132人目の素数さん垢版2022/06/26(日) 09:37:08.49ID:zkYB+DPB
高校数学スレで行列式の分母の話をする人がいるんですがそれは
0623132人目の素数さん垢版2022/06/26(日) 09:39:30.83ID:r5bJRrKo
>>622
俺が最初に持ち出した言葉じゃないぞ
0624132人目の素数さん垢版2022/06/26(日) 09:40:18.09ID:r5bJRrKo
数学科の人間が思わず出した言葉だろ
0625132人目の素数さん垢版2022/06/26(日) 09:45:44.22ID:r5bJRrKo
なんか粗探ししたいキチガイ属性持ちだな、数学科
0626132人目の素数さん垢版2022/06/26(日) 09:47:02.64ID:r5bJRrKo
俺、普段自分からは平易な言葉しか使わんから

高校生でも理解できるようにね
0628132人目の素数さん垢版2022/06/26(日) 09:52:25.65ID:r5bJRrKo
>>627
しつこいよ君
だから数学科頭おかしい、キチガイって定着すんだよ

控えな
0629132人目の素数さん垢版2022/06/26(日) 10:00:23.72ID:fpOMc4Z4
>>614
>人が判断するためには、
>直交だけの元にしないと
>理解できない
 何故?
0630132人目の素数さん垢版2022/06/26(日) 10:02:13.50ID:zkYB+DPB
行列式の分母おじさん
0631132人目の素数さん垢版2022/06/26(日) 10:02:53.91ID:fpOMc4Z4
>>615
誤 表現
正 趣味

ただ、その認識、間違ってます
0632132人目の素数さん垢版2022/06/26(日) 10:05:53.91ID:fpOMc4Z4
>>630
それ、公式を覚えるだけの高卒土方かもな
大卒に対する異常な敵愾心が特徴
0633132人目の素数さん垢版2022/06/26(日) 10:09:25.08ID:r5bJRrKo
>>632
それ俺のこと言ってる?
勘違いすんな、俺物理やねん
数学科があほであることを見切ってんだよ
0634132人目の素数さん垢版2022/06/26(日) 10:09:54.77ID:fpOMc4Z4
>>632
●違いになるのは簡単
精神病ではなく人格障害というもの
世の中の数%はコレ
ネトウヨも実際は自己愛性人格障害
政治思想でも何でもない
0635132人目の素数さん垢版2022/06/26(日) 10:11:27.01ID:fpOMc4Z4
>>633
>俺物理やねん
大学どこ?
0636132人目の素数さん垢版2022/06/26(日) 10:13:58.32ID:r5bJRrKo
>>635
私学最高峰
30年前な
0637132人目の素数さん垢版2022/06/26(日) 10:14:01.88ID:zkYB+DPB
物理って線形独立を知らない人でもできるの?
0638132人目の素数さん垢版2022/06/26(日) 10:17:08.55ID:fpOMc4Z4
>>636
ワセダ?
なんだ、オレと同じか
線形代数、誰に習った?
0639132人目の素数さん垢版2022/06/26(日) 10:17:55.03ID:r5bJRrKo
>>638
あん時、早稲田じゃない方が上だぜ
0640132人目の素数さん垢版2022/06/26(日) 10:18:58.84ID:fpOMc4Z4
>>637
どうだろ?分野によるんじゃない?
0642132人目の素数さん垢版2022/06/26(日) 10:21:18.83ID:fpOMc4Z4
>>639
>早稲田じゃない方
 神奈川県から出られない方?
 そりゃ選択ミスだな…
0643132人目の素数さん垢版2022/06/26(日) 10:22:14.42ID:r5bJRrKo
>>640
まーそうだね

線形代数は授業一回も出ずに試験突破

てか他の単位もほとんどそうだけど、リア充真っ盛りだったんで


線形代数の先生が、お袋の友達だったってのは笑ったけど
年賀状毎年やり取りしてた
0644132人目の素数さん垢版2022/06/26(日) 10:22:21.30ID:fpOMc4Z4
>>641
それは理科大
0647132人目の素数さん垢版2022/06/26(日) 10:26:06.96ID:r5bJRrKo
理科大馬鹿にしちゃいかんと思うよ

あそこ卒業すんのめちゃくちゃ大変だと思う

行かなくてよかったと思ってる
0648132人目の素数さん垢版2022/06/26(日) 10:27:41.79ID:QCzdIfI+
「直行」の概念は内積空間でないと定義できないが、
「線形独立」の概念は内積が定義されない空間でも定義可能。

内積空間において直行しているベクトルは線形独立なので、
直行は線形独立の一部分。しかし、この逆は必ずしも成り立たない。
すなわち、線形独立なら直行とは言えない。

「線形独立なら本質的に直行」

のような表現も不可能。
なんたって、内積が定義されない空間では本質的に直行が定義されないから。
0649132人目の素数さん垢版2022/06/26(日) 10:28:39.06ID:pHRrpXeD
>>632
行列は高校数学から消えて久しいけどな。

俺も物理学系だけど、行列式の由来がよくわからん。
いや、よくできてるとは思うけど。
どういう経緯で考え出されたもんなのか。
det(a_ij)=敗gn(σ)Πa_(iσ(i))とか初めて見たときは面食らったわw

そもそも日本語の用語が変じゃないか?
余因子って用語もあるんだから、determinant=決定因子でいいじゃん。
なんで行列「式」なんだよ?
0650132人目の素数さん垢版2022/06/26(日) 10:29:20.95ID:r5bJRrKo
>>648

だから最初に>>591 て言ってるでしょ
0651132人目の素数さん垢版2022/06/26(日) 10:31:05.66ID:pHRrpXeD
なんか文字化けしてる。 敗gn(σ)が敗gnに...
なんか敗けた気がするw
0652132人目の素数さん垢版2022/06/26(日) 10:31:21.62ID:r5bJRrKo
>>649
分からんよね、ただガウス記号のそのままの概念があてはまるのかな

それより3×3行列の外積がローレンツ力そのものにはビックラした
0653132人目の素数さん垢版2022/06/26(日) 10:34:54.91ID:QCzdIfI+
ここで暴れているバカは数学科ではなく物理学科のようだが、
物理学科でもノルム空間・バナッハ空間くらいは習う。

しかし、一般のノルム空間・バナッハ空間に直行性は定義できない。
それが定義されるためには内積が必要で、
与えられたノルムはその内積から誘導されている必要がある。
そのようなノルムに関して更にバナッハ空間になっているなら、
その空間はヒルベルト空間と呼ばれ、物理学徒が大好きな空間になる。

しかし、そうなってないノルム空間に対しては、直行性は全く定義されない。
たとえば、中線定理の成り立たないノルムは、決して内積から誘導されないので、
そのようなノルム空間には直行性は本質的に全く定義されない。
そして、そのような空間でも線形独立は定義される。

この時点で既に、直行と線形独立は同値な概念ではないと分かる。
もう体感レベルで「同値ではない」と分かる。

これを体感できてない時点で、例のバカタレは物理学徒としても落ちこぼれだと分かる。問題外。
0654132人目の素数さん垢版2022/06/26(日) 10:36:21.84ID:r5bJRrKo
でさ、何気に人の出身大学聞いてマウント取ろうとする態度ってどうなんだろうね

議論で負けたら権威で押し潰そうってか
最低の発想やね
0655132人目の素数さん垢版2022/06/26(日) 10:38:12.80ID:r5bJRrKo
>>653
それ高校生に聞かせて理解できるの?
0656132人目の素数さん垢版2022/06/26(日) 10:38:47.40ID:pHRrpXeD
>>651
どうも俺のブラウザ上だけの文字化けだったようだ。お騒がせしました。

余因子行列ってのも紛らわしいよなぁ。
転置余因子行列とかにしてくんないなかな。
余因子行列と言われたら、 (Δ_ij) だと思うのが普通の感覚だろうに。
0657132人目の素数さん垢版2022/06/26(日) 10:41:10.51ID:pHRrpXeD
>>653
「直行」って表現があんの?いや、直交のことだろうとは思うけど、繰り返し使ってるので
専門外の人間としては、そういう表現もあるのかな?と思って。
0658132人目の素数さん垢版2022/06/26(日) 10:43:34.10ID:QCzdIfI+
>>655
>>653は君に向けたレスなのであって、高校生に向けたレスではない。
従って、高校生が>>653を読んでどう思うかは何の関係もない。

君の今までの言動により、君は直行と線形独立の違いがあやふやになってることが判明している。
この時点で君はダメ。物理学徒としてもダメ。全くダメ。
0659132人目の素数さん垢版2022/06/26(日) 10:43:54.68ID:r5bJRrKo
>>657
高校数学ではあると思うけど
30年前の話な

n次元での話だと、再定義が必要になる理解、大学の線形数学の話ね
0660132人目の素数さん垢版2022/06/26(日) 10:46:23.97ID:r5bJRrKo
ごめん、漢字間違えてたな
交わる
0662132人目の素数さん垢版2022/06/26(日) 10:47:20.12ID:VJMe5+Zj
高校では直交使わないんだよな
何でかね
垂直しか使わないみたい
0663132人目の素数さん垢版2022/06/26(日) 10:48:26.62ID:VJMe5+Zj
直交だとほんとうに交わって突き抜けなくちゃ行けないのかな?
垂直なら突き抜けない場合も含めれる?
0664132人目の素数さん垢版2022/06/26(日) 10:49:40.48ID:r5bJRrKo
>>662
へーそうなんだ

昔どうだったかな、高校の授業や教科書覚えてないや
0665132人目の素数さん垢版2022/06/26(日) 10:50:26.10ID:r5bJRrKo
>>663
あんま、そこ関係無いから
0666132人目の素数さん垢版2022/06/26(日) 10:56:20.35ID:pHRrpXeD
まあ、たまにはこういう雑談展開もいいな。

出題爺さん以外からの質問がなかなか来ない時は特に。
0668132人目の素数さん垢版2022/06/26(日) 12:10:00.06ID:wsCA0FWs
>>646
あー、それも俺がケチョンケチョンにした奴だ

多分突っかかりかたからして同一人物だな
アホはアホのままなんだな
0669132人目の素数さん垢版2022/06/26(日) 12:34:28.87ID:wsCA0FWs
基本、大学数学での話持ち込んで解凍の説明する奴は無視した方がいいよ

後出題側で、高校数学を無視して出題する奴も
0670132人目の素数さん垢版2022/06/26(日) 12:35:44.59ID:wsCA0FWs
議論?会話?してみても、マウントとりたいだけみたいだから
0671132人目の素数さん垢版2022/06/26(日) 12:50:28.62ID:TzQu5Fwf
そもそも「出題」という概念がおかしい
頼むからスレタイ読んでくれって思う
0672132人目の素数さん垢版2022/06/26(日) 12:58:20.58ID:lgZ5FyOG
あれはわざとやってるから無視するのが一番
暴走族も無人島では走らない
0673132人目の素数さん垢版2022/06/26(日) 13:22:48.27ID:P6M0jH3I
nは正整数とする。
∫[0,t] Σ[k=0,n] a[k]x^k dx = Σ[k=0,n] ∫[0,t] a[k]x^k dx
を示せ。
0674132人目の素数さん垢版2022/06/26(日) 13:30:57.22ID:P6M0jH3I
4点(1,0),(0,1),(-1,0),(0,-1)を4頂点とする正方形の周および内部の領域をDとする。

(1)Dの点(x,y)で、以下の条件を満たすものが存在する領域Eを求めよ。
「条件」:-1≦2x+y≦1

(2)Eの周および内部の点(s,t)で、st-tを最大にするものを求めよ。
0675132人目の素数さん垢版2022/06/26(日) 13:33:11.34ID:P6M0jH3I
数列{a[n]},{b[n]}を以下のように定める。
a[1]=1,b[1]=2
a[n+1]=a[n]+b[n]
b[n+1]=-a[n]+2b[n]
このとき、a[k]とb[k]がともに3の倍数となるようなkをすべて決定せよ。
0676132人目の素数さん垢版2022/06/26(日) 13:35:09.70ID:wsCA0FWs
>>647
領域Eってなに?
なんか表現違くない?
0677132人目の素数さん垢版2022/06/26(日) 13:47:29.76ID:P6M0jH3I
(1)tを正の実数の定数とする。方程式
x^2-tx-1=0
は重解を持たないことを示し、その異なる2つの解のうち大きい方a[t]を求めよ。

(2)方程式
y^2+a[y]y+1=0
の解の個数を求め、その各々の解y[i]についてk/10≦y[i]<(k+1)/10を満たす整数kを求めよ。
0678132人目の素数さん垢版2022/06/26(日) 13:57:35.77ID:OflBeBD4
>>649
>行列式の由来がよくわからん。
>いや、よくできてるとは思うけど。
>どういう経緯で考え出されたもんなのか。
線形代数に関係するものではなかった筈
>det(a_ij)=敗gn(σ)Πa_(iσ(i))とか
>初めて見たときは面食らったわw
要するに交代多重線形形式だけどな
行列式が0でないと線形独立、というのは
それで示せる
0679イナ ◆/7jUdUKiSM 垢版2022/06/26(日) 15:08:18.25ID:ECW7PHAC
>>576
>>674
(1)-2x-1≦y≦-2x+1かつx-1≦y≦x+1で表される領域
(2)st-t=(s-1)t
s-1=-1,t=-1のときst-tは最大値(-1)^2=1
s=0,t=-1
∴(0,-1)
0681132人目の素数さん垢版2022/06/26(日) 17:58:54.59ID:wsCA0FWs
>>680
それ高校数学の解き方ちゃうやん
0682132人目の素数さん垢版2022/06/26(日) 20:05:47.46ID:evBJYu3C
a[1]=4, a[n+1]=(4a[n]+10)/(a[n]+1) (n=1,2,…)
で定まる数列{a[n]}がある。

b[n]=(a[n]+α)/(a[n]+β) が等比数列になるようなα, β(α<β)の組をひとつ求めよ

という問題がよくあり、一組求めればいいので、
 特性方程式x=(4x+10)/(x+1) の解を使えばうまくいく
という、十分性による解法が知られていますが、
もし「このようなα、βの組をすべて求めよ」だったら
どのように解けばいいでしょうか。
0683132人目の素数さん垢版2022/06/26(日) 20:14:03.75ID:Xww6GiDx
>>665
数学的にはそうでも
用語の高校生に持たせる印象を
考慮してるんじゃ無いかって推測
ジャ君はなぜ高校で直交使わないんだと思う?
0684132人目の素数さん垢版2022/06/26(日) 20:27:54.76ID:uz9M+l09
>>682
b[n]の漸化式は
[[1,α],[1,β]] [[4,10],[1,1]] [[1,α],[1,β]]⁻¹ = [[p,q],[r,s]]‥①
とおいた時に
b[n+1] = ( pb[n] + q )/( rb[n] + s )
になるのでこれが等比数列λⁿが解になるなら
λⁿ⁺¹ = ( pλⁿ+q )/( rλⁿ + s )
が全てのnについて成立する事からq=s=0、p/s = λがでる
戻って①に代入して
[[1,α],[1,β]] [[4,10],[1,1]] = [[p,0],[0,s]] [[1,α],[1,β]]
から
[1,α] [[4,10],[1,1]] = p [1, α]
[1,β] [[4,10],[1,1]] = s [1, β]
となる事がわかりそれから必要性がでる
0687132人目の素数さん垢版2022/06/26(日) 21:25:28.74ID:evBJYu3C
>>684
ありがとございます!
0689132人目の素数さん垢版2022/06/26(日) 22:35:19.33ID:pHRrpXeD
>>686
一般に、点Pの座標を(x,y,z)で表すけど、幾何ベクトルOP↓と数ベクトル(x,y,z)は一対一
対応してて、ベクトル演算も対応してることを利用する。
点HがAB↓とAC↓で張られる平面上にあることから、AH↓=sAB↓+tAC↓と書けるが、
点Cの座標はOC↓に対応しているので、最終的にOA↓、OB↓、OC↓ でOH↓を与えるベクトル方程式をたてる必要あり。なので、
OH↓=OA↓+AH↓=OA↓+sAB↓+tAC↓=OA↓+s(OB↓-OA↓)+t(OC↓-OA↓)
=(1-s-t)OA↓+sOB↓+tOC↓
あとはH,A,B,Cの座標となる数ベクトルをOH↓、OA↓...の代わりに用いてベクトル演算
0690132人目の素数さん垢版2022/06/26(日) 22:36:39.66ID:pHRrpXeD
×点Cの座標はOC↓に対応しているので、
○点Hの座標はOH↓に対応しているので、
0691 【凶】 垢版2022/06/27(月) 00:32:40.78ID:iVsMxqf0
>>679
>>686
平面ABCの法線ベクトル(1,2,-2)の単位ベクトルは、
1^2+2^2+(-2)^2=9だから、
(1/3)(1,2,-2)=(1/3,2/3,-2/3)
平面ABCの方程式はx+2y-2z=4
三角錐O-ABC=(OH/3)(BC/2){AC^2-(BC/2)^2}
(△OAB/3)OC=(OH/3)(√2)(√18)
(4/3)2=2OH
∴OH=4/3
Hの座標は、
→OH=(4/3)(1/3,2/3,-2/3)
=(4/9,8/9,-8/9)
0692132人目の素数さん垢版2022/06/27(月) 02:31:50.88ID:yuKYZYBZ
>>691

>>691
ありがとうございます。助かりました。遅くなってすみません
0693132人目の素数さん垢版2022/06/27(月) 02:47:03.19ID:pSB+7MkK
円C:x^2+y^2=1の周および内部の領域をDとする。

(1)D内の各点(x,y)を点(x+y,xy)に移すとき、移った先の領域を図示せよ。

(2)D内の点のうち、原点からの距離がr である点の集合をEとする。E上の各点(s,t)を点(s+t,st)に移すとき、移った先の曲線を(1)で図示した領域上に図示せよ。
0695132人目の素数さん垢版2022/06/27(月) 10:56:43.66ID:JECAQIXv
行列式の分母おじさんは、本質である

   行 列 式 の 分 母 

から話をそらしたがるのは何故だろうか
0696132人目の素数さん垢版2022/06/27(月) 11:40:24.17ID:eLN5FN+P
>>695
で、お前Fランなんだろww
人に出身大学聞いて自分が名乗らないの笑っちまうぜ
0697132人目の素数さん垢版2022/06/27(月) 12:07:19.52ID:D/ZjjHeD
行列式の分母おじさん、幻覚や幻聴まで発症した模様
0698132人目の素数さん垢版2022/06/27(月) 12:09:21.91ID:eLN5FN+P
発狂?茶化してFラン誤魔化してるだけだろ
0699132人目の素数さん垢版2022/06/27(月) 12:15:38.61ID:UtwRHIUM
で、本質である

   行 列 式 の 分 母 

の話は?好きなんでしょ、本質
0700132人目の素数さん垢版2022/06/27(月) 12:23:08.90ID:fqIOUR65
高2です
夏休みに行列式の分母を先取り学習したいのですが書籍を紹介して下さい
0701132人目の素数さん垢版2022/06/27(月) 12:25:45.83ID:RNwtQCb+
数学板の人は分からないから、物理板で聞くといいんじゃないかな
0702132人目の素数さん垢版2022/06/27(月) 13:40:24.13ID:u1qPzVM5
>>699
本質分からずにランク出したお馬鹿さんww
0703132人目の素数さん垢版2022/06/27(月) 13:44:01.17ID:0oZJfkuq
でFランなのか、どうなのか

言いたくないのか?
恥ずかしくてしゃべれないのか
0704132人目の素数さん垢版2022/06/27(月) 13:49:49.54ID:0oZJfkuq
ちなみに、分母分母騒いでるけど、全然気にしてへんから

25のときに、二次関数の解の公式忘れてたからな、自分

その時は、式変形して解の公式再度導いたけど、愕然としたわ


言葉とか知識とかそんなもんよ
本質は忘れないけどな

本質もわからないランク君
0705132人目の素数さん垢版2022/06/27(月) 13:56:36.89ID:BT3Yo3t6
あんなに大好きなはずの本質なのに

   行 列 式 の 分 母 

を「気にしない」と気にかけるとは、おかしな話ですね
ツンデレってやつなのでしょか?
0709132人目の素数さん垢版2022/06/27(月) 23:12:26.51ID:UMagzDQ4
>>700
大学に入ってから勉強したらいいよ。
その頃には、日本語の曖昧さが了解できるほど成長してるだろうから、
「行列式の分母」がなんたるかも理解できるはずw
0710132人目の素数さん垢版2022/06/28(火) 08:43:14.87ID:DKpgIYr9
大学生になったら「行列式の分母」の何たるかを理解できるんですか?
0711132人目の素数さん垢版2022/06/28(火) 09:41:36.12ID:OjAiUYLT
「行列式の分母」というのは行列式が分数になった時のその分母を指す。

つまりdetA=p/qという形になった時のqのことである。それ以外の意味は無い(笑)
0712132人目の素数さん垢版2022/06/28(火) 09:47:31.82ID:WOKchDYO
(笑)って久しぶりに見た気がする
0713132人目の素数さん垢版2022/06/28(火) 10:03:31.95ID:OjAiUYLT
行列式が0かそれ以外かの区別は極めて重要である。

n次正方行列Aについて考える。detA≠0の時, Aを正則行列と言って、逆行列A⁻¹を持つ。逆行列というのは逆数や逆写像と同じであってAによる線型変換f : Rⁿ→Rⁿは全単射になる。階数はrankA=n。

連立方程式はAx=bという形にまとめられて、A⁻¹が存在するので両辺に左から掛けてx=A⁻¹bという唯一つの解を持つことが分かる。

幾何学的に見ると一本の一次方程式はn-1次元超平面を現し、n本の式の連立はそれらの共通部分を指す。
n=2の時, 2本の直線の交点を表す。
n=3の時, 3枚の平面の交点を表す。一度に考えると難しいので順に考える。2枚の平面の共通部分は直線になる。その直線と3枚目の平面との共通部分は点になる。
n≧4の場合も同様。
0714132人目の素数さん垢版2022/06/28(火) 10:11:04.08ID:OjAiUYLT
平行、垂直、直交、合同、相似というのは幾何学的なイメージだけでは本質は捉えられない。整数の合同や行列の相似など。

数学ではそれぞれの場所で適切に定義されていくのでいつまで経っても「直交ってこういう意味ですよね?」とか間抜けなことを言ってると置いていかれる。それは本質を捉えられない幼稚な思考に過ぎない。

「行列式」のことを「行列式の分母」と言ってしまう鈍感さでは他人と数学について議論することは無理だろう。本人は良くても相手に迷惑だ。自分の間違いをその都度相手に「悟ってくれ」と頼むのは横暴である。
0715132人目の素数さん垢版2022/06/28(火) 10:22:03.48ID:OjAiUYLT
縮退というのは主として物理の言葉。物理の人間は縮退の辺りで大抵落ちこぼれるのでこの辺を突っつくと面白そう(笑)

縮退で落ちこぼれない奴は線型代数をちゃんとやってる奴。イメージだけで物理をやってると量子力学や相対論は無理(笑)
縮退について勝手なイメージを持ってそう。行列のrankをちゃんと勉強しないとな。
0716132人目の素数さん垢版2022/06/28(火) 11:05:29.70ID:vyxCtRpt
>>710
普通の大学生並みに君の知能が偏り無く発達すればね。

そうでない大人も多いことは、ここのスレを見るとよくわかるけど。
0717132人目の素数さん垢版2022/06/28(火) 11:14:54.35ID:vyxCtRpt
>>714
>「行列式」のことを「行列式の分母」と言ってしまう他人と数学について議論することは無理だろう。

それは一理あるが、行列式の分母というのが了解可能な言い間違いだということに気づかない鈍感さ、
というかそれを許さないアスペルガー的な柔軟性のなさもいかがなものか。
どっちもどっちとしかいいようがない。
0718132人目の素数さん垢版2022/06/28(火) 12:11:38.96ID:WOKchDYO
DDきた
0719132人目の素数さん垢版2022/06/28(火) 12:49:13.16ID:QlJ9Ouq9
>>425
完全に完全順列じゃん
0721132人目の素数さん垢版2022/06/28(火) 18:37:11.36ID:IWuNCVJY
>>717
言い間違いである時点でツッコミは入る。そのツッコミに対して
「すまん。言い間違えた。正しくはこうだ」と速攻で修正できるなら問題ない。
ところが、例のバカタレ物理学徒はいつまで経っても修正をしなかった。そこが問題。

>大学生になったら「行列式の分母」の何たるかを理解できるんですか?

のような皮肉が未だに書き込まれるのは、本人が未だに修正しないから。言い換えれば、

「アスペで柔軟性がないから、いつまでもそういう皮肉を書き込む」

のではなくて、

「本人がいつまで経っても修正しないバカタレだから、いつまでも皮肉を書き込む」

ということ。
0722132人目の素数さん垢版2022/06/28(火) 19:46:21.05ID:vyxCtRpt
>>721
その悪意のある質問には、俺も面白がって皮肉で応えるレスをつけて面白がってたわけだけど、
一言突っ込むくらいは問題ないが、簡単にわかるような言い間違いを執拗に攻撃し続けるのは
いかがなものか。

互いに意固地になってるだけだろ。
0723132人目の素数さん垢版2022/06/28(火) 19:47:24.54ID:Cy7lGJzK
行列式の分母おじさんって、昔いたオカリーと芸風がすごく被る
オカリーを知ってる人はもう少ないだろうけど
0725132人目の素数さん垢版2022/06/28(火) 19:54:07.28ID:s8e7PCOJ
実数a,b,c,dはad-bc=0を満たす。
xy平面上の点(x,y)を
(x',y')=(ax+by,cx+dy)
により(x',y')に移す。

このとき点(x',y')を
(x,y)=(sx'+ty',ux'+vy')
により点(x,y)に移すような実数s,t,u,vは存在するか。
0726132人目の素数さん垢版2022/06/28(火) 20:01:56.61ID:IWuNCVJY
>>724
例のバカタレ物理学徒君は最初からケンカ腰なんだよね。
自分の言い間違いを一言でいいから謝罪することもせずに、

>数学科の馬鹿は話になんねーな

これだからね。そんなゴミクズ、いつまでも皮肉が止まらなくたって自業自得。
0729132人目の素数さん垢版2022/06/29(水) 13:01:05.40ID:bJeDuWFx
心優しくて親切な>>36が叩かれた理由が未だにさっぱりわからない
0731132人目の素数さん垢版2022/06/29(水) 13:36:58.59ID:NnNPYaSm
nを自然数、xを実数、sを複素数、iを虚数単位とする。
-1≦sinx≦1となるとき、sが満たすべき条件をn,x,iで表せ。
0732132人目の素数さん垢版2022/06/29(水) 14:04:36.36ID:wftJYkeh
Fランうぜーよ
お前も出身大学開陳してみろよww
0733132人目の素数さん垢版2022/06/29(水) 14:18:41.43ID:AzG8/65L
もしかして:開示
0734132人目の素数さん垢版2022/06/29(水) 15:04:22.23ID:fQGoSMOx
Aさんが「開陳」という言葉を使ったとき、Aさんが使い方を誤っている確率はpであるという。
またBさんが「開示」という言葉を使ったとき、Bさんが使い方を誤っている確率はqであるという。

この一年間、Aさんは「開陳」という言葉を100回使い、Bさんは「開示」という言葉を100回使った。
そのうち、Aさんが「開陳」の使い方をちょうど[100p]回誤り、かつBさんが「開示」の使い方をちょうど[100q]回誤った確率をp,qで表せ。
0735132人目の素数さん垢版2022/06/29(水) 15:48:30.09ID:GUVW0YX5
晒せの意味含めてるから、全然おかしくないよ
0736132人目の素数さん垢版2022/06/29(水) 15:50:15.66ID:GUVW0YX5
やっぱりFランなんだ
0738132人目の素数さん垢版2022/06/29(水) 18:09:28.65ID:fQGoSMOx
>>737
3^m+5^m=2^n
を満たす正整数の組(m,n)をすべて求めたいのですが、どのように解いたら良いでしょうか?
0741132人目の素数さん垢版2022/06/29(水) 22:25:54.01ID:VJVMtkme
1/(2^n+3^n) の、n=1から∞までの和の値は求まれますか?
0743132人目の素数さん垢版2022/06/29(水) 23:08:09.62ID:64Xwjj0f
>>741
宇宙際の人なら・・・
0744132人目の素数さん垢版2022/06/29(水) 23:40:50.22ID:VJVMtkme
こんな簡単そうな和でも求まらないものなんですね
0746132人目の素数さん垢版2022/06/30(木) 06:49:53.74ID:rBlCk8TQ
11種類の銘柄のワインを一口試飲して銘柄をあてたらそのワインがもらえる。
11種類の銘柄は公表されている。同じ銘柄を複数回答えてはならない。
無作為に銘柄を答えたときに何種類の銘柄のワインをもらえる確率が最も高いか?
0747132人目の素数さん垢版2022/06/30(木) 07:03:39.58ID:8Swi0Rjf
>>745
求まらない
求まります
求まる
求まるとき
求まれば
求まれ
求まって
求まった
0749132人目の素数さん垢版2022/06/30(木) 11:20:48.40ID:ZWeBcFgq
求まりしく
求まりしから
求まりしく
求まりしかり
求まりし
求まりしき
求まりしかる
求まりしけれ
求まりしかれ
0750132人目の素数さん垢版2022/06/30(木) 13:36:04.67ID:wP5VEjtw
>>746
n種類貰える時の確率は
P(n)=nCr(11,n)*納
0751132人目の素数さん垢版2022/06/30(木) 13:56:21.57ID:wP5VEjtw
>>746
P(n)=nCr(11,n)*{シグマ[k:0,11-n](-1)^k(11-n)!/k!}/11!
P(n)=シグマ[k:0,11-n](-1)^k/(k!n!)
よって一本貰える時が1番確率が高い
0755132人目の素数さん垢版2022/06/30(木) 18:54:13.32ID:C8kwiF3d
下2桁が隠れている5桁の整数962⬜︎⬜︎は63の倍数である。下2桁を求めよ

解答(途中まで)
下2桁の2つの数をa,bとする
962abが9の倍数であるから
a + b = 1 か10
96200 = 7・13742 + 6であるから 
10a + b は7で割って1余る数となる ←ここが理解出来ないです。お願いします。
0757132人目の素数さん垢版2022/06/30(木) 20:01:30.52ID:2a2zKHd5
>>755
63 = 7×9 だからじゃね?

96200 + 10a + b = 7*13742 + 6 + 10a + b が 63で割り切れれば当然、7でも割り切れる。
0759132人目の素数さん垢版2022/06/30(木) 21:39:45.73ID:kwk8gMcB
人数1000人以下の集団でアンケートを行う。
項目Aに該当する人の割合が、小数第2位を四捨五入したパーセントで与えられたとき、
項目Aに該当する人数はかならず確定しますか?

たとえば822人の集団で、タバコを吸う人の割合が22.9パーセントと与えられたら
タバコを吸う人は188人に確定、というふうに。
0760132人目の素数さん垢版2022/06/30(木) 21:52:40.85ID:fkoqOWW3
1≦n≦1000である時、区間

n(2k-1)/2000 ≦ x < n(2k+1)/1000

の長さは

n/1000≦1

長さが1以下のの左閉、右開区間の整数の個数は常に1以下
0762132人目の素数さん垢版2022/07/01(金) 01:08:06.10ID:rGQwthsH
>>761
実数で考える。方程式は
y(1+x^2)=1 …@
x+1/x=y+1/y …A
と同値

(i) x≠y のとき
A=k (k<=-2,2<=k) とおく
z+1/z=k を解くと
z=(k-√(k^2-4))/2,(k+√(k^2-4))/2
だから @よりk=1 となる。
しかしこれはk<=-2,2<=kより不適

(ii) x=yのとき
@より
x(1+x^2)=1
これを解くと
x=((√93+9)/18)^(1/3)-((√93-9)/18)^(1/3)

以上より
(x,y)=(((√93+9)/18)^(1/3)-((√93-9)/18)^(1/3),((√93+9)/18)^(1/3)-((√93-9)/18)^(1/3))
0763132人目の素数さん垢版2022/07/01(金) 07:01:20.63ID:rxzXx8l9
【問題】
喫煙率の調査にアンケート用紙を1000枚配った。
喫煙者の割合が小数第2位を四捨五入して22.9%という報告があった。
回答数の報告がなかったのでそれを推定したい。
回答数として想定できる数は何個あるか?

ちなみに、
回答数が101であることはない。
なぜなら23/101=0.2277,24/101=0.2376なので四捨五入した割合が22.9%にはなりえないので。
0766132人目の素数さん垢版2022/07/01(金) 09:24:31.65ID:cbBmJZaW
>>751
一本貰える確率 14684571/39916800 = 536/1457 = 0.3678795
0本貰える確率  14684570/39916800 = 1001/2721 = 0.3678794
その差はわずか
0767イナ ◆/7jUdUKiSM 垢版2022/07/01(金) 09:40:54.54ID:9vV5sGvq
>>691
>>763
1000人中422人が回答してくれて、
喫煙者が97人いたとすると、
喫煙率は22.931……%
四捨五入して22.9%
回答数はいくらでもありそうな気がするけど。
0768132人目の素数さん垢版2022/07/01(金) 09:45:32.09ID:6wf5R5rJ
明らかに有限個しかないです
0769132人目の素数さん垢版2022/07/01(金) 09:48:29.08ID:0V+0pnnr
1,2,3,…,9 を並べ替えた順列をa[1],a[2],a[3],…,a[9]とするとき
|a[1]-1|+|a[2]-2|+|a[3]-3|+…+|a[9]-9| の和は常に偶数になりますか?
0770132人目の素数さん垢版2022/07/01(金) 09:50:28.80ID:Wyv6uh0F
>>767
たくさんあるけど、
例えば回答数500はありえないから
可能な回答数は何個あるかという問題。
0771132人目の素数さん垢版2022/07/01(金) 09:57:39.06ID:cbBmJZaW
>>769
順列を網羅してその式を2で割った剰余の和を計算させてみた
sum(apply(permuteGeneral(9),1,\(x) sum(abs(x-1:9))%%2))

> sum(apply(permuteGeneral(9),1,\(x) sum(abs(x-1:9))%%2))
[1] 0

全部偶数になるという結論になった。
0772132人目の素数さん垢版2022/07/01(金) 12:37:52.09ID:cbBmJZaW
>>767
計算間違ってない?
> 97/423
[1] 0.2293144
なので423人が回答はあるけど、


> 97/422
[1] 0.2298578
> 98/422
[1] 0.2322275
なので422人だと四捨五入して22.9%はならないので回答数422はありえない。
0773132人目の素数さん垢版2022/07/01(金) 13:23:48.98ID:rGQwthsH
>>769
絶対値を外せば、出てくる項はa[1],a[2],…,a[9],と1,2 …,9のみでそれらを足したり引いたりしたものになる。
足しても引いても偶奇は変わらないから、結局a[1]+1,a[2]=2,…,a[9]=9の時のみ考えれば良くて、答えは偶数
0774132人目の素数さん垢版2022/07/01(金) 14:08:41.21ID:rGQwthsH
>>763
回答数をn(1<=n<=1000、整数)、喫煙者の回答をa(1<=a<=n、整数)とおく。
このとき、条件から
22.85<=a/n*100<22.95
0.2285*n<=a<0.2295*n
457/2000*n<=a<459/2000*n
となる。つまり、これを満たすaが存在する時のnを求めればよい。
1<=n<=1000より、457/2000*nと459/2000*nは整数にならない。
したがって、aが存在するための条件は[459/2000*n]-[457/2000*n]>=1である。
プログラムで解くと
import math
a=[]
for n in range(1,1001):
if math.floor(459*n/2000)-math.floor(457*n/2000)>=1:
a.append(n)
print(a)
0775132人目の素数さん垢版2022/07/01(金) 14:22:08.80ID:rGQwthsH
>>774
35, 48, 70, 83, 96, 105, 109, 118, 131, 140, 144,
153, 157, 166, 170, 175, 179, 188, 192, 201, 205,
210, 214, 218, 223, 227, 231, 236, 240, 245, 249,
253, 258, 262, 266, 271, 275, 279, 280, 284, 288,
292, 293, 297, 301, 306, 310, 314, 315, 319, 323,
327, 328, 332, 336, 340, 341, 345, 349, 350, 353,
354, 358, 362, 363, 367, 371, 375, 376, 380, 384,
385, 388, 389, 393, 397, 398, 401, 402, 406, 407,
410, 411, 414, 415, 419, 420, 423, 424, 428, 432,
433, 436, 437, 441, 442, 445, 446, 449, 450, 454,
455, 458, 459, 462, 463, 467, 468, 471, 472, 475,
476, 477, 480, 481, 484, 485, 489, 490, 493, 494,
497, 498, 502, 503, 506, 507, 510, 511, 512, 515,
516, 519, 520, 523, 524, 525, 528, 529, 532, 533,
536, 537, 538, 541, 542, 545, 546, 547, 550, 551,
554, 555, 558, 559, 560, 563, 564, 567, 568, 571,
572, 573, 576, 577, 580, 581, 582, 584, 585, 586,
589, 590, 593, 594, 595, 597, 598, 599, 602, 603,
606, 607, 608, 611, 612, 615, 616, 617, 619, 620,
621, 624, 625, 628, 629, 630, 632, 633, 634, 637,
638, 641, 642, 643, 645, 646, 647, 650, 651, 652,
654, 655, 656, 658, 659, 660, 663, 664, 665, 667,
668, 669, 672, 673, 676, 677, 678, 680, 681, 682,
0776132人目の素数さん垢版2022/07/01(金) 14:22:34.36ID:rGQwthsH
>>775
685, 686, 687, 689, 690, 691, 693, 694, 695, 698,
699, 700, 702, 703, 704, 706, 707, 708, 711, 712,
713, 715, 716, 717, 719, 720, 721, 722, 724, 725,
726, 728, 729, 730, 733, 734, 735, 737, 738, 739,
741, 742, 743, 746, 747, 748, 750, 751, 752, 754,
755, 756, 757, 759, 760, 761, 763, 764, 765, 767,
768, 769, 770, 772, 773, 774, 776, 777, 778, 780,
781, 782, 783, 785, 786, 787, 789, 790, 791, 792,
794, 795, 796, 798, 799, 800, 802, 803, 804, 805,
807, 808, 809, 811, 812, 813, 814, 815, 816, 817,
818, 820, 821, 822, 824, 825, 826, 827, 828, 829,
830, 831, 833, 834, 835, 837, 838, 839, 840, 841,
842, 843, 844, 846, 847, 848, 849, 850, 851, 852,
853, 855, 856, 857, 859, 860, 861, 862, 863, 864,
865, 866, 868, 869, 870, 872, 873, 874, 875, 876,
877, 878, 879, 881, 882, 883, 884, 885, 886, 887,
888, 889, 890, 891, 892, 894, 895, 896, 897, 898,
899, 900, 901, 902, 903, 904, 905, 907, 908, 909,
910, 911, 912, 913, 914, 916, 917, 918, 919, 920,
921, 922, 923, 924, 925, 926, 927, 929, 930, 931,
932, 933, 934, 935, 936, 937, 938, 939, 940, 942,
943, 944, 945, 946, 947, 948, 949, 950, 951, 952,
953, 954, 955, 956, 957, 958, 959, 960, 961, 962,
963, 964, 965, 966, 967, 968, 969, 970, 971, 972,
973, 974, 975, 977, 978, 979, 980, 981, 982, 983,
984, 985, 986, 987, 988, 989, 990, 991, 992, 993,
994, 995, 996, 997, 998, 999, 1000,
0777132人目の素数さん垢版2022/07/01(金) 18:01:35.07ID:cbBmJZaW
>>763
プログラム解

f=\(n,P=22.9){
p=P/100
lo=p-0.05/100
up=p+0.05/100
flg=\(x) n*lo <= x & x <= n*up
(1:n)[flg(1:n)]
}
re=sapply(1:1000,f)
(1:1000)[sapply(1:1000,\(x) length(re[[x]]))==1]

[1] 35 48 70 83 96 105 109 118 131 140 144 153 157 166
[15] 170 175 179 188 192 201 205 210 214 218 223 227 231 236
[29] 240 245 249 253 258 262 266 271 275 279 280 284 288 292
[43] 293 297 301 306 310 314 315 319 323 327 328 332 336 340
...
...
[477] 979 980 981 982 983 984 985 986 987 988 989 990 991 992
[491] 993 994 995 996 997 998 999 1000


で可能性のある回答数は498種類
0778132人目の素数さん垢版2022/07/01(金) 18:46:59.46ID:6wf5R5rJ
なんで出題スレ使わないの?
0780132人目の素数さん垢版2022/07/01(金) 21:06:28.50ID:eaZeDM2p
xy平面上の4点(1,0),(0,1),(-1,0),(0,-1)を頂点とする正方形の内部の点を、(x,y)→(x+y,xy)で表される変換により別の領域に移す。
移った先の領域を図示せよ。
0781132人目の素数さん垢版2022/07/01(金) 22:31:24.32ID:rGQwthsH
>>780
X=x+y,Y=xyとおく。Xを固定して考える。X=x+yよりy=-x+X 
これが与えられた正方形の内部と交点を持つためには
-1<X<1を満たす必要がある。
また、このときyをxの関数と考えると(X-1)/2<x<(X+1)/2をとる。
Y=xy=x*(-x+X)=-(x-X/2)^2+X^2/4
これは上に凸の二次関数であり、軸はx=X/2
したがって(X-1)/2<x<(X+1)/2から
X^2/4-1/4<Y<=X^2/4 となる。
以上より、求める領域は
-1<X<1、X^2/4-1/4<Y<=X^2/4
Xをx、Yをyに置き換えると
-1<x<1、x^2/4-1/4<y<=x^2/4
0782132人目の素数さん垢版2022/07/02(土) 08:40:07.09ID:NcDOfPP8
age
0783132人目の素数さん垢版2022/07/02(土) 09:17:42.84ID:dW9Z9byp
ある正整数nについて、n/7を小数点以下第3位で四捨五入すると14.28となった。
nとして考えられるものをすべて求めよ。
0784132人目の素数さん垢版2022/07/02(土) 09:27:32.82ID:9gL5B5IR
14.275≦n/7<14.285
99.925≦n<99.995
整数解なし
0785132人目の素数さん垢版2022/07/02(土) 09:30:57.38ID:dW9Z9byp
領域D:「-1≦x≦1かつ(x^2/4)-(1/4)≦y≦x^2/4」
上の点P(s,t)に対し、点QをQ(s+t,st)で定める。
D内をPが動くとき、Qが動きうるすべての点からなる領域を図示せよ。
0786132人目の素数さん垢版2022/07/02(土) 10:52:21.23ID:RTa1Ki+1
>>781
>X=x+y,Y=xyとおく
(t-x)(t-y)=t^2-Xt+Y={t-(X+√(X^2-4Y))/2}{t-(X-√(X^2-4Y))/2}
t=x,y=(X±√(X^2-4Y))/2
x+y=X
x-y=±√(X^2-4Y)
-1≦x-y=±√(X^2-4Y)≦1
0≦X^2-4Y≦1
(X^2-1)/4≦Y≦X^2/4
-1≦x+y=X≦1
0787132人目の素数さん垢版2022/07/02(土) 11:33:46.36ID:i3+aann6
出題馬鹿との絡み合いにはウンザリだわ。死んでほしい。

まともな質問ないのかよ?
0788132人目の素数さん垢版2022/07/02(土) 11:40:47.71ID:RTa1Ki+1
>>785
面倒くさヤコビアンが0になるのがx=yなのでそこで分けて境界の像を考えてその内部の合併集合
0789132人目の素数さん垢版2022/07/02(土) 12:18:18.93ID:baQL4cMp
>>785
781と同じやり方で求める。
領域Dとt=-s+Xとの交わりを考えることで-1<=X<=5/4であることが分かる。
tをsの関数と考えるとxの取り得る値は
(i) -1<=X<=-3/4 のとき
-s+X=s^2/4-1/4 これを解くと
s=-2±√(4X+5)より
-1<=s<=-2+√(4X+5)
(ii) -3/4<X<=1 のとき
-s+X=s^2/4 これを解くと
s=-2±√(4X+4) より
-2+√(4X+4)<=s<=-2+√(4X+5)
(iii) 1<X<=5/4 のとき
-2+√(4X+4)<=s<=1
また、Y=st=s*(-s+X)=-(s-X/2)^2+X^2/4
これは上に凸の二次関数であり、軸はs=X/2
X/2=-2+√(4X+4)のときX=0
X/2=-2+√(4X+5)のときX=4-2√5
以上から
-1<=X<=-3/4のとき -X-1<=Y<=-9-6X+4√(4X+5)+X√(4X+5)
-3/4<x<=4-2√5のとき -8-6X+4√(4X+4)+X√(4X+4)<=Y<=-9-6X+4√(4X+5)+X√(4X+5)
4-2√5<x<=0 のとき min(-8-6X+4√(4X+4)+X√(4X+4),-9-6X+4√(4X+5)+X√(4X+5))<=Y<=X^2/4
0<X<=1のとき -9-6X+4√(4X+5)+X√(4X+5)<=Y<=-8-6X+4√(4X+4)+X√(4X+4)
1<X<=5/4のとき X-1<=Y<=-8-6X+4√(4X+4)+X√(4X+4)
0793132人目の素数さん垢版2022/07/02(土) 12:25:27.98ID:m+8rB9NZ
単位円内のすべての点(s,t)を(s+t,st)に移すことから始めて、以下点が移った領域内のすべての点(p,q)を(p+q,pq)に移して、…と繰り返すと、最終的にはどのような領域になりますか?
この操作である領域に収束するのでしょうか、それとも発散するのでしょうか。
ご教示ください、よろしくお願いします。
0796132人目の素数さん垢版2022/07/02(土) 13:47:25.61ID:fjXrCt8A
>>793
変換をFとし最初の領域をD0
Dn=F(Dn-1)と定義して
D=limDn=limF^n(D0)としたいと
君はFが原点中心の対称移動だったとき
出題のD0に対するDは(0,0)のみと考えるか
D0全体と考えるかどっちの?
0797イナ ◆/7jUdUKiSM 垢版2022/07/02(土) 15:01:24.86ID:h0Re7zyq
>>767訂正。
>>763
1000人中423人が回答してくれて、
喫煙者が97人いたとすると、
喫煙率は22.931……%
四捨五入して22.9%
0799132人目の素数さん垢版2022/07/02(土) 18:24:38.00ID:9gL5B5IR
p(n)=1という与えられた条件を使うため
0800132人目の素数さん垢版2022/07/02(土) 20:40:24.50ID:fZJ+VBwI
a[0]=s,b[0]=t
a[n+1]=a[n]+b[n]
b[n+1]=a[n]b[n]
によって定められる数列a[n]およびb[n]の一般項は求められますか?
0804132人目の素数さん垢版2022/07/03(日) 06:10:20.54ID:LbDCUOdw
【問題】
内閣支持率を調査した。
調査の対象となったのは3726人で55%から回答を得た。
「支持する」と答えた人は50%だった。
百分率はどちらも小数第1位を四捨五入しての値である。
内閣支持率として想定される分数を列挙せよ。
0805132人目の素数さん垢版2022/07/03(日) 06:26:37.80ID:LbDCUOdw
列挙はスペースをとるので改題

【問題】
内閣支持率を調査した。
調査の対象となったのは3726人で55%から回答を得た。
「支持する」と答えた人は50%だった。
百分率はどちらも小数第1位を四捨五入しての値である。
内閣支持率として想定される分数はいくつあるか?
最小と最大となる分数を述べよ。
0806132人目の素数さん垢版2022/07/03(日) 06:32:48.31ID:LbDCUOdw
>>803
検証して間違いがあればご指摘宜しく。
統計処理ソフトRで乱数発生させての作図ゆえ、
辺縁が毛羽立っていて美しくないので専用ソフトでの作図を希望。
0808132人目の素数さん垢版2022/07/03(日) 07:51:08.65ID:/dzQeLVj
とうとうプログラムおじさんまで出題するようになったな
0812132人目の素数さん垢版2022/07/03(日) 15:16:07.12ID:Ayw9HiXI
>>805
プログラムして計算させる(割り算と大小比較をする単純作業)
R (ver4.10)のコード
https://egg.5ch.net/test/read.cgi/hosp/1655094973/282
バグがあるかもしれんので別の言語での検証希望

> calc(3726,55,50)
761 cases
min : 1005/2030 = 0.4950739
max : 1025/2030 = 0.5049261
となった。


【練習問題】
内閣支持率を調査した。
調査の対象となったのは2022人で76.5%から回答を得た。
「支持する」と答えた人は回答者の43.2%だった。
百分率はどちらも小数第2位を四捨五入しての値である。
内閣支持率として想定される分数はいくつあるか?
最小と最大となる分数を述べよ。

> calc(2022,76.5,43.2,0.05)
5 cases
min : 667/1545 = 0.4317152
max : 669/1547 = 0.4323625
0815132人目の素数さん垢版2022/07/03(日) 17:26:29.40ID:xHHIYJPU
同じ奴が出題して答えている
たまに暇な奴が絡む

それの繰り返し

オナニーは余所でしてくれ
0816132人目の素数さん垢版2022/07/03(日) 17:52:52.94ID:/dzQeLVj
区間推定せよという問題なら分かるがこの問題に実用性があるとは思えん
0817132人目の素数さん垢版2022/07/03(日) 18:11:02.72ID:RFV59VHh
a[0]=s,b[0]=t
a[n+1]=a[n]+b[n]
b[n+1]=a[n]b[n]
によって定められる数列a[n]およびb[n]の一般項は求められますか?
0818132人目の素数さん垢版2022/07/03(日) 18:31:46.06ID:i7w4Bd52
>>812
>最小と最大となる分数を述べよ。
0/1と1/1
0819132人目の素数さん垢版2022/07/03(日) 19:31:33.03ID:Ayw9HiXI
これは数が少ないから列挙できるな。

【問題】
内閣支持率を調査した。
調査の対象となったのは2022人で76.5%から回答を得た。
「支持する」と答えた人は43.2%だった。
百分率はどちらも小数第2位を四捨五入しての値である。
内閣支持率として想定される分数をすべて列挙せよ。

667/1545 = 0.4317152
668/1545 = 0.4323625
668/1546 = 0.4320828
668/1547 = 0.4318035
669/1547 = 0.4324499
の5通り
0820132人目の素数さん垢版2022/07/03(日) 19:52:12.07ID:/dzQeLVj
出題したのに誰も答える人がいないので自分で答えてる人がいますね
0823132人目の素数さん垢版2022/07/03(日) 21:11:18.35ID:Ayw9HiXI
>>816
【実用問題】

内閣支持率を調査した。
調査の対象となったのは2022人で69.6%から回答を得た。
「支持する」と答えた人は回答者の33.3%だった。
百分率はどちらも小数第2位を四捨五入しての値である。

回答した全員に白饅頭を、支持すると回答した人には追加で紅饅頭を送る。
紅白の饅頭を各々何個巡視しておけばよいか?
0824132人目の素数さん垢版2022/07/03(日) 21:12:45.19ID:Ayw9HiXI
タイプミス修正

【実用問題】

内閣支持率を調査した。
調査の対象となったのは2022人で69.6%から回答を得た。
「支持する」と答えた人は回答者の33.3%だった。
百分率はどちらも小数第2位を四捨五入しての値である。

回答した全員に白饅頭を、支持すると回答した人には追加で紅饅頭を送る。
紅白の饅頭を各々何個準備しておけばよいか?
0825 【吉】 垢版2022/07/04(月) 00:12:59.26ID:PI9l4QYj
>>797
>>824
赤饅頭2022×0.333≒2022/3=674(人)
白饅頭674×2+3パー増しの61人で1348+61=1409(人)
別計算で白饅頭2022×0.696=1213.2+12.132+202.2-20.22
=1325.332+202.2-20.22
=1527.532-20.22
=1507.312
∴赤饅頭674個
白饅頭1508個
赤饅頭は選挙違反だろ?
0828132人目の素数さん垢版2022/07/04(月) 04:21:45.62ID:UmqisRMy
条件を満たすのは4通り

468/1406 = 0.3328592
468/1407 = 0.3326226
469/1407 = 0.3333333
469/1408 = 0.3330966

よって、
紅饅頭469個、白饅頭1408個
準備しておけば該当者全員に配布可能。

検証希望。方法は問わない。手計算でも計算機使用でもWolframでも可、山勘は無しで。
0829132人目の素数さん垢版2022/07/04(月) 04:25:40.03ID:UmqisRMy
>>813
東大卒の某氏はちゃんとレスしているね。
やはり、食べ物をネタにすると食いつきがいいwww
0830132人目の素数さん垢版2022/07/04(月) 06:14:53.87ID:KmsBka8/
アンケートの回答が得られてから準備するなら回答者数は既知なんだから計算不要
まったく実用的な問題ではない
0834132人目の素数さん垢版2022/07/04(月) 08:01:45.65ID:KmsBka8/
>>833
だからなに?
調査した側が饅頭を配るって話でしょ?
回答者数どころか回答者の住所まで知ってるでしょ
0835132人目の素数さん垢版2022/07/04(月) 18:55:58.68ID:R7wrQuH4
繰り返し質問失礼します。

a[0]=s,b[0]=t
a[n+1]=a[n]+b[n]
b[n+1]=a[n]b[n]
によって定められる数列a[n]およびb[n]の一般項は求められますか?
0837132人目の素数さん垢版2022/07/05(火) 21:23:41.28ID:zQeAe7xX
0.12481632641282565121024…
となる無限に続く2の倍数を小数点以下に延々とぶち込んだものは
無理数だと思うのですがそれを証明する方法ありますか?
無限に違う数字が続くから無理数だと直感的には分かるんですけど
証明方法が分からない。
0839132人目の素数さん垢版2022/07/06(水) 17:11:13.61ID:uYgeQm2e
p^2+q^2 ≠r^2
p・q ≠ 0

上式を満たす3整数 p、q、r に対し、

{(p/r)+ i・(q/r)}^n = 1

を満たす自然数 n が存在しないことを示せ。

帰納法を使わない証明はあるのでしょうか?
0841132人目の素数さん垢版2022/07/06(水) 18:30:09.04ID:hcUAgTfK
貼る画像間違えました
誰か解いて頂けますでしょうか?
よろしくお願いします
https://i.imgur.com/3ekaB6G.jpg
0842132人目の素数さん垢版2022/07/06(水) 18:35:29.18ID:HUvG9vlm
絵書いたら一発やん
0843132人目の素数さん垢版2022/07/06(水) 18:42:43.09ID:wuj/tPGF
確かに難しい問題だなと思ったら間違いか
0845132人目の素数さん垢版2022/07/06(水) 19:43:22.37ID:6l7c/beQ
解き方が解説されてるのに何が分からないのかが分からない
0846132人目の素数さん垢版2022/07/06(水) 19:48:12.91ID:787uID0V
繰り返し質問失礼します。

a[0]=s,b[0]=t
a[n+1]=a[n]+b[n]
b[n+1]=a[n]b[n]
によって定められる数列a[n]およびb[n]の一般項は求められますか?
0847132人目の素数さん垢版2022/07/06(水) 20:17:58.11ID:js2Xt3x3
0°≦θ≦180°のとき、次の不等式を満たすθの範囲を求めよ

tanθ(tanθ-1) < 0

解答
0<tanθ<1
0°<θ<45°

0<tanθ<1とありますが三角比の値の範囲は
tanθはすべての実数と書いてあり、どうして
こうなるのかわかりません。
宜しくお願いします。
0848132人目の素数さん垢版2022/07/06(水) 20:32:10.23ID:4duXPgjJ
>>847
x(x-1)<0 を解けば、0<x<1 になることは分かる?
0°≦θ≦180°の範囲で 0<tanθ<1 になるのは0°<θ<45°ってこと。
0849132人目の素数さん垢版2022/07/06(水) 20:49:13.54ID:js2Xt3x3
>>848
いまちょうどわかりました。
0にするために代入する数が0と1なんですね。
お手数かけさせてすみません。ありがとうございました。
0850132人目の素数さん垢版2022/07/06(水) 20:49:39.41ID:B4yWMUoK
>>839
左辺の絶対値は
|{(p/r)+ i・(q/r)}^n|

=|{(p/r)+ i・(q/r)}|^n

=|√{(p^2+q^2)/r^2}|^n

={(p^2+q^2)/r^2}^(n/2)
p^2+q^2 ≠r^よりnが自然数だと1にはならない。
右辺の絶対値は1
よって、両辺の絶対値が等しくならないから、
題意を満たす自然数nは存在しない。
0851132人目の素数さん垢版2022/07/06(水) 20:49:55.71ID:787uID0V
ある正整数a,bを用いて
a^2-b^2=p
と表せる素数の集合をSとする。
Sに含まれない素数をすべて求めよ。
0852132人目の素数さん垢版2022/07/06(水) 22:32:28.10ID:B4yWMUoK
a^2-b^2=p
(a+b)(a-b)=p
a,bは正の整数だからa+b、a-bも整数。
また、a+b>0であり、a+b.a-bである。
よって、(a+b)(a-b)=pが成り立つとき
a+b=p, a-b=1となる。よって
a=(p+1)/2、b=(p-1)/2
pが3以上の素数の時、pは奇数だからa,bは存在する。
よってSに含まれないのは2
0853132人目の素数さん垢版2022/07/06(水) 22:54:10.80ID:z2a93tJR
(n+1)^2-n^2=2n+1の時点で
ありとあらゆる奇数を表現できる以上
2以外のどんな奇数でもa^2-b^2で表現可能ってことか
0854132人目の素数さん垢版2022/07/06(水) 23:15:19.91ID:4duXPgjJ
>>853
子供の頃に読んだガリレオの傾斜板の実験の説明で、一定の時間間隔で斜面を円筒が
転がっていく距離xの比率をとると、奇数列1:3:5:7,,,になることから、x(t)∝t^2 っ
てのがあって、なるほど、n^2の階差数列は奇数の数列になってるな、と感心したこと
を思い出した。
0855イナ ◆/7jUdUKiSM 垢版2022/07/07(木) 02:24:15.85ID:hz3Ki0Ts
>>841
|x+1|=3x
xy平面においてy=x+1のグラフは、
(0,1),(-1,0)を通る直線だから、
第3象限の部分をx軸について線対称に返すと、
描ける。
y=3xのグラフは、
原点(0,0)と(1,3)を通る直線だから、
y=|x+1|と点(1/2,3/2)で交わる。
∴x=1/2
0856132人目の素数さん垢版2022/07/07(木) 08:35:56.17ID:qHwsVDiq
異なる素数p,q,rがp^2+q^2+r^2=990をみたすとき、p+q+rはいくらか。

これは試行錯誤で組合せを調べるしかないでしょうか。
0857132人目の素数さん垢版2022/07/07(木) 08:57:44.34ID:msabtZd0
1つが2なこと以外は基本的にそうだろうね
あとはmod3で絞ってゴリ押しなくらいか
0859イナ ◆/7jUdUKiSM 垢版2022/07/07(木) 11:10:31.54ID:OyPlCgaO
>>855
>>856
p^2+q^2+r^2=(p+q+r)^2-2(pq+qr+rp)=990
p+q+r=√{990-2(pq+qr+rp)}
11+q+r=√[990-2{11(q+r)+qr}]
(q+r)^2=990-121
=769
=27^2+40
(q+r)^2=990-169
=721
(q+r)^2=990-289
=601
(q+r)^2=990-(20-1)^2
=590+40-1
=629
(q+r)^2=990-529
=461
=21^2+20
(q+r)^2=990-(30-1)^2
=90+60-1
=149
=12^2+5
(q+r)^2=990-961
=29
∴p,q,rは2,5,31のいずれか。
p+q+r=2+5+31=38
いい年だ。
0860132人目の素数さん垢版2022/07/07(木) 21:09:14.83ID:OmMHTrwI
y=x^2を並行移動させたもので頂点がy=-2x+3上にあり、点(1、4)を通る放物線を求めよ
という問題なのですが教えていただけますでしょうか
0861 【大凶】 垢版2022/07/08(金) 00:53:59.06ID:XS7QX2rd
>>859
>>860
y=x^2のグラフをx軸方向にa,y軸方向にb並行移動させると、
y-b=(x-a)^2
頂点が(0,0)から(a,b)に並行移動するから、
頂点がy=-2x+3上にあるからb=-2a+3
並行移動した放物線上に点(1,4)があるから、
4-(-2a+3)=(1-a)^2
4+2a-3=1-2a+a^2
a^2-4a=0
a=0,4
a=0のときy=x^2+3
a=4のときy=(x-4)^2-2・4+3=x^2-8x+11
グラフを描くと2つあるとわかる。
∴y=x^2+3
y=x^2-8x+11
0864132人目の素数さん垢版2022/07/08(金) 09:21:20.97ID:LyS4zeMa
以下、プログラムおじさんの改変問題をお楽しみ下さい
0866132人目の素数さん垢版2022/07/08(金) 10:12:05.92ID:K8U3vuhd
>>856
p,q,rに2があるのは明らか。p=2としる。
q^2+r^2=986。奇数の平方の一の位は1か5か9。一の位「6」をつくるには5+1しかない。
よって5もある。q=5とする。r^2=961 よってr=31。
0867132人目の素数さん垢版2022/07/08(金) 10:32:24.39ID:LyS4zeMa
1の位が5である平方数は5以外には25があってその場合はr^2=361=19^2
0869132人目の素数さん垢版2022/07/08(金) 11:47:34.42ID:mzjebkqn
>>865

>842 名前:132人目の素数さん[] 投稿日:2022/07/06(水) 18:35:29.18 ID:HUvG9vlm
絵書いたら一発やん

に応じたきちんとTPOを弁えた作図である。
あんたが作図して投稿してもいいんだぞ。
ちなみにTPOは和製英語。
0870132人目の素数さん垢版2022/07/08(金) 12:00:17.59ID:mzjebkqn
>>856
Brute Attack(総当り)で算出。
可読性が悪いが一行でできる。
> sum(comboGeneral(Primes(floor(sqrt(990))),3)[apply(comboGeneral(Primes(floor(sqrt(990))),3),1, \(x) sum(x^2)==990),])
[1] 38

ちなみにp,q, rの組み合わせは
> comboGeneral(Primes(floor(sqrt(990))),3)[apply(comboGeneral(Primes(floor(sqrt(990))),3),1, \(x) sum(x^2)==990),]
[1] 2 5 31
0872132人目の素数さん垢版2022/07/08(金) 18:49:22.69ID:zeZYe8x+
>>871
まー、頭ん中で絵描けない奴もおるし、いいんじゃないの
0873132人目の素数さん垢版2022/07/08(金) 21:49:04.51ID:l2eHClOs
Σ[k=1,n] k^2 = 2^m
となるような正整数の組(n,m)が存在するならば、すべて求めよ。
0876132人目の素数さん垢版2022/07/10(日) 08:02:16.28ID:L2MSsXq+
ax^2+bx+c<0ってのを実生活で考える時ってどういう場面がありますか。

a<0なら2階からボールを投げた放物線みたいなイメージだと思うんですが、
a>0だと具体例がピンとこないもんで。
0877132人目の素数さん垢版2022/07/10(日) 18:54:15.71ID:i2BGBOwQ
「素数ってほとんど偶数だからね」
って言ったら笑われた

なんで? 2以外は奇数ばかりやん? ( '‘ω‘)
0878132人目の素数さん垢版2022/07/10(日) 19:09:34.62ID:i2BGBOwQ
>>876
その疑問はちょっとナンセンスだな、
関数を図にしたものが、たまたま、榴弾の軌道でそれを放物線と呼んでいるだけ。

その式は単に非線形について考えるための、
もっとも簡単な次数の関数を学ぶための例にすぎない。

a * xの2次 + b * xの1次 + c * xの0次 = z
0879132人目の素数さん垢版2022/07/10(日) 19:13:06.92ID:i2BGBOwQ
a > 0 の物としては戦力と兵数を表すランチェスターの法則がある。

例えば、 a = 1, b = 0 , c = 0 として
f(x) = x^2
↑ この式

これは兵士の数と戦力を表すのに使われる。
例. A軍兵士 10人 と B軍兵士 20人 がぶつかって戦争をする場合、
どちらかが全滅するまで戦い続けたとすると、どうなるか?
直感的には A軍 0人 B軍10人 … となりそうだがそうはならない。
(1vs1 のケンカを繰り返す訳じゃなく、1 vs 2 などの状況が局所的に生まれるため)

法則より 戦力は 100:400 と表される。
どちらかが全滅して戦力が0になるまで戦い続けたとすると
戦力は (100-100) : (400-100) = 0 : 300 となる。
これを再び兵士の人数に戻すと A軍 0人 B軍 √300 = 17人
となる。
実際に チンピラが10人と20人で抗争を起こした場合もこれに近い結果になる。
0882132人目の素数さん垢版2022/07/10(日) 22:40:23.66ID:+0Ga4QAm
>>879
>実際に チンピラが10人と20人で抗争を起こした場合もこれに近い結果になる。

根拠は?
0883132人目の素数さん垢版2022/07/10(日) 23:13:09.70ID:+0Ga4QAm
>>879
A軍の時点tにおける兵の数がa(t)、A軍の兵の武器の性能がp
B軍の時点tにおける兵の数がb(t)、B軍の兵の武器の性能がq、p/q=rとする
時点t=0で開戦して片方が全滅するまで戦い時点t=nでどちらかが全滅するとする

兵の損耗の速度がその時点における敵軍の兵数と敵軍の武器性能に比例する場合、
da/dt=-qb かつ db/dt=-pa だから db/da=ra/b ∫bdb=r∫ada
b(n)^2-b(0)^2=r(a(n)^2-a(0)^2)=r(0-a(0)^2) b(n)=√(b(0)^2-ra(0)^2) 
だからr=1のとき b(n)=√300≒17

自軍の兵の損耗の速度が敵軍の武器性能に比例する場合 da/dt=-q db/dt=-p だから
db/da=r b(n)-b(0)=r(a(n)-a(0)) b(n)=b(0)-ra(0) r=1ならb(n)=10

兵の損耗率をどう考えるかによって全く変わってくるわけだが
チンピラの抗争について前者のモデルが適合すると考える根拠がわからん
0884132人目の素数さん垢版2022/07/11(月) 12:06:24.28ID:hdnDmodd
>>877
ほとんどすべて偶数だし、ほとんどすべて奇数でもある
0885132人目の素数さん垢版2022/07/11(月) 18:09:15.21ID:mdPI6A8J
斜辺25で高さ7の直角三角形の底辺の長さを求める問題って
細長い直角三角形の斜辺って、どうせ底辺+1になるんだから
ズバッと24って出しちゃダメなんですか?
(´・ω・`)
0887132人目の素数さん垢版2022/07/12(火) 14:48:38.22ID:tcM5Pdmy
>>886
そこで偶数出すのセンスねーよ
0888132人目の素数さん垢版2022/07/12(火) 17:46:28.46ID:zPJLZ/zj
互いに素&少なくとも一つは4の倍数
であればズバッと底辺or高さ=斜辺−1
なんかの?
0889132人目の素数さん垢版2022/07/12(火) 18:05:30.30ID:zTLOA+1j
小泉進次郎
  「これ、意外と知られてないんですけど
    素数って、ほとんどが奇数なんですよね」

( '‘ω‘) ね
0892132人目の素数さん垢版2022/07/13(水) 01:40:47.15ID:rJoAQ6kC
斜辺と第二辺の辺長差が 1 というは、
(2n+1)^2 + (n^2+(n+1)^2-1)^2 = (n^2+(n+1)^2)^2
という恒等式を背景にしたものだが、それは、全原始ピタゴラス数の一部に過ぎない。

下に、原始ピタゴラス数の生成プログラムと出力結果が載っているので、確認してみると良い。

http://codepad.org/VKGibeHo
0893132人目の素数さん垢版2022/07/13(水) 11:20:57.78ID:ZSrjmoKe
>>892
斜辺と高さnの差が1となる直角三角形ならn=(m^2-1)/2 (mは奇数)で与えられ、底辺の長さはm
斜辺と高さnの差が2となる直角三角形ならn=4m^2-1 で底辺の長さは4m
斜辺と高さnの差が3となる直角三角形ならn=3(m^2-1)/2 (mは奇数)で、底辺の長さは3m
etc...
25,24,7よりも細長いのだと、
差が2の場合、m=4で65,63.16、m=5で101,99,20,m=6で...
差が3の場合、m=9で、123,120,27,m=11で183,180,33,m=13で...

といくらでもある。
0895132人目の素数さん垢版2022/07/13(水) 15:36:36.02ID:r0KB9lmq
a^10+b^100=c^1000
を満たす正整数の組(a,b,c)について考える。

(1)5^1000を3で割った余りを求めよ。

(2)このような組(a,b,c)は存在しないことを証明せよ。
0900132人目の素数さん垢版2022/07/14(木) 22:39:12.49ID:6bi50Sha
p,q,rは相異なる素数とする。
1/p+1/q+1/r = (pq+qr+rp)/pqr
は既約分数であることを証明せよ。
0901132人目の素数さん垢版2022/07/14(木) 22:57:22.98ID:t0cIrUqB
2000以下の素数の個数を求めよ,という問題は何年のどこの入試か教えてください.
0904132人目の素数さん垢版2022/07/15(金) 00:41:38.35ID:GN/sJjrW
n=1から∞の無限級数 Σ(sin(pi*sqrt(n))/n は収束しますか?
0905132人目の素数さん垢版2022/07/15(金) 01:25:43.98ID:Z8vqLjPP
>>900

与えられた分数が既約分数でないならば、pq+qr+rpはp,q,rのどれかで割りきれなければならない。
pq+qr +rp をpで割った余りはqr をpで割ったあまりに等しい。p,q,rは相違なる素数。よってpq+qr+rpはpの倍数ではない。同様にq,rの倍数でない。
以上から既約分数
0906132人目の素数さん垢版2022/07/15(金) 01:47:12.19ID:Z8vqLjPP
>>904
します。ラーベの収束判定法
0911132人目の素数さん垢版2022/07/15(金) 20:18:45.13ID:wBXU0ipP
>>908
この問題はフェルマーの定理とは関係ありませんよ
式の形をよく見てください
あと(1)が絶妙のヒントになっています、これだけでは難しいかもしれませんが…
0912132人目の素数さん垢版2022/07/15(金) 20:45:11.76ID:aHhsSQCb
>>911
自分で答えが分かってるのならここに書いちゃだめだろ。
ここは「質問スレ」であって出題スレじゃないよ。
頭悪すぎ。
0920132人目の素数さん垢版2022/07/16(土) 17:21:34.71ID:rk1MZq0v
1/√((n+1)^6+1) < 1/√(x^6+1) < 1/√(n^6+1)より
1/√((n+1)^6+1)< I[n] < 1/√(n^6+1)
したがって
√(n^6+1)/√((n+2)^6+1) < I[n+1]/I[n} < √((n+1)^6+1)/√((n+1)^6+1)=1
(左)=√(1+1/n^6)/√((1+2/n)^6+1/n^6) → 1
よってはさみうちの原理から1
0923132人目の素数さん垢版2022/07/17(日) 09:21:00.10ID:MQ0Va+Wo
未だにジンバブエとか言ってるあほがいるのか。
0924132人目の素数さん垢版2022/07/17(日) 09:24:48.67ID:wvVa84/o
>>921
そう思えるアンタのおめでたさが羨ましいよw
もう病気って言われるかもしれないけどなw
実際社会でもここでもゴミ扱いなのにw
0925132人目の素数さん垢版2022/07/17(日) 17:47:54.00ID:oUrQl+RZ
>>922
先週は麻酔2?件と内視鏡で20万円ゲット
んで、あんたは何の仕事をしてんの?
尿瓶おまる洗浄?
0927132人目の素数さん垢版2022/07/17(日) 21:05:38.81ID:i1TFJVKS
∫[-∞,∞] f(x) = 1 をみたし∫[-∞,∞] xf(x) dxが収束するような連続関数f(x)に対し、E(f) = ∫[-∞,∞] xf(x) dx と定める。
以下のf(x)に対しE(f)が収束するかどうか調べよ。

(1)e^(-x^2)
(2)log(1+x^2)*e^(-x^2)
0928132人目の素数さん垢版2022/07/17(日) 21:52:38.80ID:ndJNoTW0
こいつの出題に律儀に答える馬鹿がいるのかねぇ?
どんなアホ面してるのか顔が見てみたいわw
0929132人目の素数さん垢版2022/07/17(日) 21:57:47.10ID:i1TFJVKS
複素数平面上の円C:|z|=1上の2点A(α),B(β)に対し、点PをP(α+β)と定める。
A,BがC上を自由に動くとき、Pが動きうる領域の面積を求めよ。
0930132人目の素数さん垢版2022/07/17(日) 22:07:22.89ID:i1TFJVKS
相異なる実数r,s、および実数tに対し、r<p<sなる有理数で、|p-(r+s)/2|<1/tを満たすpを1つ構成せよ。
0931132人目の素数さん垢版2022/07/18(月) 04:12:34.82ID:JwRSDH3t
>>929
|z|<=2
0932132人目の素数さん垢版2022/07/18(月) 06:13:32.59ID:YAsqYHAi
>>926
んで、仕事は何してんの?
やっぱり尿瓶おまる洗浄業務に従事してんの?

昨日は救急搬送症例は軽症で入院にならなかったので
インセンティブが1諭吉1一葉にしかならなかった。
連休中にハイリスクを入院させると急変時の迅速対応が困難だからリスク回避できたと考えてよしとしよう。
0933132人目の素数さん垢版2022/07/18(月) 06:15:28.93ID:YAsqYHAi
>>923
半日バイトで8万円の稼ぎが羨ましいらしいね。
尿瓶おまる洗浄だと何日分にあたるんだろう?
0935132人目の素数さん垢版2022/07/18(月) 08:11:59.90ID:SvdbLJBg
フリーでスポットしまくるお医者さんのための情報交換スレ6
956 :卵の名無しさん[sage]:2022/02/17(木) 11:37:49.80 ID:wuk6JbHF
>>954
元、消化器外科医だよ。
俺の世代は麻酔もアンギオも外科医の仕事だった。
救急では消化管出血の止血とか緊急胆汁ドレナージとかやってた。

>緊急胆汁ドレナージ

>緊急胆汁ドレナージ

>緊急胆汁ドレナージ

自称元消化器外科医が緊急"胆汁"ドレナージww

こいつこんなことほざいてるただの脳内医者ですw
胆道(胆管)ドレナージという医療行為はありますが

ググればすぐ分かることですが胆汁ドレナージという言葉はありませんw
おそらく胆汁ドレナージバッグという商品名から勘違いしたのだと思われます、本当の医者ならありえないミスです
0937132人目の素数さん垢版2022/07/18(月) 09:13:59.65ID:IftbYfRQ
別に医者であろうがなかろうがそんなことはどうでもいいが、高校数学の質問スレで
数値計算の結果を紹介して意味があるケースはほとんど皆無でしょ。
そう指摘されてもあらためずにしつこく繰り返すから嫌われるんだよ。

頼むから他のスレで頑張ってくれ。現状ではスレ参加者に対する嫌がらせでしかない。
0940132人目の素数さん垢版2022/07/18(月) 11:41:26.57ID:tiAEuRMZ
>>939
PTCDもERBDも肝臓切離断端からの胆汁ドレナージも全部やったことあるぞ。
んで、あんた何の仕事してんの?
尿瓶おまる洗浄なんだろ?
0941132人目の素数さん垢版2022/07/18(月) 11:46:13.59ID:P8NNcIJ4
問題
労働時間は1日8時間、尿瓶おまる洗浄係の時給は最低賃金とする
どこに勤務しているか不明なのでどの都道府県に勤務しているかは人口に比例する重みをつけた確率分布を仮定する

下記のデータを用いて、
尿瓶おまる洗浄係が8万円を稼ぐためには何日働く必要がある期待値とその95%信頼区間を求めよ。


都道府県 人口 最低賃金
北海道 5381733 889
青森県 1308265 822
岩手県 1279594 821
宮城県 2333899 853
秋田県 1023119 822
山形県 1123891 822
福島県 1914039 828
茨城県 2916976 879
栃木県 1974255 882
群馬県 1973115 865
埼玉県 7266534 956
千葉県 6222666 953
東京都 13515271 1041
神奈川県 9126214 1040
新潟県 2304264 859
山梨県 834930 877
長野県 2098804 861
静岡県 3700305 858
愛知県 7483128 866
岐阜県 2031903 877
三重県 1815865 880
富山県 1066328 913
石川県 1154008 955
福井県 786740 902
滋賀県 1412916 896
京都府 2610353 937
大阪府 8839469 992
兵庫県 5534800 928
奈良県 1364316 866
和歌山県 963579 859
鳥取県 573441 821
島根県 694352 824
岡山県 1921525 862
広島県 2843990 899
山口県 1404729 857
愛媛県 1385262 821
香川県 976263 848
徳島県 755733 824
高知県 728276 820
福岡県 5101556 870
佐賀県 832832 821
長崎県 1377187 821
熊本県 1786170 821
大分県 1166338 822
宮崎県 1104069 821
鹿児島県 1648177 821
沖縄県 1433566 820
0944132人目の素数さん垢版2022/07/18(月) 16:35:41.62ID:6HJ+jg9L
つべのおすすめ動画でまさに
>>841のようなアンポンタンな事言ってるカスが多い」って言ってる動画流れてきたな
https://youtu.be/vz9cZnB1d1c
ちゃんと勉強した人間の威風は違うよ
>>841は永遠に統計学なんぞわからんやろ
0945132人目の素数さん垢版2022/07/18(月) 19:12:31.34ID:F480AssT
p,qは0≦p≦1かつ0≦q≦1の実数の定数である。
f(x)=|(x-p)(x-q)|+|x(x-1)|を考える。

(1)f(x)の最小値をm(p,q)とする。m(p,q)をpとqで表せ。

(2)p,qが動くとき、m(p,q)の最大値を求めよ。
0950132人目の素数さん垢版2022/07/19(火) 19:09:45.55ID:WhwAxw4Y
>>945
(1)
p<=qとする。絶対値を外すと
f(x)=2(x-(p+q+1)/4)^2+pq-(p+q+1)^2/8 {x<=0,1<x}
-(p+q-1)x+pq {0<x<=p,q<x<=1}
-(2(x-(p+q+1)/4)^2+pq-(p+q+1)^2/8) {p<x<=q}
となる。1/4<=(p+q+1)/4<=3/4であるから、x<=0で減少x>1で増加
また、p<x<qでは上に凸だから、f(p)とf(q)より小さい値はとらない。
p+q-1>=0のとき0<x<=p、q<x<=1で減少
よって最小値はf(1)=(1-p)(1-q)
p+q-1<0のとき0<x<=p、q<x<=1で増加
よって最小値はf(0)=pq
p>=qのときも同じように議論ができる。
対称性からm(p,q)はp+q-1<=0のときpq、p+q-1>0のとき(1-p)(1-q)

(2)
まずp+q-1<=0の時を考える。pを固定すると0<=q<=1-pである。
よってpqが最大となるのはq=1-pのときでp(1-p)
pを動かすと-(p-1/2)^2+1/4だから最大となるのはp=q=1/2で1/4である。
次にp+q-1>=0の時を考える。pを固定すると1-p<=q<=1である。
よって(1-p)(1-q)が最大となるのはq=1-pの時で(1-p)p
これは先ほどと同じ。
以上からm(p,q)の最大値はp=q=1/2で1/4
0951132人目の素数さん垢版2022/07/19(火) 19:24:07.45ID:Pxbm/o2e
a,bは実数の定数とする。
f(x)は実数係数の多項式で、
f(x)=∫[0,x] (at+b)f''(x) dt
を満たす。f(x)の次数を求めよ。
0952132人目の素数さん垢版2022/07/19(火) 19:58:24.44ID:GRIMHvJz
葉っぱのような形、アメフトのボールのような形、
レモンのような形、人の目のような形の図形を
幾何学用語で何と呼びますか?
ベン図の2つの円が重ね合わさった共通部分のところの形です。
0953132人目の素数さん垢版2022/07/20(水) 02:28:23.78ID:ZiC19WD1
>>952
日本語としては紡錘形かな
0954132人目の素数さん垢版2022/07/20(水) 11:39:41.19ID:TQ3/w9ND
mは1≦m≦99の整数とする。
100次方程式x^100+x^m+1=0が実数解をもつような、mが満たすべき条件を求めよ。
0957132人目の素数さん垢版2022/07/20(水) 14:31:06.16ID:eEvyI5cB
x=0以外では定義されず、x=0でのみf(0)=0と定義された関数f(x)は
x=0で微分可能ですか?
0959132人目の素数さん垢版2022/07/20(水) 15:16:18.85ID:TQ3/w9ND
>>957
x≠0で定義されてないんだから、微分の定義である
lim[h→0] {f(x+h)-f(x)}/h
がそもそも使えない
つまり微分ができない、微分が存在しない
0960132人目の素数さん垢版2022/07/20(水) 21:35:44.60ID:tj9M1Rc5
>>954
f(x)=x^100+x^m+1とおく。実数解が存在するためには、最小値が0以下でなければならない。
f'(x)=x^(m-1)*(100x^(100-m)+m)
(i) mが偶数の時
f'(x)はx=0で負から正にかわる。よって最小値はx=0の時で
f(0)=1>0 したがって実数解は存在しない。
(ii) mが奇数の時
f'(x)はx=(-m/100)^(1/(100-m))で負から正に変わる。-1<-m/100<0であるから
-1 <(-m/100)^(1/(100-m))< 0 である。
よってf((-m/100)^(1/(100-m)))> 0^100+(-1)^m+1=0だから実数解は存在しない。
以上から、実数解をもつようなmは存在しない。
0961132人目の素数さん垢版2022/07/21(木) 01:07:08.96ID:J9ItnJmt
>>954
[1] 0≦xの時 各項が非負なので、不成立
[2] -1<x<0の時 x^100>0,|x^m|<1 なので、不成立
[3] x≦-1の時 |x^100|-|x^m|>0 なので、不成立
0962132人目の素数さん垢版2022/07/21(木) 12:22:57.66ID:uJiTcJ6U
Oを原点とするxy平面において、円C:x^2+y^2=1と曲線D:y=1/x(x>0)の異なる2つの交点をP,Qとする。
∠POQと72°の大小を比較せよ。
0963132人目の素数さん垢版2022/07/21(木) 13:05:25.44ID:88zA5GvD
何をどうすればいいですかね
分母同じなので分子だけで考えても詰まっちゃって
0965132人目の素数さん垢版2022/07/21(木) 15:44:15.40ID:kGeC0K/c
>>962
原点と曲線D上の点A(s,t)との距離dは
d=√(s^2+t^2)=√(s^2+1/s^2)>=√(2√(s^2*1/s^2))=√2>1
よって原点を中心とする半径1の円Cと曲線Dは交わらない。
0966132人目の素数さん垢版2022/07/21(木) 16:34:18.80ID:Tn+nLrqM
質問です
70000グラムの物体が時速150キロで走る車から落ちた場合
どれくらいの重さが物体に掛かりますか?(何トンですか?)
力積Fなのは分かりますがいまいち計算式が分かりません
0967132人目の素数さん垢版2022/07/21(木) 17:27:46.61ID:uJiTcJ6U
ABを直径とする半円C上に点Pをとり、∠PAB=30°となるようにする。
またPAに関してBと反対側のCの弧上に、点QをQA=QPとなるようにとる。
∠QAPを求めよ。

という問題で、「円に内接する四角形の対角の和は180°」を使わずに答えを出すことはできますか?
0968132人目の素数さん垢版2022/07/21(木) 21:47:38.44ID:WUbqF3QL
2030年 イーサリアムの時価総額は20兆ドル、
ビットコインは28.5兆ドル=ARKインベストが予測

キャシー・ウッド氏が率いるARKインベストの新しいレポートでは、
イーサリアム(ETH)が今後10年以内に時価総額が20兆ドル以上になると
予測している。これは1ETHが17万ドル〜18万ドルになるということだ。
ARKのレポートではビットコイン(BTC)について、「国が法定通貨として
採用するにつれてスケールしていき、1ビットコインの価格は2030年までに
100万ドルを超えるだろう」と予測している。
0970イナ ◆/7jUdUKiSM 垢版2022/07/21(木) 23:54:08.07ID:ZMoyFrTs
>>902
>>962
円x^2+y^2=1とy=1/x(x>0)は交わらないから、
題意のP,Qは存在せず、∠POQも然り。
∴∠POQと72°の大小を比較することはできない。
0971132人目の素数さん垢版2022/07/22(金) 23:28:51.48ID:7akyji5Q
ビーカーAとBに食塩水が重量比1:6で入っている。
Bの食塩水の濃度はAの食塩水の濃度の2倍である。
それぞれのビーカーに100gずつ水を加えたところ、
Bの食塩水の濃度はAの食塩水の濃度の3倍になった。
最初ビーカーAに入っていた食塩水は何gか。


これは高校生でも手におえますか
0973132人目の素数さん垢版2022/07/23(土) 00:14:45.94ID:OhCJTnWd
最初ビーカーAにはいっていた食塩水をx[g]、濃度をy[%]とおく。
ビーカーBにはいっていた食塩水は6x[g]、2y[%]である。
題意から
3*{(x*y/100)/(x+100)}*100=(6x*2y/100)/(6x+100)*100
x=150
よって150g
0974132人目の素数さん垢版2022/07/23(土) 11:42:05.02ID:iQ4T4Qr0
最初は重量比が1:6で濃度が1:2なので食塩の量は1:12
それぞれに水100gを足した後は濃度が1:3で食塩の量は1:12なので重量比は1:4
最初のAの重量を①とするとBは⑥
足した後のAは①+100、Bは⑥+100
このときBはAの4倍なのでBは④+400でもある
⑥+100=④+400
②=300
①=150

小学生はこうやるんかな?
0975132人目の素数さん垢版2022/07/23(土) 14:43:06.19ID:MfZZI7YP
濃度a%の食塩水A100gと、濃度b%の食塩水B100gがある。
x,yを0<x≦50、50≦x<100、x+y=100の実数とする。
xgのAとygのBをとり、ビーカーに入れて100gの食塩水C[1]を作る。
また残った食塩水を混ぜ合わせ、100gの食塩水D[1]を作る。

以下同様に、xgのC[n]とygのD[n]をとり、ビーカーに入れて100gの食塩水C[n+1]を作る。また残った食塩水を混ぜ合わせ、100gの食塩水D[n+1]を作る。

食塩水C[n]の濃度をX[n](%)とするとき、極限値 lim[n→∞] X[n] をaとbで表せ。
0976132人目の素数さん垢版2022/07/23(土) 15:32:47.10ID:muVjlH44
>>975
なんでおまえの命令に従わないといけないんだよ。
質問スレで命令口調はやめろよ、礼儀知らず!
0978イナ ◆/7jUdUKiSM 垢版2022/07/23(土) 15:50:39.34ID:c4EZfr1g
>>970
>>971
Aをx g濃度c %とすると、
題意より0.0cx/(x+100):0.0cx×12/(6x+100)=1:3
4(x+100)=6x+100
300=2x
∴x=150(g)
0979132人目の素数さん垢版2022/07/23(土) 19:39:19.17ID:OhCJTnWd
C[0]=A、D[0]=Bとする。よってX[0]=aである。また、D[n]の濃度をY[n]とおく。
漸化式を立てるとn>=0で
X[n+1]=x/100*X[n]+y/100*Y[n]
Y[n+1]=(100-x)/100*X[n]+(100-y)/100*Y[n] となる。
よって X[n+1]+Y[n+1]=X[n]+Y[n]=X[0]+Y[0]=a+b が成り立つ。
したがって、X[n+1]=x/100*X[n]+y/100*(a+b-X[n])
X[n+1]-y(a+b)/(100-x+y)=(x-y)/100*(X[n]-y(a+b)/(100-x+y))
X[n]=y(a+b)/(100-x+y)+((x-y)/100)^n*(100a-ax-by)/(100-x+y)
|(x-y)/100| <1より
lim[n→∞] X[n] = y(a+b)/(100-x+y)
0981132人目の素数さん垢版2022/07/24(日) 00:37:49.17ID:PL8LlYzJ
>>979
訂正
100-x=y より
y(a+b)/(100-x+y)=y(a+b)/(2y)=(a+b)/2
0982132人目の素数さん垢版2022/07/24(日) 09:48:15.30ID:THiGXUs6
全ての素数は6k±1で表せると聞いたのですが、本当なのですか?あまり釈然としません
もし表せるのだとしたら、これを自明として解に用いていいのですか?
0983132人目の素数さん垢版2022/07/24(日) 10:23:36.68ID:QmfskIz8
簡単なんだから、自衛のためにも証明しておけば
0984132人目の素数さん垢版2022/07/24(日) 10:26:33.16ID:KZVf/Htm
>>982
自然数は6k, 6k±1, 6k±2, 6k+3のどれかになって6k±1以外は2か3の倍数
だから
2,3以外のすべての素数は6k±1で表せる
2,3はそうは表せないのを忘れないのと
これを自明として良いかどうかは説明を受ける側の判断次第
ま、心配なら2,3の倍数ではないのでと断るだけで十分と思うが
0986132人目の素数さん垢版2022/07/24(日) 11:02:10.28ID:zVZEuYYX
・正の整数全体を6で割った余りで分類すると
6k→合成数のみ 6の倍数
6k+1→1と素数と合成数
6k+2→2以外全て合成数 2の倍数
6k±3→3以外全て合成数 3の倍数
6k-2→全て合成数 2の倍数
6k-1→素数と合成数

・6で割った余りで分類すると
6の倍数、2の倍数(2種類)、3の倍数、それ以外(2種類)の6種類になり、まとめると
定理 : 2, 3以外の全ての素数は6k±1の形をしている。
0988132人目の素数さん垢版2022/07/24(日) 16:44:37.92ID:BFt/ksfH
ttps://uploader.cc/s/5t0w7809isq15jc7nptgxganioin529gzk9qdxxxc4zp7bvuua1kf0t3wc74stmj.jpg
この画像が示している計算の間違いをご指摘願います
0989132人目の素数さん垢版2022/07/24(日) 17:58:04.20ID:yuchowMk
複素数の指数法則は一般的には成り立たない。
0990132人目の素数さん垢版2022/07/24(日) 18:12:10.36ID:cmGlze3T
>>988の補足
xy平面上で (-40,3)、(-12,3)、(-10,3)、(0,3)、(12,3)、(18,3)、(20,3) を満たす関数の形として
「y=3・e^i880πx」が与えられたもの。

そもそもxy平面の連続関数を考える時に「y=3・e^i880πx」は妥当なのかどうか、という話。
0993132人目の素数さん垢版2022/07/24(日) 23:54:07.24ID:sVjRUiIG
>>990
要するに e^2πix =(e^2πi)^x =1^x =1 になっちゃうよー、っていう話でしょ?
指数関数の底は1以外ってことにしとけば1^xとはおけないってことでどですかね?
>>989と同じことか。
0994132人目の素数さん垢版2022/07/25(月) 12:31:07.10ID:InFtII4b
a[1]=√2として、漸化式|a[n+1]-2]|≦1/2|a[n]-2| の証明は
|a[n+1]-2]|<1/2|a[n]-2|を示せば十分みたいなこと言われたんですが本当ですか

あと、参考書なんかには√(a[n]+2)≧0であることを用いて
|a[n+1]-2]|≦1/2|a[n]-2| を示していたのですが
そもそも√(a[n]+2)>0であって 、√(a[n]+2)≧0の等号は成り立たないと思うのですが
0995132人目の素数さん垢版2022/07/25(月) 13:28:45.09ID:gGNAQEe4
>>994
質問の意図がよくわからんが、A>BならばA≧Bは必ず成立するので、前者は十分条件になってる。
0996132人目の素数さん垢版2022/07/25(月) 15:00:27.68ID:DvOeP9iE
>>995
a[1]=√2として、漸化式|a[n+1]-2]|≦1/2|a[n]-2| の証明が問題となっていて
先生が|a[n+1]-2]|<1/2|a[n]-2|を示せば良いって言ってたのですが、なぜそれでいいのか分かりません
0997132人目の素数さん垢版2022/07/25(月) 15:06:48.89ID:DvOeP9iE
>>995
すみません、分かりました!
変な勘違いをしてました
0998132人目の素数さん垢版2022/07/25(月) 18:33:40.90ID:TmhLOn1e
趣味としての高校数学はコスパがかなりいい
ような気がしないでもない
(´・ω・`)
1000132人目の素数さん垢版2022/07/26(火) 07:13:00.32ID:XU5L9sv8
日本の学部卒止まりゼネラリスト気取りの程度の低さの根源が学部入試受験数学の程度の低さ。
10011001垢版Over 1000Thread
このスレッドは1000を超えました。
新しいスレッドを立ててください。
life time: 66日 8時間 26分 18秒
10021002垢版Over 1000Thread
5ちゃんねるの運営はプレミアム会員の皆さまに支えられています。
運営にご協力お願いいたします。


───────────────────
《プレミアム会員の主な特典》
★ 5ちゃんねる専用ブラウザからの広告除去
★ 5ちゃんねるの過去ログを取得
★ 書き込み規制の緩和
───────────────────

会員登録には個人情報は一切必要ありません。
月300円から匿名でご購入いただけます。

▼ プレミアム会員登録はこちら ▼
https://premium.5ch.net/

▼ 浪人ログインはこちら ▼
https://login.5ch.net/login.php
レス数が1000を超えています。これ以上書き込みはできません。

ニューススポーツなんでも実況